2ちゃんねる ■掲示板に戻る■ 全部 1- 最新50    

■ このスレッドは過去ログ倉庫に格納されています

スレッドを立てるまでもない質問スレッド Part 373

1 :名無しさん@英語勉強中 :2022/06/16(木) 19:28:38.06 ID:eDmJztfL0.net
■ 質問する方へ
・常識的な内容なら何を質問してもOKです。
・できるだけ分りやすく、具体的に質問して下さい。ソースがあれば必ず書いて下さい。

■ 回答する方へ
・YAHOO!知恵袋のコピペを使った嫌がらせが後をたたないので、回答の前に、
 知恵袋に同じ質問が出ていないか、質問文の中のワードでググってみたほうが賢明です。
・回答する方は、質問者の立場になって答えてあげましょう。

■ 辞書
・単語や熟語の意味なら辞書で調べた方が速くて正確です。
 研究社 - ルミナス英和・和英辞典
 http://www.kenkyusha.co.jp/modules/08_luminous/index.php?content_id=1
 無料で引けるオンライン辞書(英英辞典) - おさんぽ英語 おすすめ散歩道
 https://osanpo-english.com/site-book/online-dictionary.html

■ ChMateなどのアプリを使っていて、音声入力で英語を書き込みたいときは:
・(Androidの場合) 書き込む→工具マーク→入力方式の切り替え→音声入力に切り替え 。
・(iOSの場合) キーボードを表示させて左下のマイクのアイコンをタップする 。

過去スレ

スレッドを立てるまでもない質問スレッド Part 371
https://lavender.5ch.net/test/read.cgi/english/1645843745/

スレッドを立てるまでもない質問スレッド Part 372
https://lavender.5ch.net/test/read.cgi/english/1649681478/

2 :😉三年英太郎🌈 :2022/06/16(木) 19:29:47.45 ID:eDmJztfL0.net
>>2

> テンプレではありませんが過去スレで問題提起がありましたので転載します
> (このスレ内でレスしている様に見えない様、アンカーのみ削除させていただきました)
>
> 881 名無しさん@英語勉強中 (オッペケ Sr03-crGe)[sage] 2020/10/12(月) 18:48:11.75 ID:BiNVjerEr
> 例文貼りまくり厨はマジ自重すべき
> 今このスレ、モバイルで開けらんない時あるぞ
>
> 924 名無しさん@英語勉強中 (ワッチョイ 9fa1-AMwI)[] 2020/10/15(木) 07:54:22.61 ID:oZ2vtNXk0
> ・同じ用法に関する例文は2つまで
> ・文法的な解説以外の、英語の新聞や雑誌の記事などを「同じ単語や表現が出てくるから参考になる」という理由で貼るのは避ける

3 :名無しさん@英語勉強中 :2022/06/16(木) 19:54:12.33 ID:YUmXPSUBd.net
スレ伸びてるからって人が多いわけじゃないからな。
↓実態はこういう事だからみんな乞食にエサやらないようにね。

0005 連投規制になったら他の板に応援要請するけどいい? 2019/05/06 10:22:34
(中略)
実は、2ちゃんねるは、あらかじめ書かれた原稿を自動書込ソフトによって流し、
本物の書込だけ、その板の2ch運営スタッフが誹謗中傷、煽りなどにより相手し、
アクセスを増やす自作自演のヤラセ掲示板なのです。
(後略)

0009 連投規制になったら他の板に応援要請するけどいい? 2019/05/06 10:42:37

『2ちゃんねるはなぜ潰れないのか?』
↓検索
「プロ固定 悪質手法」
プロ固定は、壮絶な自作自演を繰り返す事によって、
あたかも多数のユーザが行っているように見せかけ、
何も知らないユーザらを巻き込んで行きます。
<プロ固定&プロ名無し>
(2chのサクラ、専業の煽り屋)
(後略)

4 :名無しさん@英語勉強中 :2022/06/16(木) 21:01:10.49 ID:7twOsNjp0.net
前スレ>>998の回答どなたかお願いします

5 :名無しさん@英語勉強中 :2022/06/16(木) 21:31:48.78 ID:xiLwyU2Ar.net
>>1
Otts.

>>4
I want to watch the video of a baseball tournament.

6 :Seeker2022 :2022/06/16(木) 22:51:12.66 ID:GOUfNRtN0.net
>>4

I would like to watch videos of baseball tournaments on YouTube.

7 :名無しさん@英語勉強中 (ワッチョイ 2329-GGE9):2022/06/17(金) 16:50:12 ID:dwxu1zBI0.net
I didn't recognize you with your mask on!
マスクをつけているから、あなただと気付かなかった!

with以降がなんとなく謎ですが、
with your putting mask on

の省略と考えればいいのでしょうか?

8 :名無しさん@英語勉強中 :2022/06/17(金) 18:25:21.14 ID:hg6NVqvvM.net
英検準2級の長文問題をしていますが、1問を精読すると2時間かかってしまうのですがかかりすぎていますか?
わからない単語、熟語があるたびに調べて単語カードに書いています。

9 :名無しさん@英語勉強中 :2022/06/17(金) 18:40:57.86 ID:qnZbfNJGM.net
>>8
俺は面倒臭いから分からない単語か表現だけマーカーで引いて全訳か解説で該当部分に印付けてるくらいしかしないな。その後気が向いた時にたまに読み直す程度。
英検トイクなんて別に下線部和英訳ある訳じゃないからそこまで必要無いでしょ。下手すりゃ単語帳すら要らんと思う。
辞書引っ張り出して調べる時間は極力減らしたいから参考書は全訳と解説に重要表現や需要単語訳がピックアップされてる奴の方が良い。

10 :Seeker2022 :2022/06/17(金) 19:37:53.99 ID:wshZLRPy0.net
>>7
This will help you.

https://global-square.com/blog/ancillary-situation-with/

11 :名無しさん@英語勉強中 :2022/06/17(金) 20:02:08.09 ID:GLiYBgvp0.net
仮定法過去でIf ...were..., ...would(could)....てのありますけど、
would couldじゃなくてmayを使う場合はmightにしないとだめですか?

12 :名無しさん@英語勉強中 (ワッチョイ 9bf0-HA6y):2022/06/18(土) 02:54:14 ID:HwxDNsRI0.net
Like most TV pro-Trumpers, Kellyanne dominates the conversation,
with a non-stop barrage of well-rehearsed, antidemocratic,
politically supercharged messages. Bill tries but he is just not the same as before.

pro-Trumpersとはどういう意味でしょうか?

13 :名無しさん@英語勉強中 :2022/06/18(土) 04:03:55.03 ID:aJUD0tIl0.net
>>11
もちろん

>>12
トランプ支持者じゃないの

14 :名無しさん@英語勉強中 :2022/06/18(土) 06:14:03.78 ID:Fa01tOOf0.net
>>13
リンク貼ったら規制されるので控えるんですけど、
なんかヤフー知恵袋のq1157214531で論争がでてるんですよね

15 :名無しさん@英語勉強中 :2022/06/18(土) 06:36:00.51 ID:aJUD0tIl0.net
今見たけど、論争になってるというか、普通にいやmightでしょみたいな感じで話終わってない?

16 :名無しさん@英語勉強中 :2022/06/18(土) 06:37:56.53 ID:Fa01tOOf0.net
そうかもです
ありがとうございます

17 :名無しさん@英語勉強中 :2022/06/18(土) 13:45:46.53 ID:BM/ph0yf0.net
スレ伸びてるからって人が多いわけじゃないからな。
↓実態はこういう事だからみんな乞食にエサやらないようにね。

0005 連投規制になったら他の板に応援要請するけどいい? 2019/05/06 10:22:34
(中略)
実は、2ちゃんねるは、あらかじめ書かれた原稿を自動書込ソフトによって流し、
本物の書込だけ、その板の2ch運営スタッフが誹謗中傷、煽りなどにより相手し、
アクセスを増やす自作自演のヤラセ掲示板なのです。
(後略)

0009 連投規制になったら他の板に応援要請するけどいい? 2019/05/06 10:42:37

『2ちゃんねるはなぜ潰れないのか?』
↓検索
「プロ固定 悪質手法」
プロ固定は、壮絶な自作自演を繰り返す事によって、
あたかも多数のユーザが行っているように見せかけ、
何も知らないユーザらを巻き込んで行きます。
<プロ固定&プロ名無し>
(2chのサクラ、専業の煽り屋)
(後略)

18 :名無しさん@英語勉強中 :2022/06/19(日) 05:10:13.98 ID:arvA/7It0.net
よく名詞の前にaをつけるかtheをつけるかは、話者と聞き手が共通認識しているもの(同時に指で示せるもの)はtheでそれ以外はaという説明をされますけど嘘ですよね?

例えば「昨日道でAさんと会った」を英語で言うときにも、
I met A on the street yesterday
とthe streetというのが普通で、聞き手はどの道かなんてわかりっこないのにtheを使いますよね

いったいどういうことなんですか

19 :名無しさん@英語勉強中 :2022/06/19(日) 05:56:25.60 ID:OHUmB9tB0.net
道っていうより外でっていう漠然とした意味だから

20 :名無しさん@英語勉強中 :2022/06/19(日) 06:09:35.67 ID:ANt+FFYN0.net
>>18
おはようございます。
私の覚え方ですが、「あなたと私の間でさっき話題になっていたものを指すthe=vと
「あなたは知らないかもしれないけど、それは構わずにみんな普通に知っているものを
指すthe=vに大まかに分けています。
他にも用法はあるようですが、この2パターンでほぼ乗り切れています。

21 :名無しさん@英語勉強中 :2022/06/19(日) 07:27:10.52 ID:ANt+FFYN0.net
>>20
今回のI met A on the street yesterday≠ニいう文だったら、話者がthe streetの
補足説明をするかもしれないし、相手がWhere?≠ネんて尋ねたりするかもですね。
一応自分のレスに補足しました。

22 :名無しさん@英語勉強中 :2022/06/19(日) 07:41:41.66 ID:jwuqNtYA0.net
>>18
そんな話ははじめて聞きました。
特定の街路と言う意味でtheは付けますよ。
えいが「My Fair Lady」には、On the Street Where You Live
という有名は歌が出てきます。
theの使い方は従来の説明どおりです。

23 :名無しさん@英語勉強中 :2022/06/19(日) 07:49:01.52 ID:jwuqNtYA0.net
>聞き手はどの道かなんてわかりっこないのにtheを使いますよね
その文脈では、話者同士がふだん通っている街路という言外のニュアンスが
含まれていると思いますよ。つまり限定された範囲の街路を指しているんでしょう。

24 :名無しさん@英語勉強中 :2022/06/19(日) 08:20:23.23 ID:u48SLXLp0.net
石田の冠詞本の1冊くらいは読んでおくことをおすすめする

25 :名無しさん@英語勉強中 :2022/06/19(日) 10:33:33.47 ID:kYlJuykoM.net
「ネイティブの感覚で冠詞が使える」

26 :名無しさん@英語勉強中 :2022/06/19(日) 14:43:35.50 ID:arvA/7It0.net
>>23
それは違うと思いますよ
アメリカ人と話してても私はアメリカになんていったことないし、アメリカでどんなスーパーがあるかなんて知らないのにそのアメリカ人は「昨日スーパーでりんごを買ったんだけど」みたいな話をするときにも the supermarketて言いますよ

>>20さんの説明がしっくり来ますね
なんで関正生先生はこういうとこまで突っ込んで教えてくれないんですかね

27 :名無しさん@英語勉強中 :2022/06/19(日) 15:12:45.91 ID:OHUmB9tB0.net
それはいつものって意味

28 :Seeker2022 :2022/06/19(日) 22:41:23.02 ID:xhPNbwP30.net
>>26
He lived on the street.

この英語が分かりやすいでしょう。この英語は、「彼はその特定の通りで暮らしている」の意味の他に、「彼は路上生活者だ」という意味もあるのです。
このtheは「特定」を示すtheではなく、「概念全体」を意味するthe なのです。「彼は通りと呼ばれる概念に相当する場所で暮らしている」という意味になります。

似たものに、She likes to swim in the sea. があります。このtheも特定の場所をささず、「一般に海と呼ばれている場所のどこか」という意味です。
theは、「概念全体」(~と呼ばれるもののどこか)という意味で使われることもあるのです。

参考
https://english.stackexchange.com/questions/225219/why-not-on-a-street

29 :Seeker2022 :2022/06/19(日) 22:54:20.68 ID:xhPNbwP30.net
I met him on the street.の場合は、イディオム化していて、9割程度の確率で、不特定の場所を表し、「彼と街で会った」程度の意味になります。

30 :名無しさん@英語勉強中 :2022/06/19(日) 23:23:52.29 ID:MNZXmypA0.net
1 a piece of furniture
1つの家具

2 a single of furniture
たった1つの家具

3 a single piece of furniture

3は2と何が違う?
頭痛が痛いみたいな同じこと2回言ってね?みたいな言葉に見えるけど

31 :名無しさん@英語勉強中 :2022/06/20(月) 16:10:10.00 ID:QzyaTZb00.net
>>28
そういえばin the morningのtheも概念全体を表していると考えれば納得はできますね
ありがとうございます

「「せ〜の…」で皆が一斉に指をさせるものにthe を使うのです。」なんて無茶苦茶なこと教えてる関正生先生は何なんですか
i met him on the streetといわれて聞き手がどのstreetかわかりっこないのに
なんでこんな指導方法がまかり通ってるんですかね

32 :😉三年英太郎🌈 :2022/06/20(月) 17:19:19.09 ID:y9VPEOEM0.net
おかしなことゆーとるで、自分ら🤭

33 :名無しさん@英語勉強中 (ワッチョイ 3b96-MKW/):2022/06/20(月) 17:36:29 ID:QzyaTZb00.net
おかしいならちゃんとした説明してくれよおお

34 :名無しさん@英語勉強中 :2022/06/20(月) 18:19:08.89 ID:zeZArc5a0.net
>>31
今回の「theの慣用表現」(で良いかな?)手元にある文法書をざっくり見直してみました。
「一億人の英文法」が一番分かり易かったかな?
「フォレスト6th edition」、「英文法の鬼100則」、「青ロイヤル」ともにこの件については
サラッと触れているだけでした。そして「aとtheの底力」に期待して目次を探したの
ですが、なぜか見つからなかったですね。私の探し方が悪い可能性大です。
やっぱり意外と学習者になじみがないのかなと思います。
そして、件の「世界一わかりやすい英文法の授業」ですが、私も心に響かなかったです。
分かったような分からないような、消化不良な感じです。
ということで、一億人の英文法を一度ご覧になってみては?

35 :名無しさん@英語勉強中 (ワッチョイ 1af0-KtZt):2022/06/20(月) 18:53:48 ID:ZjxDZEc+0.net
>>30

2 a single of furniture
たった1つの家具

こんな英語ないわ

36 :名無しさん@英語勉強中 :2022/06/20(月) 19:20:51.15 ID:efcD+TmG0.net
「概念全体」を指しているの説明で分かる人は勿論、それで十分なんだけど、
the に特有の「(あるものを)特定する」意味合いをも考えて、

その文脈によって示唆される「あるカテゴリー」に含まれる複数のものの中の一つ
(「the 単数」の場合)を、他との対比の中で特に指していると考えてもいい。

上の例で言うと、morming , afternoon, evening などの中の一つ。
sea なら、river , pond ・・・・・場合によっては mountain なども・・・

身体の部位をカテゴリーととらえるとhand, leg, face, shoulder, head などが
その一つになる。

live on the street なら、bench とか house , building、 abandoned house
hotel ・・・・(前にくる前置詞はそれぞれで変わるが・・・・)

自分はこの考えでずっとやってきたが・・・・・飛躍しすぎてたら・・・ごめん

37 :三年英太郎 :2022/06/20(月) 20:00:30.29 ID:y9VPEOEM0.net
>>33

「theの用法は同定可能という原理だけで説明可能です」
樋口昌幸 『英語の冠詞』

もう少し大きく分けると、3つに分けられる。

1.話し手と聞き手の共通認識
  (聞き手は知っているだろう、という話し手の期待を含む)
2.総称のthe
3.慣用表現

seekerのいう「概念全体」とやらは2のことを言ってるのだろうが、
on the street も in the morning も3で理解した方がいい。
(おかしな独自理論で理解しない方が、その後の学習が楽)

38 :三年英太郎 :2022/06/20(月) 20:01:27.82 ID:y9VPEOEM0.net
訂正

× もう少し大きく分けると、3つに分けられる。
〇 少しだけ細分化すると、3つに分けられる。

39 :三年英太郎 :2022/06/20(月) 20:18:53.96 ID:y9VPEOEM0.net
in the morning は樋口的にいえば、限定された期間だからということになろうか。
が、もともとはそうだったとしても、今は慣用表現と見た方がようだろう。
CaGEL p.408 にあり。

路上生活~ という時は、on the streets(複数)の方が普通らしい。
the street で「概念全体」を表すなら、どうして複数形が必要なのか?

40 :名無しさん@英語勉強中 :2022/06/20(月) 20:32:40.72 ID:QzyaTZb00.net
>>37
>>26のスーパーマーケットの件はどうなるの?
それも慣用表現ってこと?
俺が知りもしないスーパーなのに、
I bought apples at the supermarket
みたいにアメリカ人に言われたんだけど?

41 :😉三年英太郎🌈 :2022/06/20(月) 20:38:09.51 ID:y9VPEOEM0.net
(聞き手は知っているだろう、という話し手の期待を含む)

42 :😉三年英太郎🌈 :2022/06/20(月) 20:42:42.97 ID:y9VPEOEM0.net
ホントに聞き手が知ってるかどうかという事実は重要でない

聞き手もわかってんだろ、という話し手の態度が決定事項なのである

43 :名無しさん@英語勉強中 :2022/06/20(月) 21:12:18.33 ID:uaOu+C8g0.net
変な本ばっか読んでないで栄太郎みたいに樋口読むとか、石田読むとかしなさい

44 :Seeker2022 :2022/06/20(月) 22:31:42.55 ID:Nv4QONug0.net
>>39
「ふつう」とまでは言えない。
>>28のリンク先でネイティブが
「The similar usage is when people say He lived on the street. They are not necessarily referring to an actual street (or group of streets) but they actually mean he did not have a home to live in.」と言っている。
誰もそれに反論・言及していない。「ふつう」と言ってはいけない。

>>37
慣用表現にも由来があるのである。
「met him on the street」(街で会った)がなぜ出てきたのか語源を考えてみよう。
1.話し手と聞き手の共通認識
2.総称のthe
のどっちから出てきたのか? 2から来ていると私は判断する。そこを説明したまで。なぜそのような不特定のtheの慣用表現が生まれたのかと言えば、「総称のthe」(~というもの・概念全体)が基になっていると考える。
「慣用表現だからそれで納得しなさい。由来は聞くな」というだけでは、人を納得させるには不十分である。2じゃない由来があるのであれば、ぜひ挙げてみてほしい。

45 :名無しさん@英語勉強中 :2022/06/20(月) 22:37:34.29 ID:QshWw2j50.net
さすがです
ここは定冠詞をつけて
ザ英太郎さんと

46 :名無しさん@英語勉強中 :2022/06/20(月) 22:42:12.17 ID:k0kyl9H20.net
貴方の配信が観たいです
ってなんて言うんでしょうか?

47 :名無しさん@英語勉強中 :2022/06/20(月) 22:47:52.87 ID:XVVo+4d90.net
>>46
直訳ならi want to watch your webcast.
回りくどい言い方したいならyour webcast is a long time comming.

48 :名無しさん@英語勉強中 :2022/06/20(月) 22:49:51.07 ID:XVVo+4d90.net
×Your webcast is a long time comming.
○To meyour webcast is a long time comming.
こうか、すまん

49 :Seeker2022 :2022/06/20(月) 23:47:49.09 ID:Nv4QONug0.net
一応、補足。
met him on the streetに関して、私は>>29でこれはイディオム(慣用表現)だと言っているように、「慣用表現である」というのが最終結論なので、誤解なきよう。

50 :三年英太郎 :2022/06/21(火) 00:11:40.05 ID:Rw6RSfN80.net
>>44
on the streets
1. homeless
(ODE)

(idiom) on the streets
1. homeless
(NOAD)

on the street(単数)もOALDには出ている。

> 「総称のthe」(~というもの・概念全体)が基になっていると考える。

それ、初出年代をちゃんと調べたうえで言ってんの?
総称のthe+複数形は、nations しかダメってCaGEL(p.407)にあるけど、
どーゆー理屈で複数化したの?

51 :Seeker2022 :2022/06/21(火) 00:27:01.38 ID:kc7GoPC40.net
>>50
on the street だと、「ホームレスの意味にふつうならない」ってどこかに書いてあったの?
>>28でふつうに使われてるから、「ふつうならない」は間違いだよ。

>複数化したの?
私が書いた例文は全部単数形なんだけど・・・。 どこjから「複数形」が出てきたの?
「the+複数形」なんて一言も言ってないぞ。

まず、「met him on the street」の慣用表現がなぜ生じたかを説明してみて。
自分は、「総称」が由来ではないかと考えている。(あくまでも私の持論)
それに対する反例が挙げられるなら、大歓迎だ。ぜひ挙げて欲しい。

52 :三年英太郎 :2022/06/21(火) 00:35:26.88 ID:Rw6RSfN80.net
とりあえず the をつければ総称(抽象)表現になると思っているのがseekerの大きな間違いやわ。
どの文法書にも出てるが、そんなに万能には使えないとゆーこと。

****************************

The boundaries of this usage with singular nouns(※総称のthe+単数) are somewhat indeterminate, but it is clearly facilitated in the contest of spicies, inventions, and areas of study, interest, or expertise.

Compare, for example,
*The hospital doctor is overworked.
with
The hospital doctor is an engangered species round here

(CaGEL p.407)

****************************

んで、英語の一般的なルールとして、具体物から抽象化するときには、
theがつくどころかその反対で、むしろ無冠詞になるとゆーことを忘れてる。

53 :三年英太郎 :2022/06/21(火) 00:39:28.44 ID:Rw6RSfN80.net
>>51
「腹が立つ」という慣用表現が存在する事実に対して、
なぜ「腹が立つ」という慣用表現ができたかを説明する必要はない。

わかるかな?

54 :三年英太郎 :2022/06/21(火) 00:45:43.04 ID:Rw6RSfN80.net
>>52

×The hospital doctor is an engangered ...
〇The hospital doctor is an endangered ...

55 :Seeker2022 :2022/06/21(火) 00:48:21.11 ID:kc7GoPC40.net
on the streets
1. homeless
(ODE)

これをいくら列挙しても、「on the streetに『homeless』の意味がない」ことの証明にはならない。
どちらでも良いというケースなので。

まとめ
meet O on the streetは、イディオム(慣用表現)で、>>29に書いたように、「街で会う」という程度の意味である。
なお、このthe streetのtheは「総称のthe」に由来するものと私は推測する。

He lived on the street.が「彼は路上生活者(ホームレス)だ」という意味になるとき、これもイディオムである。
このtheも、「総称のthe」に由来するものと私は推測する。

もし、他のまったく異なる由来があるのであれば、ぜひ挙げて欲しい。大歓迎である。

56 :😉三年英太郎🌈 :2022/06/21(火) 00:56:50.01 ID:Rw6RSfN80.net
おれが突っ込むまで、推測とか一言も言ってなかったじゃん🤭

>>29もさあ、9割ってどこから出てきた数字なん?

一言で言って雑🤭

57 :Seeker2022 :2022/06/21(火) 01:04:26.70 ID:kc7GoPC40.net
>>52
現代英文法じゃなく、由来だからね。語源辞典を調べないと・・・
現代英語では、イディオム(慣用表現)が正解だから。

on the streetのtheの由来が書いてある文献じゃないと何を言っても無意味なのを理解してほしい。

>>53
それって、「すべての慣用表現は、語源を語る価値はない」と言ってるのと同義なんだが ^^

>>56
>>28のリンク先で「When people say I met him on the street, they are usually not referring to a specific street.」ってあるから、私の「9割説」と一致してるよね。雑に感じるのは、>>56の理解不足からきているんだよ ^^

58 :名無しさん@英語勉強中 :2022/06/21(火) 14:41:55.84 ID:PdbkwqY80.net
The population of this county ~

という文があったのですがin this countryではないのですか?違いを教えて下さい

59 :😉三年英太郎🌈 :2022/06/21(火) 19:06:04.78 ID:Rw6RSfN80.net
ある時は「イディオムだから、で納得するな」(>>44)と言い、ある時は「イディオムだからそれ以上は説明できない」(>>57)という。

当然、語源辞典なりを当たった上で由来を説いてるのかと思えば、「調べないと…」「文献を示して欲しい」と、なんとこちらに調べろと言う。

長文先生にもその雑さを最大限皮肉って突っ込んでやったものだが、ついに治ることはなかった。けだしこれは、生まれながらの気質なのだろう🙄

60 :名無しさん@英語勉強中 :2022/06/21(火) 20:32:45.79 ID:Ir3yFxcWa.net
>>58
"population of ~"で"~の人口"を表す定番の表現。"population in ~"だとofほど結合が強くなくて、"population" + "in ~"で"~での人口"って感じじゃないかな。
意味はほぼ同じでどちらを使っても良さそうだけど、ofの方がかなり使われてる感じ。

参考
https://netspeak.org/#q=population+of+%7C+population+in

61 :名無しさん@英語勉強中 :2022/06/21(火) 20:50:09.00 ID:N+Eln1Alr.net
>>59
ワロタ

62 :名無しさん@英語勉強中 :2022/06/21(火) 20:57:55.08 ID:JuKAgYs10.net
ああギークの爆笑劇場か

63 :Seeker2022 :2022/06/21(火) 22:19:55.73 ID:kc7GoPC40.net
>>59
まーた、でっちあげる。「イディオムだからそれ以上は説明できない」なんて一言も言ってないぞ。
ちゃんと「総称のtheの可能性が高い」という持論を展開してるし。
ソースがなくても、持論を展開する自由はあるべきだし、それにより、反証が出てくれば、掲示板閲覧者のためにもなる。
これは、「弁証法的方法で真実に近づく」という、議論の場ではとても大切なプロセスなんだ。

まあ、そのプロセスにより、英太郎は私との議論で、on the street と on the streets、両方が正しいことを学べたし、met him on the streetがふつうは特定の通りを指さないと学べたんだから、弁証法的真理接近法は否定してはいけないんだよ。

64 :Seeker2022 :2022/06/21(火) 22:21:15.95 ID:kc7GoPC40.net
>>62
私はギーク氏じゃないから、誤解なきよう。

65 :😉三年英太郎🌈 :2022/06/21(火) 22:40:15.62 ID:Rw6RSfN80.net
>>63
on [the] street が、総称の the に由来するというなら

1. なぜ、ホームレスの意の場合には on the streets (複数)がより一般的なのか。総称の the は、ほとんどの場合に単数と共起する(>>50)のではないか。

2. live/meet on the street も swim in the sea も "in the contest of spicies, inventions, and areas of study, interest, or expertise." (>>52)ではないが、どうして元は総称だと言えるのか。それを支持する研究を挙げよ。

66 :Seeker2022 :2022/06/21(火) 23:21:58.45 ID:kc7GoPC40.net
>>65
1について
「on the streets (複数)がより一般的」だというソースは? まあ、それは置いておいて・・・
theというのは、それをつけると「もう他に要素はないよ」という意味を持っているのである。

例) These are cars that he has. ←ほかにも別の所に「車を持っている」ことを示唆する
   These are the cars that he has.←他にもう車は持っていないことを意味する

このようにtheをつけると「彼が持っている車のすべて」を指せるのである。「全部」を示すtheなのである。
文法書では「~というもの」と訳すtheを特に「総称のthe」と呼んでいるのであるが、実は「~というもの」と訳さないtheであっても「全部をさす」ことができるのである。

ゆえに、「ぜんぶを指すthe」という点においては、単数形であろうと複数形であろうと関係ないのだ。
「on the streets」と「on the street」は文法分類上違って見えるかもしれないが、両者とも「全部を指すthe」なのである。

67 :Seeker2022 :2022/06/22(水) 00:11:42.85 ID:UcyqZ1gP0.net
>>65
前レスでも述べた通り、theというのは、基本的に「他にはもう該当するものがない時」「全部を指す」場合に付けるのである。
これは、文法上「総称のthe」に分類されていなくても、「総称的意味」を出せることを意味する。

例 the water on the earth →地球上に存在する水の総称 
   the sea on the earth →地球上の海の総称
    ※「穏やかな海」の場合、すべての海を指せないので、theをつけることはできず、「a calm sea」となる。
     (正しい)This is a calm sea in the world.
the sea は、「他に海はない」つまり自動的に「海全体を指せる」のである。

   on the streetに関しては、
(正) The apple is a highly nutritious fruit. (リンゴというものは~だ)←総称
   この用法が大昔はいろいろな名詞に広く使えていたなごりではないかと予測する。予測なのでソースはないが、他に理由が見当たらない。
   もし、万一、別の考えがあるのなら挙げてみてほしい。そこから、弁証法的進歩が始まるのだ^^

68 :名無しさん@英語勉強中 :2022/06/22(水) 00:20:27.71 ID:rP1wKpELa.net
低学歴の質問で申し訳ありません。
everyone says that suzuki kanon is the fat of the group but thay cant see that mizuki is also fat thats why she got big boobs because of her fatness.
という文章を見つけたのですが後半部分がいまいちよくわかりません。
下品な内容で申し訳ありません。

「みずきが巨乳なのは彼女が太っているからということを彼らはわからない」という文章なんでしょうか?
それとも「みずきが太ってることを彼らはわからない」で一度文章が切れて「彼女が巨乳なのは太ってるからだ」と追記してる形なんでしょうか?
thats why と because of で繋げる形は英語圏では一般的でしょうか?
英語でも日本語でもいろいろ検索したのですが、「thats whyとbecauseの意味」が近すぎており
「thats whyとbecauseの違い」を説明したものばかりヒットしてしまいます。申し訳ありません。

69 :😉三年英太郎🌈 :2022/06/22(水) 00:46:33.33 ID:dWIiWba60.net
>>66
>文法書では「~というもの」と訳すtheを特に「総称のthe」と呼んでいるのであるが、実は「~というもの」と訳さないtheであっても「全部をさす」ことができるのである。

ぷぷー🤭
総称のtheが「全部」とか爆笑もんやわ🤭
塾講師失格やで🤭🤭🤭

表現英文法より
https://i.imgur.com/Z0Ix3lz.jpeg
https://i.imgur.com/fiQlF1p.jpeg

70 :😉三年英太郎🌈 :2022/06/22(水) 01:05:08.18 ID:dWIiWba60.net
seeker先生向け🤭
https://toiguru.jp/generic-reference

71 :名無しさん@英語勉強中 :2022/06/22(水) 01:05:16.19 ID:vTiVyPTf0.net
表現英文法持って単価

72 :68 :2022/06/22(水) 01:44:17.61 ID:92tqZQCPa.net
正確な訳が知りたいというより、
文法的な知識とか英語圏で一般的な表現なのかとかを知りたいです。

73 :😉三年英太郎🌈 :2022/06/22(水) 02:06:09.33 ID:dWIiWba60.net
Everyone says that suzuki kanon is the fat of the group, but they can't see that mizuki is also fat. That's why she got big boobs because of her fatness.

万が一、英語ネイティブが書いてるとしたら統失ですわ

74 :名無しさん@英語勉強中 (ワッチョイW e39d-wiep):2022/06/22(水) 04:03:20 ID:UBM7D/ju0.net
しょうもない

75 :名無しさん@英語勉強中 :2022/06/22(水) 11:49:59.31 ID:dsTWcaZN0.net
>>58 については、>>60 の言う通りだと思う。ちなみに、
the population of XXX in YYY
っていう言い回しも使われている。

例:
The population of Whites in Harris County, TX (Whites + Hispanics) 3,989,796.

こういう例文をいろいろ読んでいるうちに、
the populatoin of XXX in YYY
と書いてあれば、XXX にはどういう名詞が来て、YYY にはどういう名詞が来て、
of と in ではここではどういう意味合いで使われているのか、どういう違いがあるのか
なんて、他人に尋ねなくても自分でわかるようになる。

76 :名無しさん@英語勉強中 :2022/06/22(水) 12:34:05.38 ID:dsTWcaZN0.net
>>72 >>68
もう少しわかりやすく書き直すとしたら

everyone says that suzuki kanon is the fat of the group
--> Everyone says that Suzuki Kanon is the fattest of the group.

but thay cant see that mizuki is also fat
--> But they can't see that Mizuki is also fat.

thats why she got big boobs because of her fatness.
--> And her fatness makes her look busty.

77 :68 :2022/06/22(水) 12:34:09.03 ID:92tqZQCPa.net
>>73
ちなみにあなたは英語ネイティブですか?
失礼な質問で申し訳ありません。
たくさんの人に「いいね」されていて、誰にも英語間違ってるぞ!と指摘されていません。
https://haropro-confessions.tumblr.com/post/124496347383/everyone-says-that-suzuki-kanon-is-the-fat-of-the
thats why と becauseで繋げる形、実は英語圏でネットスラング的に使われてるものだったりしませんか?
thats why と because の意味が似通り過ぎてるため、 ネットで検索してもなかなかこの文法を説明する記事などにはヒットしませんが。

78 :68 :2022/06/22(水) 12:36:49.42 ID:92tqZQCPa.net
>>73
失礼な言い方になってしまって申し訳ありません。
返信してくれるのがあなただけだったので、あなただけが頼りでした。
(質問のレベルが低すぎて、全員からスルーされるレベルなのかな?と思ってしまいました。)
他にも返信してくれる人がいたようなので先ほどのレスは無視してくださって構いません。

79 :68 :2022/06/22(水) 12:41:11.31 ID:92tqZQCPa.net
>>76
返信ありがとうございます。
普通の日本人の英語力でもそういうことを言いたいのであろうと想像つくのですが、
私が聞きたいのは、thats why と because of で繋げるような文法の形が存在するのかということです。
英語圏の人や英語に精通してる人しか知らないようなネットスラング的な言い回しなのかも?と気になりました。

80 :68 :2022/06/22(水) 12:46:55.95 ID:92tqZQCPa.net
>>76
あと the fat of the group は the fattest of the group と言い換える必要あるんでしょうか。
The Good, the Bad and the Ugly みたいに the fat だけで太ってる人みたいな印象になる気もします。
but thay cant see that mizuki is also fat の部分も同じですし、
結局は最後の文法の部分だけが問題になってると思われます。
レベルの低い返信で申し訳ありませんが、最後の部分の文法が英語圏などで見られる形式なのかどうかが一番知りたいです。

81 :😉三年英太郎🌈 :2022/06/22(水) 12:51:28.82 ID:dWIiWba60.net
非ネイティブが書いたデタラメな文だっつーに
英語添削フォーラムでもあるまいに、そんなこといちいち間違ってるとかゆーうるさい人はいないの

82 :😉三年英太郎🌈 :2022/06/22(水) 12:56:31.53 ID:dWIiWba60.net
ん?🤔
僕は人類のために死ねるー!って言ったあと、反出生主義に転向したファッション哲学おじいさんが戻ってきてる?🤔

83 :😉三年英太郎🌈 :2022/06/22(水) 13:18:57.21 ID:dWIiWba60.net
50歳「20年間引きこもってわかった。情報ばかり見てると『人間は愚かだ』から抜けられなくなる」 [373226912]
https://greta.5ch.net/test/read.cgi/poverty/1654423608/

世を呪うおじいさんに捧ぐ

84 :名無しさん@英語勉強中 :2022/06/22(水) 14:37:28.89 ID:XesI3zBX0.net
>>83
呪詛というより
達観じゃね?

85 :名無しさん@英語勉強中 :2022/06/22(水) 14:42:46.89 ID:nvrYRUlv0.net
>>60
サンクス

86 :😉三年英太郎🌈 :2022/06/22(水) 15:34:57.67 ID:+6HH2TuXM.net
>>84
立花孝志マンセーからの反出生主義だから、どうだかね🤭

安楽椅子からの快適生活支えてんの誰よ?
ネットでポチれば洋書が届くのは自然現象か?

自給自足生活してからそういうこと言いなさいよ

87 :名無しさん@英語勉強中 :2022/06/22(水) 15:49:59.68 ID:vTiVyPTf0.net
今はホームレスやっても食にだけは困らんらしいな
何なら収入も月数万はある

88 :😉三年英太郎🌈 :2022/06/22(水) 16:40:46.38 ID:dWIiWba60.net
ホームレスとは言わないまでも、出家とか、せめてネット断ちでもしたら、「あのおじいさんの思想は本物だったか」とワイも涙を流して😢改心するのだが…

今のところ、SNS大好き・人的資源活用しまくりおじいさんだからね😥
そんな人の反出生主義ってファッションでしょ?としか…😥

89 :名無しさん@英語勉強中 :2022/06/22(水) 18:12:51.91 ID:+JsnAcVX0.net
i see potential.って意訳するとワンチャンあるよであってんのかな?

90 :Seeker2022 :2022/06/22(水) 19:10:15.46 ID:UcyqZ1gP0.net
>>69 ついに、三年ちゃん反論できなくなって、暴れだしましたね。(笑)
 自分の言葉で説明できないのなら、もっと早くから降参しておけばいいのに・・・。

では、最終兵器をだしましょう。「on the street」のtheが「通り」という概念全体を指す働きをしていることの証拠をだすとしましょう。

「(不特定の)通り[路上]で遊ぶな」
Don't play on the street. (正) ※streetの概念全体を指すthe
Don't play on a street. (誤)

「私はある(不特定の)にぎやかな通りを歩いていた」
I was walking on a busy street. (正) 
I was walking on the busy street. (誤)
 ※「そのにぎやかな通り」と訳せる場合は、on the busy streetは正しい。

三年ちゃん、なぜ、形容詞が付くと、いきなり形勢逆転する(theが使えなくなる)のか説明できる?

91 :Seeker2022 :2022/06/22(水) 19:15:31.30 ID:UcyqZ1gP0.net
90の解説

streetにbusy(形容詞)が付くと、streetが意味する場所の「にぎやかな」部分のみを指す→「streetの概念の一部」であるため、もう「概念全体を指すthe」が使えなくなるのだよ!
以上より、「(不特定の)通りで」の「on the street」のtheが、「概念全体を指すthe」であることが立証されたも同然です。めでたし、めでたし。

これでもまだ、根拠なく、「概念全体を指すthe」じゃないと言い張る人がいたなら、きっと・・・いや、やめておきましょう ^^

補足
私が言っている概念全体を指すtheというのは、「~というもの」と訳さないという点で、文法書で言う「総称用法」とは異なるものである。
単に、「他に該当するものはない」というtheが元来持っている本質的意味合いを応用させたものに過ぎない。従って、文法書の「総称用法」の説明を引用しても無意味である。

三年ちゃん、わかったかな?^^

92 :Seeker2022 :2022/06/22(水) 19:18:51.26 ID:UcyqZ1gP0.net
>>69
塾講師ならこの時間にレスできないんだが・・・
また、でっちあげる・・・ 人をおとしめるためにでっちあげしちゃあダメよん

93 :名無しさん@英語勉強中 :2022/06/22(水) 19:24:48.49 ID:DO933x/aa.net
ざこ鬼が上弦の弐に血戦を挑んでおる

94 :Seeker2022 :2022/06/22(水) 19:33:23.72 ID:UcyqZ1gP0.net
>>93
だから、理論で反論してください。罵倒しかできないなんて、恥ずかしくないの?
議論においては、「誰が言ったか」よりも「何を言ったか」のほうがずっと大切なのだ。前者で判断する人は、学者には向いてないと思う。

95 :😉三年英太郎🌈 :2022/06/22(水) 19:46:06.25 ID:dWIiWba60.net
独自解釈に逃げた🤭
無価値なので読まないが

seeker先生の最近の書き込みを晒そう

**************

A "It's been a while since he died."
B "It's been a while since he has died."
この場合、Aのみが正解になります。

https://lavender.5ch.net/test/read.cgi/english/1649817763/353
**************

96 :Seeker2022 :2022/06/22(水) 19:56:50.47 ID:UcyqZ1gP0.net
>>95
独自解釈でも間違っていれば自分なら突っ込むよ。読まない?・・・そりゃ、反論できないので、「読まないことにしておこう」となるのはわかる ^^

ついに昔の問題を出してきちゃったのね・・・ これは、前に説明したけど、「Bも特例であり得るけど、英語の試験ではAのみが正解になる」という意味だよ。そして、それは間違ってないよ。

まあ、この問題はここまでにしましょう。

97 :😉三年英太郎🌈 :2022/06/22(水) 20:01:34.05 ID:dWIiWba60.net
>>95の英語も満足に読めない人の独自理論を、丁寧に読むバカいる?🤦🏻‍♂🤭

98 :Seeker2022 :2022/06/22(水) 20:03:07.04 ID:UcyqZ1gP0.net
補足

英語の試験は、「もっとも正しいものを選べ」という文句がふつう付いてるので、

A "It's been a while since he died."
B "It's been a while since he has died."

AがBよりもふつうであり、Aを正解にしたほうが絶対いいです。
Bは「死んだ時間のほうにあまり意識がない」という意味合いで標準的ではないので、減点される可能性が高いです。

99 :Seeker2022 :2022/06/22(水) 20:05:55.23 ID:UcyqZ1gP0.net
>>97
いや、学習するために読むんじゃなく、反論するために読むわけで^^
私ならアホ論があったら、喜んで読んで反論するぞ

アホ論を学習するために読んだらアホだけど、反論するために読むならまっとうでしょう

100 :名無しさん@英語勉強中 :2022/06/22(水) 20:37:37.90 ID:XesI3zBX0.net
>>86
元のスレ見てわかったことスレタイが言いたいことは
人間は愚かだ、じゃなくて、
言いたいことは、
自分以外の他人は皆愚かだ、ということ

外に出て人と話してわかったことは、
他人を必ずしも馬鹿と決めつけちゃいけないということ。
中には馬鹿じゃない人も居るかも、ということ。
可能性を否定しちゃダメだってこと。

oedさんが、立花派から反出世主義に転じてとか何かを信じた
のは多分、自分自身にとって都合が良かったから


人はそんなもの。だって基本馬鹿なのが人間というものだから。


安楽椅子での快適生活なんて僕はしてない。この椅子には
低反発クッションを置いてる。元は結構座りにくいよ。
安物椅子。拾ってきた椅子なんだ。この前に持っている椅子は
木の椅子。全然安楽椅子とは言えない。

101 :名無しさん@英語勉強中 :2022/06/22(水) 20:38:38.02 ID:XesI3zBX0.net
ネットでポチれば小説だって届くけど、それは僕が
Amazonでkindle 本をポチったから。光フレッツを使ってるから
小説が届いた。他のものも送料なしで届く。これはクレカとか
アマプラに入っているから。

それぞれの人は自分がやるべきことだけやってる。自分のやりたいことだけ
やってる人も居るだろうな。大抵仕事が好きなはずだ。

自分のことだけ頑張ってれば良いんだよ。自分のやりたいことだけやりたい。
ブルーハーツの甲本さんもそれがロッカーの生き方だって言ってた

102 :名無しさん@英語勉強中 :2022/06/22(水) 20:53:44.72 ID:XesI3zBX0.net
みんながロッカーになってやりたいことだけやる
というふうになればもっと世の中は楽しいものになると思うよ

103 :名無しさん@英語勉強中 :2022/06/22(水) 20:54:24.28 ID:XesI3zBX0.net
このスレは質問スレでした。ごめんなさい。すれ違いなことを書きました

104 :名無しさん@英語勉強中 :2022/06/22(水) 21:55:55.35 ID:poVLQm0/0.net
"I needed to study for 3 days without sleeping to pass the exam."という英文は、
「試験に受かっていること」と「3日間寝ずに勉強したこと」は現在の事実として
達成されていないと成立しないでしょうか?
それとも、実際には試験に落ちていて、試験に落ちた言い訳のようなニュアンスでも
成立しますか?

105 :三年英太郎 :2022/06/22(水) 21:58:12.25 ID:dWIiWba60.net
>>101
ワイの言いたいことちゃうで~

ポチっとクリックすりゃ本が届くのは、背後で膨大な人的資源が動いてるから

そーゆー人間の構築してきたシステムをフルに享受してるくせに、なにが反出生やねん

半世紀以上生きて、行きついた先が中二病かよ!

106 :😉三年英太郎🌈 :2022/06/22(水) 22:07:37.66 ID:dWIiWba60.net
>>104
例えば自分は実際に死んでなくても
I died that day.と言えるでしょ。
I meant my soul died!😢

文の最終的な意味を決定するのは文脈である🧐

107 :名無しさん@英語勉強中 :2022/06/22(水) 22:58:49.67 ID:XesI3zBX0.net
>>105
雑談スレに移りましょう
以降はそちらで

108 :名無しさん@英語勉強中 :2022/06/23(木) 21:58:48.59 ID:dXCNYthK0.net
SNSに絵を上げたら海外の方からSunshine!とコメントが来た
なんとなく褒められてるんだろうなとは思うんだけどよくわからない&返事の仕方がわからない
Thank youとかでいいんだろうか

109 :名無しさん@英語勉強中 :2022/06/24(金) 12:47:27.24 ID:h+bc5BFE0.net
You are my sunshine, my only sunshine♪
You make me happy when skies are gray♪
You'll never know, dear, how much I love you♪
Please don't take my sunshine away♪

110 :😉三年英太郎🌈 :2022/06/24(金) 17:49:17.39 ID:Ktp1mLmV0.net
すんタヒね
といった可能性もなきにしもあらず🤔

111 :😉三年英太郎🌈 :2022/06/24(金) 20:06:21.11 ID:Ktp1mLmV0.net
>>66
https://i.imgur.com/DWSt6pS.jpeg
https://i.imgur.com/bLxCe82.jpeg

オツムの出来の悪いseeker先生に理解できるか不安だが😔そのtheも結局は同定可能で説明できるのである。決して「全部を示すthe」ではない。

112 :名無しさん@英語勉強中 :2022/06/24(金) 22:24:52.13 ID:pbcz3xKI0.net
Asteria for Business(Z会)を経験された方いますか
続けやすさや実際の到達点?を教えて頂けると嬉しいです

113 :Seeker2022 :2022/06/24(金) 23:34:44.56 ID:iUDvfNA/0.net
>>111
虎の威を借るキツネのように文献に依存し、自分の頭で考え説明することができない三年ちゃん。たまには自分の言葉で説明しようよ^^
説明できなくなると「罵倒しかできない」愚かな反応。とても、科学的態度とは言えないな。
(これは一般論だが、文系の学問<特に暗記系>は側頭葉で知識を得れば前頭葉を使わなくてもある程度は行けるから真の知能がなくてもそこそこ成功できるのだ。)

では、三年ちゃんにもわかるように説明してあげよう。
「同定可能」と「全部を表す」は、実は表裏一体で、必ずしも別々のものではいのだよ。わかる?
「同定可能」と「全部を表す」を完全に別扱いしている点で、三年ちゃんは私から見ると「理解が浅く」見えるんだ。

もっとわかりやすく言わないとわからない?なら、教えてあげる。
on the streetのtheは、「通りという概念を全部指してる」と私は言ったけど、それって裏を返せば、住んでる街の通り全部であると「同定」できるんだよ。

The animals in this parkは、「同定可能」であると表現できると同時に、「公園にいる動物全部」を表現できるということ。
「同定可能だから全部を指すものではない」という三年ちゃんの理論は、的外れもいいとこなんだよ。「同定可能」をもって、「全部を指す」を否定できるという単純な話じゃないんだよ。分かったかな?

114 :Seeker2022 :2022/06/24(金) 23:55:48.49 ID:iUDvfNA/0.net
補足

the water on the earthは、「地球上に存在するところの水」と同定可能だが、見方を変えると「地球上の水全体」を指しているとも言える。
このthe をある方向から見ると「同定可能」であり、別の方向から見ると「全体・全部」なのだ。

こんなことまで説明しないとだめなのかなあ^^

115 :😉三年英太郎🌈 :2022/06/25(土) 00:06:37.60 ID:7xo1d/fp0.net
どんどんオレの方に寄せてくるねえ🤣

116 :名無しさん@英語勉強中 :2022/06/25(土) 00:30:59.32 ID:xtPHlGWy0.net
>>113
いや、partitionというものについて説明させてくれ。
Ωの部分集合Aで互いに素なものの系をaと書こう。
この時aをpartition と呼ぶとする。その中にA1, A2
があるなら、A1やA 2はそれぞれ同定されるが、
A1やA2はΩではない。

これを持って同定可能と全体性を持つことはちがうという
ことがわかった。

117 :Seeker2022 :2022/06/25(土) 00:46:22.41 ID:2t02YKF80.net
>>115
「寄せてくる」・・・またまたでっちあげの印象操作する~

私は、一度も三年ちゃんの「同定可能」という言葉を否定したことはないのだ。両者表裏一体とすでに知っていたから。


>>116
The books on the desk are mine. のThe booksは、同定可能である。
116は机の上に私のではない本もある可能性があると言っているようだが、それはあまりにも英語を知らなすぎる。
このtheは、机の上にある本はすべて私の本であることを示している。

机の上に私のではない本もある可能性がある論拠を述べよ。

118 :Seeker2022 :2022/06/25(土) 00:57:39.95 ID:2t02YKF80.net
The books on the desk の the booksは、同定可能と同時に、必ず「机の上の本すべて」をさしているのである。

119 :名無しさん@英語勉強中 :2022/06/25(土) 01:44:03.22 ID:xtPHlGWy0.net
というか机の上に、二固まりの本があるとすると
the booksと言わずに、
these booksというから、一固まりしか本はないかな。
文脈次第だよね。
同定可能性と全体性が一致するのは

120 :名無しさん@英語勉強中 :2022/06/25(土) 01:46:34.17 ID:xtPHlGWy0.net
seeker さんはoed does not love me on earth.
さんですか?

121 :名無しさん@英語勉強中 :2022/06/25(土) 01:48:18.82 ID:sx7DpPEo0.net
〜and will help hide the sensitive data we are transceiving.
この文章ってhelpとhideがどちらも動詞で重複しているように見えるんだけど、これって問題ないの?

122 :名無しさん@英語勉強中 :2022/06/25(土) 02:13:18.36 ID:xtPHlGWy0.net
He helped me (to) move the table

無問題

123 :名無しさん@英語勉強中 :2022/06/25(土) 02:18:30.87 ID:sx7DpPEo0.net
なるほど、to不定詞の省略形取れ見れるのか
回答ありがとうございます

124 :名無しさん@英語勉強中 :2022/06/25(土) 02:19:54.76 ID:+JCAGhsA0.net
>>119
意味不明

125 :名無しさん@英語勉強中 :2022/06/25(土) 02:20:01.34 ID:qf/I1fPo0.net
help 原形動詞で「-するのを助ける」「-するのに役に立つ」という頻出表現だ
動詞の連続は他にも、go grab a coffee や come visit us again など珍しくない

126 :名無しさん@英語勉強中 :2022/06/25(土) 02:24:55.68 ID:sx7DpPEo0.net
やっぱりvintageには載ってなかったですね、最近vintageが物足りなく感じることが多くなってきたのですが、次にやるべき教科書・参考書でおすすめのものはありますか?

127 :名無しさん@英語勉強中 :2022/06/25(土) 02:25:04.39 ID:qf/I1fPo0.net
let go of something ってのも動詞の連続だな

128 :名無しさん@英語勉強中 :2022/06/25(土) 09:40:48.87 ID:0MbHIbOBa.net
原形=原形不定詞

129 :名無しさん@英語勉強中 :2022/06/25(土) 09:58:43.58 ID:94JsU/3V0.net
help 原形動詞はvintageに普通に載ってる

130 :名無しさん@英語勉強中 :2022/06/25(土) 10:45:12.67 ID:0MbHIbOBa.net
>>126
参考書もいいけど辞書買って都度調べる習慣付けた方がいいよ
コンパスローズとかおすすめ

131 :名無しさん@英語勉強中 :2022/06/25(土) 10:49:22.27 ID:rhh2qGin0.net
ネットで調べたら発音音声付きで語訳パターンも載ってるから辞書を買うメリットは無いと思うけどな

132 :名無しさん@英語勉強中 :2022/06/25(土) 10:50:26.13 ID:2oNsNQUrr.net
辞書を持ってなくても辞書を引けるようになるのは大事かと
今回のだってhelpで調べれば秒でわかる案件だし

133 :名無しさん@英語勉強中 :2022/06/25(土) 11:24:17.76 ID:m242Qj6T0.net
教えてください。
We do this to ensure the site remains scammer free.

「そのサイトが詐欺師が自由に残る事を確実にするため私達はこれをします。」と間違えて訳をしてしまいます。

remain 残る、free自由は他の訳し方があるのでしょうか?
正しい訳は「詐欺師から守るために」です。

134 :名無しさん@英語勉強中 :2022/06/25(土) 11:26:55.18 ID:94JsU/3V0.net
>>133
fat freeとかduty freeとか聞いたことない?

135 :名無しさん@英語勉強中 :2022/06/25(土) 11:28:49.19 ID:2oNsNQUrr.net
>>133
remain *-free で辞書を引きましょう

136 :名無しさん@英語勉強中 :2022/06/25(土) 11:31:52.33 ID:xtPHlGWy0.net
カフェインフリーとか
Intelフリーとか

137 :名無しさん@英語勉強中 :2022/06/25(土) 11:34:39.03 ID:xtPHlGWy0.net
-freeで出てくるけど、正規表現で辞書で検索するときってどうするんだろう?

138 :名無しさん@英語勉強中 :2022/06/25(土) 11:39:31.40 ID:xtPHlGWy0.net
>>137は解決しました。問題なしです

139 :名無しさん@英語勉強中 :2022/06/25(土) 11:41:18.94 ID:rhh2qGin0.net
今ググったらensureは○○からブロックするとか、○○から守ると訳される事もあるみたいね。

140 :名無しさん@英語勉強中 :2022/06/25(土) 12:58:50.73 ID:yYNz9PTg0.net
文字通りの意味と解釈としての意味

141 :名無しさん@英語勉強中 :2022/06/25(土) 13:10:01.00 ID:/jKpnCiI0.net
ビッグ・ファット・キャットの世界一簡単な英語の大百科事典という本のストーリーに
ついて質問があります。
この話の最後で、主人公が子供のときに結婚すると約束した女性にデートを申し込むシーンが
あるのですが、この女性はすでに子供がいるのでしょうか。
読んでて割と衝撃を受けたのですが和訳が書かれていないため読み間違いかと思い質問しました。

142 :名無しさん@英語勉強中 :2022/06/25(土) 15:11:30.32 ID:W2z0geh+a.net
The clown is one of the key person.
His appearances can help to keep the show smoothly and fun.

これ文法的に正しいですか?
1文目peopleで 2文目はsmoothかなと思ったんですが

143 :133 :2022/06/25(土) 15:13:43.12 ID:m242Qj6T0.net
free の訳し方ありがとうございます!
またensureのその意味は知りませんでした。ありがとうございます!

144 :名無しさん@英語勉強中 :2022/06/25(土) 16:39:44.74 ID:sx7DpPEo0.net
辞書舐めてましたね、今度から引くように心がけます

145 :名無しさん@英語勉強中 :2022/06/25(土) 16:42:13.03 ID:xtPHlGWy0.net
>>142
personっていいのかな?
ここは複数のpersons じゃないのかな?
smoothだと思うよ

146 :名無しさん@英語勉強中 :2022/06/25(土) 17:40:37.12 ID:2K5Ma+Y40.net
Because Mr.Tanaka will be at the trade show, other marketing team menbers will ()be there as well.

(A)sharply
(B)longer
(C)probably
(D)almost

Cが正解なのですが、なぜDでは駄目なのでしょうか?

147 :名無しさん@英語勉強中 :2022/06/25(土) 17:48:00.99 ID:rhh2qGin0.net
>>146
probablyは恐らく〜するでしょう。almastは殆どの〜がと訳せる単語。

148 :名無しさん@英語勉強中 :2022/06/25(土) 17:57:45.16 ID:JJDqEuRl0.net
>>146
副詞はとりあえず直後の単語修飾したいやん
そこに入れて何almostにしたんってまず思う

149 :名無しさん@英語勉強中 :2022/06/25(土) 17:58:54.31 ID:NdarmzF/0.net
145といい147といいレベル低すぎるから回答者に回るなよ

150 :名無しさん@英語勉強中 :2022/06/25(土) 18:04:07.14 ID:2K5Ma+Y40.net
もう少し分かりやすくお願いします。

151 :名無しさん@英語勉強中 :2022/06/25(土) 18:11:42.80 ID:NdarmzF/0.net
>>150
ここにalmostが入ると思った訳を書けよ
どう考えても入らんだろ

152 :名無しさん@英語勉強中 :2022/06/25(土) 18:21:23.54 ID:qf/I1fPo0.net
>>146
まず文法的に入らない。
GRAMMAR: Word order

Almost usually comes after ‘be’ when it is the main verb:
I am almost ready.
https://www.ldoceonline.com/dictionary/almost

153 :😉三年英太郎🌈 :2022/06/25(土) 18:30:40.07 ID:7xo1d/fp0.net
Almost comes after the first auxiliary verb:
She has almost decided.
It could almost be described as luminous.
...🤔

will almost be
will be almost
どっちでもいけない🤔

154 :名無しさん@英語勉強中 :2022/06/25(土) 18:31:22.51 ID:JJDqEuRl0.net
>>152
他方でprobablyは大丈夫って理由も必要にならんか?
副詞キックしてるわけでもないとは思うけどさ

155 :名無しさん@英語勉強中 (ワッチョイW 0f30-1gF+):2022/06/25(土) 18:53:22 ID:JJDqEuRl0.net
>>150
類推でだが「他のほとんど全部のメンバーがいる」
という意味で使いたいんだろうとすると、
全部にalmostつけるべき
almost all the other ~s
と頭から使いたいわな
なおtheはつけてもつけなくてもいいんだが、
敢えてつけたの考えるともう一つ気づく
元の文にはtheがついていないって

つまりチームの残りのメンバー全員いるってわけでもない

違う意味だと
ぼやあっと存在してるって意味なら
almost exist!
almostly exist?
わからんわからん

156 :名無しさん@英語勉強中 (ワッチョイ 3f9d-lObo):2022/06/25(土) 18:54:19 ID:qf/I1fPo0.net
>>154
(A) I will probably be late.
(B) I will be late probably.
(C) I probably will be late.
(D) I will be probably late.

上の4つの語順のうち、正解はAのみ。

157 :名無しさん@英語勉強中 (ワッチョイ 3f29-yeJY):2022/06/25(土) 18:55:35 ID:bwgEa9ww0.net
(C)他のメンツも「おそらく」そこへ…
(D)他のメンツも「もう少しで」そこへ…
前者のほうが意味が通るように思いました

158 :名無しさん@英語勉強中 (ワッチョイW 0f30-1gF+):2022/06/25(土) 18:57:09 ID:JJDqEuRl0.net
>>156
そうなんよなあ
2つ目はコンマ打って許してくれる?

159 :名無しさん@英語勉強中 :2022/06/25(土) 19:32:12.78 ID:qf/I1fPo0.net
文法的には will be almost ~とすれば行けるが、almost 自体が現在形の文章だと「今はまだなってないけど、もうすぐそうなる」という未来の意味になるので
それなら現在形で十分ってことなので、will be almost ~はあまり使われないのかな。
予言みたいな言い回しの時は使うんだろうけど。

160 :三年英太郎 ◆3CZBjOt3.Y (ワッチョイ 3f86-m+5P):2022/06/25(土) 20:18:12 ID:7xo1d/fp0.net
WILL ALMOST BE
they will almost be taking on a new identity.
it will almost be unregulated for a while

WILL BE ALMOST
The eReaders will be almost identical
you will be almost sure to find? fresh?

PROBABLY WILL BE
you probably will be fine.
I probably will be doing a Great American Songbook tour

WILL BE PROBABLY
this will be probably the most
you will be probably better

161 :三年英太郎 :2022/06/25(土) 20:50:19.57 ID:7xo1d/fp0.net
almost も probably も共に中位(mid-position)を好む副詞だとPEU(§198)にある。
中位は、助動詞と動詞の間ね。
これはLDOCEの almost の位置の記述とも一致する。

> "Almost" comes after the first auxiliary verb:
> ・She has almost decided.
> ・It could almost be described as luminous.
https://www.ldoceonline.com/jp/dictionary/almost

(※ 副詞の位置はかなり自由なので、これが絶対のルールではない)

>>146
> other marketing team members will () be there as well.
のようにちゃんと中位を開けているから、統語的に問題あるように見えないが。

162 :Seeker2022 :2022/06/26(日) 00:18:56.79 ID:IvkxcLqp0.net
「ほとんど」の程度に「そこにいる」ってことは、幽霊かなんかかな? 脚だけがない幽霊みたいな ^^

163 :名無しさん@英語勉強中 :2022/06/26(日) 01:35:56.56 ID:Bsenm2bB0.net
But whatever y’all did, put two on it

ラップの歌詞ですがput two on itはどういう意味でしょうか?

164 :😉三年英太郎🌈 :2022/06/26(日) 01:46:17.57 ID:x7RgDy2PM.net
>>156
B以外容認されることはすでにコーパスで確認済みだが、Bも問題なかろう🤔

>(2)文頭にも置くが, 文末には置かないのが普通.
(ウィズダム)

裏を返せば、文末に置いてもよいということである。

165 :名無しさん@英語勉強中 :2022/06/26(日) 09:01:42.49 ID:ErRwd+IU0.net
>>164
裏を返せば、うんぬんはその通り。

だけど入試は学校文法の範囲内での正解を問うものだから、
教科書に書いてないことを書いたら✖になります、
というルールなんじゃないですか?
柔道やってる小学生が絞技したらダメみたいなものかも

166 :名無しさん@英語勉強中 :2022/06/26(日) 10:25:08.64 ID:PFoVsDjc0.net
>>156
(A) 恐らく遅くなると思う。(淡々と自分の考え、予想、可能性を述べている)

(B) 遅れるかも、恐らくだけど。(遅れる可能性はあるが、あくまで自分の推量
 でしかない。大した問題とも思っていない)

(C) 恐らく遅くなりそう。(いつもは時間厳守のつもりのおれだが、今回は
 どうも遅れそう)

(D) 恐らく、遅れるんだろうな。(天候や交通など不可抗力の事情で時間通りに
 着けないのは確かだが、早く着くのより遅れる可能性が高いかな・・・・)

どれも文脈によって、微妙に意味や強調する部分が違うとしても、文法的に
成り立つのならということで訳してみた。・・・・・もっと正確な、あるいは
より適切な日本語訳があったら、・・・誰か・・・・・訂正頼む。

167 :😉三年英太郎🌈 :2022/06/26(日) 11:00:37.69 ID:pmtWOr6+0.net
念の為に英国人に聞いたら
They are all acceptable. It's just a difference of emphasis (Where you place the stress)...🤓
と言っておったぞ

Aが無標な言い方
Cはwillに強勢があるんでないかい

168 :名無しさん@英語勉強中 :2022/06/26(日) 11:07:01.91 ID:OPpIf0sM0.net
英語を勉強し直したいのですが、
解説が充実している初心者向け(小中学生レベル?)の問題集はありますか?
問題を解く→解説を読む→別で買った参考書を読むみたいな流れで勉強したいです
田舎なのでネットで買えると嬉しいです

169 :名無しさん@英語勉強中 :2022/06/26(日) 11:35:21.79 ID:bVNMtdLSM.net
>>168
オススメかどうか分からんが中学レベルの英語も怪しい時代だった時は県立高校の入試問題集をひたすら解いてたら英語に苦にならなくなったな。年度はどれも変わらんからメルカリかブクオフで500円くらいで売ってるから安いのを買うと良いよ。


https://i.imgur.com/HZPFfxZ.jpg

170 :名無しさん@英語勉強中 :2022/06/26(日) 12:59:02.50 ID:wgqsUsg4M.net
these hospitals tell owners what food to give their pets.
これの訳が
「これらの病院はベットにどんなえさを与えるべきかを飼い主に教えてくれる。」
グーグル翻訳だと
「これらの病院は飼い主にペットに与える食べ物を教えています。」
なんですが、what food以降をペットに与えるべき何の食べ物と訳してしまい、わけがわからなくなってしまいました。
what foodがなぜ「食べ物」だけで訳せるのでしょうか?

171 :😉三年英太郎🌈 :2022/06/26(日) 13:12:30.22 ID:pmtWOr6+0.net
>>170
https://nexseed.net/blog/how_to/

172 :名無しさん@英語勉強中 :2022/06/26(日) 14:16:07.31 ID:jjwcqCOf0.net
>>170
「これらの病院は飼い主にペットに与える食べ物を教えています。」
別にこの訳だとしても
病院が何を教えてると解釈した日本語に受け取れてるん?
例えば犬連れてったらこの病院は何を教えるん

何の食べ物という訳だけど
これは何の食べ物?
という言葉が発せられる状況を想像して
どういう答えが出るのか想像できる?

曖昧な日本語をついているとは思うけど解釈はその

173 :名無しさん@英語勉強中 :2022/06/26(日) 14:24:29.94 ID:LeSzy6u8M.net
>>170
単なる訳し方の話でしょ。
↓2文ともほぼ同じ意味じゃん。
「これらの病院はベットにどんなえさを与えるべきかを飼い主に教えてくれる。」
「これらの病院は飼い主にペットに与える食べ物を教えています。」

what food を、to give their pets を修飾しているということを理解して
どう自然に訳すか? という問題なだけ。

174 :名無しさん@英語勉強中 :2022/06/26(日) 14:25:53.02 ID:LeSzy6u8M.net
>>172
日本語で書いてるの?
何言ってるか、全然わからん。

175 :名無しさん@英語勉強中 :2022/06/26(日) 14:25:55.69 ID:etXaVCft0.net
まず、tellはO1、O2を取る動詞なので「誰に」、「何を」「教える」で良いですよね。
また、”to give their pets”は”what food”にかかる形容詞句ですね。
ここで文を見返してみます。
文の骨はThese hospital(S) tell(V) owners(O1) what food(O2).≠ナす。
で、ここでwhatを辞書で調べてみます。
foodの前についているので名詞の前なので形容詞≠ニ当りを付けます。
ジーニアスでは「形容詞 @疑問形容詞 何の、何という、どんな:どれほどの」という
項目が出てきます。この中で適当なものは「どんな」だなと思えてきませんか?

以上の説明はどうでしょうか?

176 :名無しさん@英語勉強中 :2022/06/26(日) 14:32:05.38 ID:jjwcqCOf0.net
>>174
想像しろって言ってるだけだけどね

訳をしてその文章の意味考えないとかありえないでしょ

177 :名無しさん@英語勉強中 :2022/06/26(日) 14:37:27.86 ID:LeSzy6u8M.net
>>176
もう少し国語の勉強をした方が良いと思うよ

178 :名無しさん@英語勉強中 :2022/06/26(日) 14:41:10.80 ID:jjwcqCOf0.net
>>177
で、君は何のスポーツが好き?

これを所有格でしか受け取れないやつはアスペだよ

179 :名無しさん@英語勉強中 :2022/06/26(日) 14:52:13.41 ID:LeSzy6u8M.net
>>178
お前、どっか頭おかしいの?

お前の >>172 の文章力の無さを言ってるだけだよ。
「所有格でしか受け取れない」という文章も不正確。
もっと、日本語を磨く努力をしような。

180 :名無しさん@英語勉強中 :2022/06/26(日) 14:54:18.01 ID:jjwcqCOf0.net
>>179
お前どっか頭おかしいの?
キチガイはこれだから困るわあ

とか枕詞つけてマウント取るしか脳ないやつから言われても褒められてるみたいなもんだがなw

181 :名無しさん@英語勉強中 :2022/06/26(日) 15:01:28.98 ID:LeSzy6u8M.net
>>180
で、君は何のスポーツが好き?

を「所有格で受け取る」の意味言ってみろ、バカ。

182 :名無しさん@英語勉強中 :2022/06/26(日) 15:03:04.60 ID:LeSzy6u8M.net
>>172 の文章を誰か他人に読んでもらえ
尋常じゃないほどのアホだと誰からも思われるぞ

183 :名無しさん@英語勉強中 :2022/06/26(日) 15:05:39.88 ID:LeSzy6u8M.net
(ワッチョイW 0f30-1gF+)のような日本語もうまく操れないバカが回答すると
余計に質問者が混乱するので、できれば回答をお控え下さい。

184 :名無しさん@英語勉強中 :2022/06/26(日) 15:14:20.89 ID:jjwcqCOf0.net
主語がでかいのも感動したわ
誰からもw

what food以降をペットに与えるべき何の食べ物
って訳はわかりにくいだけで訳は成立してるってことだわさ
普通はどんな食べ物と訳すし、混乱した理由は
何の食べ物→whose foodって解釈してわからんかっただけでしょ

でもどんな食べ物って訳もまたあんまり好きじゃないのは俺がアスペだからだろうかw

185 :名無しさん@英語勉強中 :2022/06/26(日) 15:17:40.54 ID:LeSzy6u8M.net
>>184
残念だけど、実際に「誰からも」だ

自分で
>>172 の酷い日本語を読み返して見なさい

186 :名無しさん@英語勉強中 :2022/06/26(日) 15:39:01.95 ID:eN+zgktI0.net
英語の例文入力したら、svomとか品詞を振ってくれるサイトとかアプリってないですよね?
あったらいいなぁ

187 :名無しさん@英語勉強中 :2022/06/26(日) 15:45:33.65 ID:LeSzy6u8M.net
>>170
these hospitals tell owners what food to give their pets.

ご存じかもしれませんが
'what food' の'what'は、疑問関係詞で「どんな」という意味で、'food'にかかっています。
'to give their pets'は、'what food' を修飾しているto不定詞です。

ですから訳としては
「これらの病院はベットに<どんな食べ物>を与えるべきかを飼い主に教えてくれる。」
になります。

「これらの病院は飼い主にペットに与える食べ物を教えています。」は意訳のようなものです。

「どんな食べ物が好きか教えて」も「好きな食べ物教えて」も、同じような意味でしょ。

188 :名無しさん@英語勉強中 :2022/06/26(日) 15:48:01.23 ID:yQIYsmNqa.net
>>186
これなんかどうだ?

enHack
https://enhack.app/app/#!/index/you/home/

webよりスマホアプリの方が高機能らしい

189 :名無しさん@英語勉強中 :2022/06/26(日) 15:51:15.77 ID:LeSzy6u8M.net
>>186
自動翻訳ソフトは
元の英文について、これが主語で、これが目的語で・・・
と解釈しているのだから
技術的に不可能ではないでしょうが、多分存在しないでしょう。

ですから、「英文解釈の技術100」とか薬袋先生の解釈本とかで勉強して、自分でできるようにするのが一番です。
自分でどうしても解決できれば、こういうスレで聞いたりすればいいと思います。

因みにSVOMは「品詞」ではありません。

190 :名無しさん@英語勉強中 :2022/06/26(日) 15:51:35.32 ID:yQIYsmNqa.net
>>188
こんな感じで解析してくれる

https://i.imgur.com/kLas8Z7.jpg

191 :名無しさん@英語勉強中 :2022/06/26(日) 15:53:10.74 ID:LeSzy6u8M.net
>>188
あるのですね。
失礼しました。

192 :名無しさん@英語勉強中 :2022/06/26(日) 16:01:26.67 ID:XM7WDujja.net
>>142
どなたかこれお願いします

193 :名無しさん@英語勉強中 :2022/06/26(日) 16:16:44.73 ID:LeSzy6u8M.net
person→persons
smoothly→smooth

194 :名無しさん@英語勉強中 :2022/06/26(日) 16:17:58.25 ID:Kx9QTrPdr.net
いやいやいや
有料どころかOSSでいくらでもあるから。
NLTKとか好きなの使えばいいよ

195 :名無しさん@英語勉強中 :2022/06/26(日) 16:18:43.52 ID:Kx9QTrPdr.net
>>194>>189へのレス

196 :名無しさん@英語勉強中 :2022/06/26(日) 16:23:03.35 ID:LeSzy6u8M.net
>>193
コーパス検索したら
key persons は、key people より圧倒的に用例が少なかった
間違いではないと思うけど・・・

197 :名無しさん@英語勉強中 :2022/06/26(日) 16:30:47.93 ID:LeSzy6u8M.net
>>194
SVOMって範疇分析ではなく、所謂5文型の構成要素なんだけど
NLTKで対応してんの?

198 :名無しさん@英語勉強中 :2022/06/26(日) 16:36:05.02 ID:Kx9QTrPdr.net
>>197
なぜググれば秒で済むことをいちいち聞くの
https://www.nltk.org/book/ch08.html

199 :名無しさん@英語勉強中 :2022/06/26(日) 16:37:33.06 ID:jjwcqCOf0.net
>>142
theも付いてるしpersonsあるいはpeopleどっちかに直す方が普通だとも思うけど
key personを不可算で使っても問題なさそう
結論わからん

to keep the show smoothly and fun
smoothly and fun
副詞and名詞
smooth and fun
形容詞and名詞
でどっちも嫌なんですが状態
smoothlyでよくね?
結論わからん
だった
サンドバックにしてくれ

200 :名無しさん@英語勉強中 :2022/06/26(日) 17:21:01.94 ID:3vBx3ay40.net
・高度IT人材、富士通は最大年収3500万円へ
・AI人材の獲得に超本気 NECが新人事制度を9人に適用、富士通は最大年収3500万円へ
・【年収3500万円も】富士通、「ジョブ型」人事制度を導入 幹部社員から 高度IT人材
・来年度から副業解禁 人材多様化へ―大同生命次期社長
・第一生命HD、副業解禁 約1万5000人対象
・第一生命HD、副業解禁 1万5000人対象―大手生保初
・IHI、国内8000人の副業解禁 重厚長大企業も転機
・IHI、社外兼業を解禁 社内副業もルール化

201 :名無しさん@英語勉強中 :2022/06/26(日) 17:47:14.31 ID:5D9S1/F6a.net
>>199
fun形容詞で使われてると思うんですけど

202 :名無しさん@英語勉強中 :2022/06/26(日) 17:48:09.67 ID:8bGJ0gqy0.net
R280点くらいなんですけど、part7対策でおすすめの参考書ありますか?

203 :名無しさん@英語勉強中 :2022/06/26(日) 17:53:40.87 ID:BcwAm+B90.net
>>196
むしろkey peopleなんて言い方あるんだ?
なんかヘンテコに見えるw

204 :名無しさん@英語勉強中 :2022/06/26(日) 17:57:57.18 ID:bVNMtdLSM.net
>>202
多読やるしか無いと思う

205 :名無しさん@英語勉強中 :2022/06/26(日) 20:30:43.46 ID:LeSzy6u8M.net
>>198
SVOMという構成要素を表示する方法なんて載ってないけど

206 :名無しさん@英語勉強中 :2022/06/26(日) 22:15:28.68 ID:YJLRfHR7r.net
>>205
>>189のレス見る限り技術系の知識ゼロだろうに
素直にわかりませんって言えばいいのに

207 :名無しさん@英語勉強中 (ワッチョイ 8f70-tCSL):2022/06/27(月) 00:48:08 ID:0+R8sXLK0.net
>>186
亀レスですまんが
https://english.edward.io/

文型までは判断してくれないけど、品詞分析なら十分に使える

208 :名無しさん@英語勉強中 :2022/06/27(月) 01:17:35.92 ID:cD2ATnbWa.net
>>206
お前さんも本職の技術系のレスには見えんがな
いいとこ浅薄な知識でマウント取りたいだけの坊や

209 :名無しさん@英語勉強中 :2022/06/27(月) 01:40:24.20 ID:3SrUndRrM.net
>>206
方法が載ってない

わからん

何が違うのか?ガイジ?

210 :三年英太郎 :2022/06/27(月) 01:50:33.05 ID:dh3LASZs0.net
>>198
これって自動的に分析してくれるんか?
なんかプログラム書いたら、樹形図になって現れるだけのように見えるんやけど

211 :名無しさん@英語勉強中 :2022/06/27(月) 02:32:32.86 ID:xNtz9Hp70.net
リンク先は長いから読んでないけど手元でいじってみたら形態素解析だけじゃなくてちゃんと構文解析もできるのな

212 :名無しさん@英語勉強中 :2022/06/27(月) 02:42:36.38 ID:3SrUndRrM.net
>>211
でも、SVCOMとか出力させるには
自分でプログラム書かないとダメじゃん?

213 :名無しさん@英語勉強中 (ワッチョイW 4fbc-oZtE):2022/06/27(月) 08:18:36 ID:98oGjvd90.net
downtown、uptownのアクセントはそれぞれどこでしょうか?
Weblioで調べてもそれぞれ dówn・tówn 、úp・tówn とあり、前と後ろどこに来るかが分かりません。。

https://ejje.weblio.jp/content/downtown
https://ejje.weblio.jp/content/uptown

214 :名無しさん@英語勉強中 :2022/06/27(月) 10:51:38.69 ID:1wsgMsI20.net
ほんと書き込み屋が自演してるだけの板

215 :😉三年英太郎🌈 :2022/06/27(月) 15:21:27.41 ID:dh3LASZs0.net
ピュトホーンの勉強でもしよっかな

216 :名無しさん@英語勉強中 :2022/06/27(月) 15:37:35.13 ID:J0Ye8gbqM.net
What day suits you?
でどうして、あなたの都合の良い曜日はいつですか?になるのでしょうか?
文法としてよくわかりません。

217 :😉三年英太郎🌈 :2022/06/27(月) 16:06:33.46 ID:dh3LASZs0.net
What day (s)
suits (v)
you? (O)

218 :名無しさん@英語勉強中 :2022/06/27(月) 16:17:53.81 ID:J0Ye8gbqM.net
>>217
それだと何曜日があなたに合います
なりませんか?

219 :名無しさん@英語勉強中 :2022/06/27(月) 16:18:26.18 ID:J0Ye8gbqM.net
>>218
何曜日があなたに合います
になりませんか?

220 :名無しさん@英語勉強中 :2022/06/27(月) 16:37:22.14 ID:5t84UdQ10.net
やっぱり日本語の問題やん

何曜日があなたに合います
ってどういう意味で受け取っているのか説明できる?
できないならこの回答は多分無理

221 :名無しさん@英語勉強中 :2022/06/27(月) 16:39:46.19 ID:/Z0Czhfca.net
主語を問う疑問文だと普通の形では?
whoでなら良くある形で、216は無生物主語だからwhatになってるだけ

222 :名無しさん@英語勉強中 :2022/06/27(月) 16:48:44.29 ID:WfRueJ3A0.net
大会優勝おめでとうございます

って英語で何て言うんでしょうか?

223 :名無しさん@英語勉強中 :2022/06/27(月) 16:54:53.49 ID:5t84UdQ10.net
コングラッチュレイションズ ユーウィン、グレート、ワンダホー
とか言うときゃええやんとわれ思う

224 :😉三年英太郎🌈 :2022/06/27(月) 16:54:55.66 ID:dh3LASZs0.net
congrats on winning the championship🤗

225 :名無しさん@英語勉強中 :2022/06/27(月) 16:57:17.12 ID:WfRueJ3A0.net
回答ありがとうございます
大会名も入れたい場合はどうしたらいいんですかね?
例えば
甲子園優勝おめでとうございます、なら

congrats on winning the koushien
になるんでしょうか?

226 :😉三年英太郎🌈 :2022/06/27(月) 17:10:31.26 ID:dh3LASZs0.net
the Koshien tourney

227 :名無しさん@英語勉強中 :2022/06/27(月) 17:12:42.40 ID:WfRueJ3A0.net
congrats on winning the koushien tourney
でいいんでしょうか?

228 :😉三年英太郎🌈 :2022/06/27(月) 17:42:01.44 ID:dh3LASZs0.net
いいと思います🙂

229 :名無しさん@英語勉強中 :2022/06/27(月) 18:04:48.10 ID:B/tPp+V/0.net
どなたか >>213 教えていただけると助かります (>_<)

230 :名無しさん@英語勉強中 :2022/06/27(月) 18:09:20.34 ID:tDvAdRkCr.net
>>209
簡単なプログラミングも理解できない上に
「ガイジ」連呼でまともな日本語の対話もできず
おまけに簡単な英語のマニュアルも読めないとか
救いようがなくて笑えますわ

231 :😉三年英太郎🌈 :2022/06/27(月) 18:23:28.50 ID:dh3LASZs0.net
>>229
ここは誰かに英和を引いてもらうスレじゃないですよ🤔
>>1に英和のリンクあるでしょう?

232 :Seeker2022 :2022/06/27(月) 18:57:05.29 ID:gT5kirf60.net
>>231
>>229はちゃんと辞書ひいてるんだが^^

>>229
アクセント記号が両方の音節にあるということは、両方とも同じだけ強く読むということで、そもそもどっちかを強く読むという単語じゃないのです。

233 :😉三年英太郎🌈 :2022/06/27(月) 19:44:46.51 ID:dh3LASZs0.net
>>232
この方は、他の辞書を見てもこんなトンチンカンなこと言ってるの?🤔

234 :名無しさん@英語勉強中 :2022/06/27(月) 19:45:52.45 ID:83WKTa150.net
ネットのコメント欄にカナダ人が

Omg!
You two are so great together! Im shipping you two 🥰

って書いてて、
「二人とも素敵です
 仲良しぶりが伝わってきます」って
DeepLではなるんですが

shipで
なんでそうなるのでしょうか?

235 :名無しさん@英語勉強中 :2022/06/27(月) 19:49:21.98 ID:tDvAdRkCr.net
>>234
輸送するのshipじゃなくてrelationshipから来てるみたいね

?[transitive] ship A and/with B (informal) to think that two people should be in a romantic relationship
Some people are already shipping Alex and Meredith.
People ship Emma with Jack because he’s so hot and funny!

236 :名無しさん@英語勉強中 :2022/06/27(月) 22:46:19.00 ID:7qpAwftqM.net
>>221
whoで置き換えて考えてみたら何となくですがイメージがわかりました。
何の曜日が○○ですか?ってことかなと思いました。
「何曜日」というのが名詞だと思ってしまいました。

237 :Seeker2022 :2022/06/27(月) 22:48:58.43 ID:gT5kirf60.net
>>233
まーた、論拠も示せず罵倒かな ^^

トンチンカンじゃないのにトンチンカンだと思う三年ちゃんがトンチンカンだといい加減に気づこうね

238 :Seeker2022 :2022/06/27(月) 23:03:03.48 ID:gT5kirf60.net
>>233
辞書を見てもわからなかった質問者に、ただ「辞書を見なさい」ってアドバイスはひどすぎ(笑)

三年ちゃんは、質問者のレベルに合わせて思いやりをもって回答するともっと良くなると思うよ。

239 :Seeker2022 :2022/06/27(月) 23:32:01.89 ID:gT5kirf60.net
>>234
shipには「2人(目的語)が互いによきロマンス相手である(orになりそうだ)と思う」みたいな意味もあります。
235の言うとおり、「relationship」が語源です。

上記の意味が意訳され、「仲良しぶりが伝わってきます」となります。

ship
transitive verb

informal
: to wishfully regard (specific people or fictional characters) as being or having the potential to become romantically involved with one another

出典
https://www.merriam-webster.com/dictionary/ship

240 :名無しさん@英語勉強中 :2022/06/28(火) 00:22:15.63 ID:6cURD9nI0.net
どうも!

「相思相愛」みたいな感じだと思うんですが

ならどうして
最初に

I'mがあるのでしょう?

241 :😉三年英太郎🌈 :2022/06/28(火) 00:37:00.01 ID:fjdYlEk30.net
語誌を見ると、もろもとはフィクション内の登場人物間での妄想が始まりらしい

日本でも腐がBL妄想してるじゃん

あーしはカツオ×中島でカップリング(妄想)するよという時に、I ship Katsuo and Nakajima😍😍😍 と言ったようだ

242 :😉三年英太郎🌈 :2022/06/28(火) 00:38:58.30 ID:fjdYlEk30.net
つーか今でもそーゆーノリっぽいが

243 :名無しさん@英語勉強中 :2022/06/28(火) 00:55:41.24 ID:XznSXsRYa.net
>>236
老婆心ながら疑問文で悩んだときは平叙文から考えてみるのも手かと

例えば、日曜日があなたに適した日です、だと
Sunday suits you.

今回の場合、Sundayが何か聞きたいわけだから、Sundayを疑問詞のWhatに置き換えてみる。
What suits you?

Whatってなんやねん?人かよ?物かよ?となるので曜日だって事を加えてみると
What day suits you?

目的語の場合はちょっと手順が増えるけど、例えば
You look for it.

でitを聞きたければ
You look for what?

whatは基本先頭なんで
What you look for?

こういう場合はdoがいるので
What do you look for?

244 :名無しさん@英語勉強中 :2022/06/28(火) 03:19:34.42 ID:kkcDTUKf0.net
“I think she had went to go jump and she kind of second-guessed herself and went to go to grab the rail and then I think she just landed on a beam. Just wasn’t her time to go,"
https://www.wbrz.com/news/it-wasn-t-her-time-to-go-woman-rescued-from-mississippi-river-bridge-following-suicide-attempt/

この "went to do" の使い方って辞書に載ってるのかな?今日、初めて知ってビックリした。
こういう表現に出会うと文法とか勉強しても意味ないよね。

245 :名無しさん@英語勉強中 :2022/06/28(火) 08:19:24.71 ID:sAQEm+V5M.net
>>232
ご回答ありがとうございます!よく分かりました。
フォローもしてくださりありがとうございます。

246 :名無しさん@英語勉強中 :2022/06/28(火) 08:38:10.07 ID:6cURD9nI0.net
>>241
な〜るほど!納得です!
インスタで
たまたま筋トレ男子同士が肩組んでる画像に
ゲイらしき人がつけたコメントなんです!
ありがとうございました。これは
なかなかわかんない表現でした。

もし可能なら、その語誌
教えてくれませんか?
EnglishJournalを15冊くらいしか持ってないんです
あれちょっと左翼臭がしてw

247 :名無しさん@英語勉強中 :2022/06/28(火) 08:45:03.54 ID:mw1TtSUVa.net
>>244
ネイティブにも文法的に誤った英語を使うのがいるというだけ。

248 :名無しさん@英語勉強中 :2022/06/28(火) 08:53:57.05 ID:VAvx8Riwa.net
オタク用語とかネットスラングはurbandictionaryで十分じゃないかね。

ファンの二次創作とかで勝手に付き合ってることにされてるキャラ同士が原作でもその後の展開で実際そうなって公式カップルになるとcanonということになる。

https://www.youtube.com/watch?v=LCDgJiPBxfI

249 :名無しさん@英語勉強中 :2022/06/28(火) 09:01:31.31 ID:7dD0myyc0.net
日本語検定どれだけの日本人が間違わずに解けるのか、と思ったな

https://www.nihongokentei.jp/check/

250 :名無しさん@英語勉強中 :2022/06/28(火) 09:17:04.89 ID:+04odmWDa.net
日本語文法に限れば、日本人が学校で習う文法と日本語学習者が習う文法は基本構造が違うってのもあるだろうけどな

251 :名無しさん@英語勉強中 :2022/06/28(火) 09:49:04.51 ID:kkcDTUKf0.net
>>247
文法的に間違ってはいないそうだ。
https://www.youtube.com/watch?v=ziHvcygfwdc

I went to go to the supermarket. ≒ I was going to go to the supermarket, but something happened and I couldn't. と同じ意味になるとか

252 :😉三年英太郎🌈 :2022/06/28(火) 09:59:24.79 ID:fjdYlEk30.net
>>246
https://en.m.wiktionary.org/wiki/ship

253 :名無しさん@英語勉強中 (アウアウウーT Sad3-MgET):2022/06/28(火) 10:18:22 ID:A2eGhQ/ka.net
>>251
go to 動詞は普通に辞書にも参考書にも教科書にも載ってる。
had wentがhad goneにならないと誤り。

254 :名無しさん@英語勉強中 (ワッチョイ 3f9d-lObo):2022/06/28(火) 10:26:21 ID:kkcDTUKf0.net
>>253
went to = was going to = was trying to なんて辞書には載ってないだろ。
こんな使い方はネイティブしか知らないよ

255 :名無しさん@英語勉強中 (アウアウウーT Sad3-MgET):2022/06/28(火) 10:27:53 ID:A2eGhQ/ka.net
>>254
17c〔+to do〕〈…しようと〉する.
・When I went to get up from the chair, I felt a pain. いすから立ち上がろうとした時痛みを覚えた.

256 :三年英太郎 :2022/06/28(火) 11:43:26.63 ID:fjdYlEk30.net
文法的誤りやなくて、非標準やな
コーパスでもそれなりの数が確認できる

gone の行っちゃった感を出さないように、自然と went を代用してるのかもしれない
と、用例をみて思った

257 :三年英太郎 :2022/06/28(火) 11:46:41.47 ID:fjdYlEk30.net
2nd WOMAN: I don't think he said it like that. I don't think I would have went for that,' We're getting married.'

Source SPOK: ABC_DayOne
Date 1994 (19940207)
Title ABC_DayOne / 19940207

258 :名無しさん@英語勉強中 :2022/06/28(火) 12:44:25.28 ID:VbAMUPjy0.net
had went
https://www.englishforums.com/English/HadWent/bkwxn/post.htm

259 :😉三年英太郎🌈 :2022/06/28(火) 13:03:44.65 ID:fjdYlEk30.net
こっち出さんかーい
https://bridgingtheunbridgeable.com/english-today/have-went/#:~:text=Went%20implies%20going%20to%20a,of%20the%20eighteenth%20century%20on.

260 :名無しさん@英語勉強中 :2022/06/28(火) 14:16:50.02 ID:2Ev4QKdl0.net
諺の
Do as the Romans do, when in Rome.
って、本当に、日本の「郷に入れば郷に従え」に相当するモノなの?

逆じゃないのかな?
ローマ帝国の治下に入った以上、従前の田舎の掟は通用しない、ローマ帝国共通の道徳や作法に従え
ってところではないんかい?

261 :三年英太郎 :2022/06/28(火) 14:37:53.34 ID:fjdYlEk30.net
そのローマはローマ市やそうや
https://en.wikipedia.org/wiki/When_in_Rome,_do_as_the_Romans_do

262 :名無しさん@英語勉強中 :2022/06/28(火) 14:45:50.31 ID:Zcp+cw6b0.net
>>251
面白い
というかbe trying to以上に仮定法みたいになるんやな
>>255
は立てなかったニュアンスが含まれるんか

>>260
ことわざは特殊な事例がジェネラライズしたものやから
仮にそうでもほぼ郷に~と同じ意味にまで変わってると思うのと、
rome帝国のローマとは限ら
従属のニュアンスのdoとも思
wikipediaでおわりだったのか

263 :名無しさん@英語勉強中 :2022/06/28(火) 22:54:47.99 ID:DXFe1ck60.net
>>260
それって何が逆なの?

264 :名無しさん@英語勉強中 :2022/06/29(水) 03:42:31.32 ID:+17k81RF0.net
what is to replace it, we shall see ,won't we?
「代わりのもが何なのか
理解できるようになるんじゃないのかな?」

という意味ですか?

265 :名無しさん@英語勉強中 :2022/06/29(水) 04:17:27.34 ID:zW5u6aY3r.net
>>264
違います。

266 :名無しさん@英語勉強中 :2022/06/29(水) 08:32:29.54 ID:LTu6D8Ub0.net
>>263
田舎に来たら田舎の流儀に従え

田舎の流儀は通用しないグローバルスタンダードに従え

逆じゃん

267 :名無しさん@英語勉強中 :2022/06/29(水) 08:40:01.08 ID:GRQHmMdua.net
When in Rome, do as the Romans do[1] (Medieval Latin sī fuerīs Rōmae, Rōmānō vīvitō mōre; sī fuerīs alibī, vīvitō sīcut ibī; often shortened to when in Rome...),[2] or a later version when in Rome, do as the Pope does,[3] is a proverb attributed to Saint Ambrose. The proverb means to follow the traditions or customs of a place being visited.

Saint Monica and her son, Saint Augustine, discovered that Saturday was observed as a fast day in Rome, where they planned to visit. However, it was not a fast day where they lived in Milan. They consulted Saint Ambrose who said "When I am here (in Milan) I do not fast on Saturday, when in Rome I do fast on Saturday." That reply is said to have brought about the saying "When in Rome, do as the Romans do."[4][5]

268 :名無しさん@英語勉強中 :2022/06/29(水) 09:03:11.53 ID:sP2D+mXj0.net
I couldn’t actually believe how good this new release was, how she had managed to change herself
so utterly effectively again just at the moment that her image had begun to get stale,
that her profound, chameleon intuition had allowed her to lay low for a while, dye her hair black,
get divorced from Sean Penn, and re-emerge, triumphantly, with a fantastic, epic record of such monster proportion.
I had always really liked Madonna, but now I loved her.

最後の一文だけ教えて欲しいのですが文脈のために
その前の文章も貼りました

(歌手の)マドンナのことがずっと好きだったが
今は大好きだ・
みたいな感じだとしたら
なぜ最後の loveが lovedなんでしょうか?

269 :名無しさん@英語勉強中 :2022/06/29(水) 10:25:52.95 ID:YckBwDAT0.net
>>268
この now はマドンナが金髪から黒髪にイメチェンした1989年当時のことだろう。
https://goldfm.lk/life/other/5231/madonna-1989

それまではマドンナのことを超好きだったけれど、イメチェンした今となっては好きが愛になった。

270 :名無しさん@英語勉強中 :2022/06/29(水) 10:45:28.03 ID:+4qd7AR40.net
>>266
なるほど、元はそういう意味なのか?
郷に入れば〜で、田舎って意味を意識したことなかったわ

271 :名無しさん@英語勉強中 :2022/06/29(水) 12:59:20.06 ID:FIGMQ0bxaNIKU.net
>>270
郷は故郷の意味もあるけど、郷に入れば~の方は土地の意味だな

272 :名無しさん@英語勉強中 :2022/06/29(水) 17:38:20.84 ID:gsM4j6zV0NIKU.net
多読学習してる者なんだけど、audibleで多聴も始めたが全然聞き取れる気しない…
一冊通読する→本見ながら聞く→本無しで聞く。とやっても名詞や動詞は聞き取れてもリエゾンが全く聞き取れないです
同じ本を何度も読むのも辛いし、諦めて色んな本を多読多聴するか、一冊だけ聞き取れるようになるまで多聴するか、graded readersのような簡単な本で多聴すべきか、どうすればいいと思いますか?

273 :名無しさん@英語勉強中 :2022/06/29(水) 18:28:04.34 ID:jdeIdFGm0NIKU.net
>>272
audiobook や audible などの youtube は一本が何十分、何時間と長いので、
慣れないと根気が続かない。特に、途中知らない単語や表現にぶつかって
頭が真っ白になるとやる気が失せると思う。

それより、興味のあるテーマの1本10分程度の、いろいろな youtube で
発音やスピードもみな違うものがいい、勿論、内容も違う。
興味のあるテーマに関係する英単語で「検索」すればいくらでも見つかる。

そしてその youtube の中で出てきた新しい単語と表現を一つずつ身に着け、
着実に聞き取れるようにすると上達が早い。効果的な学習法を自分で工夫する。
(自動再生の字幕でも、完璧ではないが、単語や表現は把握できる。
似たようなテーマだと類似表現が頻発するので応用力も付く)

まず一カ月、…6か月、そして一年続ければ必ず成果が出てくるはず。

274 :名無しさん@英語勉強中 :2022/06/29(水) 18:31:00.42 ID:jdeIdFGm0NIKU.net
追加、
多読は多読で、語彙力や表現力を身に着けるために並行して続けるとなおいい。

275 :名無しさん@英語勉強中 :2022/06/29(水) 20:32:26.68 ID:4M3v2Cc2rNIKU.net
>>272
リエゾンじゃなくてアンシェヌマンのことじゃないの?

276 :名無しさん@英語勉強中 :2022/06/29(水) 21:36:29.23 ID:hhASg3H9dNIKU.net
>>275
普通はリエゾンだろ
アンシェヌマンなんか英語で使うの聞いたことない

277 :名無しさん@英語勉強中 :2022/06/29(水) 21:38:29.30 ID:4M3v2Cc2rNIKU.net
>>276
英語におけるリエゾン何て
イギリス英語でplayer isと続くときの母音化してたrぐらいでしか言わないけどね、普通は

278 :名無しさん@英語勉強中 :2022/06/29(水) 21:44:44.64 ID:4M3v2Cc2rNIKU.net
アンシェヌマンは確かに言わない。
ただリエゾンは完全に誤用なんでせめてリンキングと言いましょう。

279 :名無しさん@英語勉強中 :2022/06/30(木) 05:13:51.78 ID:Z5P9VbSWM.net
It costs a lot to attend university.
大学に通うにはかなりのお金がかかります

It is costing me a lot to attend university.
大学に通うのにかなりのお金がかかっています。

現在形のcostのあとにmeがありますが、現在形の場合me(目的語)を入れる必要があるのでしょうか?
違いがいまいちわかりません。

280 :279 :2022/06/30(木) 05:30:42.77 ID:Z5P9VbSWM.net
>>279
現在形ではなく現在進行形でした。

この現在形と現在進行形の違いですが
現在形は大学に通うことはお金がかかるという普遍的なことを説明していて、現在進行形は"今まさに私が一時的に大学の学費を支払っていてお金が掛かっているところ"という意味で進行形になって目的語のmeを伴っているのでしょうか?

281 :名無しさん@英語勉強中 :2022/06/30(木) 06:10:09.27 ID:RROLH53v0.net
辞書を

282 :名無しさん@英語勉強中 :2022/06/30(木) 06:13:11.32 ID:RROLH53v0.net
>>272
やらないといけないのは多聴じゃなくて、
映画英語のリスニングシリーズのようなlinking assimilation elision reductionに焦点を当てた発音の本を1冊やることだよ

283 :名無しさん@英語勉強中 :2022/06/30(木) 20:14:15.47 ID:yMvmJukQ0.net
なんでit is dangerous that…
とかit is difficult that…
とか言わないんですか?
it is difficult toとかit is dangerous to..
というのに。

284 :名無しさん@英語勉強中 :2022/06/30(木) 20:17:11.88 ID:yMvmJukQ0.net
it has taken centuries to
のit ってto 以下の形式主語
と言えるんですか?
それともそのit は事情を意味するit なんですか?

285 :名無しさん@英語勉強中 :2022/06/30(木) 20:21:23.93 ID:LMecF+Bu0.net
耐衝撃
Impact resistance

Impact resistant
どちらが正しいですか

286 :名無しさん@英語勉強中 :2022/06/30(木) 20:55:31.46 ID:yMvmJukQ0.net
名詞と形容詞の違いじゃない?

287 :名無しさん@英語勉強中 :2022/06/30(木) 20:56:59.03 ID:yMvmJukQ0.net
it takesとかit has taken ってit の項目に載ってないんだよね。
文例ならいくらでもあるのに。不思議だな

288 :名無しさん@英語勉強中 :2022/06/30(木) 21:51:06.67 ID:yMvmJukQ0.net
英文法解説、ロイヤル青、黄、オーレックス、ウィズダム、cobuild、collins dictionary、
の中にはit takes が入っておらず、英文法総解の中にただ慣用句の分類の中に入っているだけ
ということだった。

289 :😉三年英太郎🌈 :2022/06/30(木) 22:17:30.48 ID:keayMWM00.net
Take 引けば

290 :名無しさん@英語勉強中 :2022/06/30(木) 22:45:09.22 ID:RROLH53v0.net
w

291 :名無しさん@英語勉強中 :2022/06/30(木) 22:59:54.09 ID:stMpLP5n0.net
>>272
ワイは英語圏のユーチューブ見まくっただけで大体分かるようになってきたと思う
リエゾンで聞き取れない点を分からないまま字幕をすぐ見るのではなく何と言ってるか絶えず予測するのが大事
たとえその予測が外れても効果はあるはず

292 :名無しさん@英語勉強中 :2022/07/01(金) 09:11:06.04 ID:IzXNPiwF0.net
13星座占いそのものを指す呼び方どういうのが一般的?
12星座占いは「12 horoscopes」もしくは「zodiac signs」らしいが

293 :Seeker2022 :2022/07/01(金) 10:54:18.99 ID:+2xRIEcT0.net
>>292
13-sign sidereal astrology または 13-sign astrology

13星座占いは、1995年に占星術師のウォルター・バーグが独自に勝手に編み出した手法であり、正統占星術師ならこの理論がおかしいことがわかる。

論拠
西洋占星術で言う牡羊座とかおうし座とかは、実は実際の星座とは関係がない。
占星術で重要なのは天空の方角でしかない。その方角を12で割って、たまたまその方向にあった星座の名前をつけただけである。

ゆえに、「黄道にへびつかいざ座が存在するのになんでそれを使わないの?」というウォルター・バーグは、占星術の基本原理が分かってない。
まともな占星術師なら13星座なんて恥ずかしくて言えない。

294 :Seeker2022 :2022/07/01(金) 11:11:18.84 ID:+2xRIEcT0.net
追記
歳差現象により、西洋占星術が登場した当時と現在の星座の位置はかなりずれている。星座も移動するので。
これが西洋占星術用天文暦と現在の天文学の天文暦が一致しない理由である。

しかし、現在の西洋占星術は、現在の天文学の天文暦を使わず、太古の星座の位置に基づいた西洋占星術用天暦を使い続けてまったく問題がない。
なぜなら、重要なのはその星座が太古に位置してた方角だからである。星座自体が人の運命に影響を与えるわけではない。これが正しい西洋占星術理論である。

295 :名無しさん@英語勉強中 :2022/07/01(金) 11:11:23.36 ID:tyHK4+CR0.net
インスタなんかで
モデルの体のポーズと顔の表情両方が素晴らしいっていう
表現の時

This pose and expression is Killer

は間違いで やっぱpose and expressionだから

複数形で areですか?

他にいい表現あるでしょうか?
ちょいセクシー系ですw

296 :名無しさん@英語勉強中 :2022/07/01(金) 11:54:52.40 ID:IzXNPiwF0.net
>>293
ありがとう

297 :😉三年英太郎🌈 :2022/07/01(金) 12:05:54.29 ID:uHxQqTLg0.net
seeker先生、どこからコピペしてきたん
相変わらず適当すぐる🤦🏻‍♂

>天空の方角

天球上の位置であって、「方角」でない
北極星周辺以外、星の方角は変わる

>星座自体が人の運命に影響を与えるわけではない。

seeker先生の占星術はそうなだけ
それぞれの宮(宝瓶宮、磨羯宮等)には性格があるとゆーのが、中世以来、王道な西洋占星術。近代になると、それぞれの宮の守護神やら、対応するタロットやらヘブライ文字やら、色々絡ませてくる人たちが出てくる

298 :😉三年英太郎🌈 :2022/07/01(金) 12:11:07.41 ID:uHxQqTLg0.net
ダンテの神曲は占星術がちょっとしたスパイスになってるのだが、原訳のたとえば天国篇22歌の注釈には次のように書かれている。

「双子座の光は文筆や学問、学芸の力を地上に伝え、知性を活性化するとされた。」(p.339)

299 :Seeker2022 :2022/07/01(金) 12:35:35.33 ID:+2xRIEcT0.net
>>297
そんなに私に反論されたいの? マゾですか^^

「天空の方角」というのはわかりやすく説明するための表現。
そもそも、地球全体(地球の中心点)から見てと考えれば、「天空の方角」は間違いではないよ。

>それぞれの宮(宝瓶宮、磨羯宮等)には性格があるとゆーのが、中世以来、王道な西洋占星術。

あのさあ、宝瓶宮の宮ってさ、「星座自体」を指さないんだよ。「宮」というのは、かつて星座があった方向にあるゾーンのこと。
専門用語でいえば、 黄道十二宮の名称。星座の名称じゃないのさ。
占星術で言う「水瓶座」は「宝瓶宮」を簡略にしたものなんだ。だから、各宮には性格があるというのは、「ゾ-ン」に性格があるのであって、星座自体は関係ないのだよ。

ここ↓を読んで勉強したまえ
https://ja.wiktionary.org/wiki/%E7%99%BD%E7%BE%8A%E5%AE%AE

300 :Seeker2022 :2022/07/01(金) 12:45:16.86 ID:+2xRIEcT0.net
>>298
「ふたご座」という訳語には2つの意味がある。一つは天文学で言う「ふたご座」、もう一つは占星術でいう「ふたご座」。
占星術での「ふたご座」は、「双児宮」or「双子宮」のことを指し、これは黄道12宮の名前であって、星座自体を指さないのである。

3年ちゃんはここが理解できていない。下記リンクを読んでじっくり学習しようね

https://en.wikipedia.org/wiki/Gemini_(constellation)
https://en.wikipedia.org/wiki/Gemini_(astrology)

301 :Seeker2022 :2022/07/01(金) 12:47:42.54 ID:+2xRIEcT0.net
>>300
のリンクが不完全でしたね。( )内もリンクに含めてね

302 :Seeker2022 :2022/07/01(金) 12:50:37.98 ID:+2xRIEcT0.net
簡単にまとめておきましょう。

Gemini(ふたご座)には、二つの定義があって、一つは占星術用語。もうひとつは天文学用語。
前者は、黄道12宮のゾーン(方角)の名前であり、星座自体の名前ではないのです。

>>298はまずこの違いを理解しましょう。

303 :😉三年英太郎🌈 :2022/07/01(金) 12:50:55.63 ID:uHxQqTLg0.net
>各宮には性格があるというのは、「ゾ-ン」に性格があるのであって、星座自体は関係ないのだよ。

😂🤣🤣無知すぎワロタ😂🤣😂

例えば天秤宮なら、天秤→公平性・正義のような連想が当然なされたのだよ

The main connotations of planets under this sign are sociable, rational, and detached when dealing with situations.[10] Libra is symbolized by the scales and is associated with the Roman deity Iustitia. According to the writer Manilius, Roman judges are born under the sign of Libra.[11][page needed]
https://en.m.wikipedia.org/wiki/Libra_(astrology)

古代~中世は象徴とアレゴリーの時代。バカバカしいほど、記号は意味を持つ

304 :😉三年英太郎🌈 :2022/07/01(金) 12:55:03.89 ID:uHxQqTLg0.net
>>300
この解説では双子座=双児宮で良い
ダンテの歩み↓
https://i.imgur.com/915Wn2f.jpeg

305 :😉三年英太郎🌈 :2022/07/01(金) 12:58:45.05 ID:uHxQqTLg0.net
>地球全体(地球の中心点)から見てと考えれば、「天空の方角」は間違いではないよ。

黄道理解してまっかー?🤣
朝と夜で方角変わりまっせ🤣🤣🤣

306 :😉三年英太郎🌈 :2022/07/01(金) 13:03:08.30 ID:uHxQqTLg0.net
seeker先生はエンターティナーだなぁ
これからも楽しませていただきますね😊

307 :Seeker2022 :2022/07/01(金) 13:04:39.09 ID:+2xRIEcT0.net
>>303
3年ちゃん大丈夫?論理が破綻してるけど。

>例えば天秤宮なら、天秤→公平性・正義のような連想が当然なされたのだよ

これは、私の「星座自体は関係ない」を否定してないけど・・・ 
「天秤宮なら、天秤→公平性・正義のような連想が当然なされた」←これを私は一度も否定していないけど・・・

もう一度教えてあげるね。「天秤宮」は「天文学で言う星座」を指してはいないの。「宮」と「星座」は別物なのだよ。
3年ちゃんは「天文学でいう天秤座が公平性・正義という性格を与える」と思ったから私に反論してるんだよね?
そうなら、それは間違い。

308 :名無しさん@英語勉強中 :2022/07/01(金) 13:09:19.54 ID:hspoc9+Ka.net
最近ギーク爆笑劇場のリリースペースが上がっとるな

309 :Seeker2022 :2022/07/01(金) 13:11:57.89 ID:+2xRIEcT0.net
>>305
地球の中心点からの方角は朝でも夜でも変わらないよ。地球をひとつの点とみたときの方向は朝でも晩でも変わらないよ。

>>304
これもおかしな反論。
私は、「ふたご座」には、「天文学でいうふたご座」と「黄道12宮の双児宮」の2つの意味があると言っているだよ。
>>304はぜんぜん私の意見に反論していないぞ。双子座=双児宮の場合があることは誰も否定していない

どうした?大丈夫か?^^

310 :Seeker2022 :2022/07/01(金) 13:14:03.92 ID:+2xRIEcT0.net
>>308
私はギーク氏じゃないけどね。ギーク氏とはある掲示板で知り合って、いろいろやりとりしていた関係。
それにしても、この議論、私が圧倒的に正しいのが理解できないとは・・・

311 :Seeker2022 :2022/07/01(金) 13:19:25.13 ID:+2xRIEcT0.net
3年ちゃんは、「~座」という表現に「天文学で言う星座」と「黄道12宮の名称」の2つがあることを理解できてないから、トンチンカンな反論をするのだよ。

まずは、そこの違いをじっくり勉強してから反論してほしい。

312 :名無しさん@英語勉強中 (アウアウウー Sad3-7WNR):2022/07/01(金) 13:48:00 ID:GlHFus4sa.net
オカルト板でやってくれ

313 :Seeker2022 ◆iymFBv30wPKz (ワッチョイ 8f22-tCSL):2022/07/01(金) 13:52:15 ID:+2xRIEcT0.net
わかりました。以後、この話題を禁止にしましょう。反した者は、悪人ということで ^^

314 :名無しさん@英語勉強中 :2022/07/02(土) 23:20:56.90 ID:p7VEcCX00.net
>>289
it takes 時間 to-の例文と訳は出てくるけど、
itがto不定詞を示す形式主語なのか、それとも
事情?を表すitなのかは、takeのところを
見るだけではわからないから上の書き込みを
したわけです。

315 :名無しさん@英語勉強中 :2022/07/03(日) 06:17:23.59 ID:fBHYh/S/0.net
>>314
All IN ONEという本では392頁に「時を表すitの構文」の補足として、「形式主語のitを
用いて(it は to Vb を指す)」、「It takes … for N to Vb. / It takes N … to Vb.
(NがVするには…がかかる)」と載っています。

316 :名無しさん@英語勉強中 :2022/07/03(日) 16:41:50.41 ID:BZNHQiJ60.net
オールインワンの考え方例を書いてくれてありがとう

317 :名無しさん@英語勉強中 :2022/07/04(月) 05:25:37.97 ID:es6H4YfTM.net
あるフレーズ集の本でI'll be right here.の訳がここでお待ちします(ビジネス)と書いてあったのですが、どうしてそうなるのでしょうか?
このフレーズを調べたら色々な意味があって混乱してしまいます。
普通に訳すと私は「すぐにここに来ます」でよろしいですか?

318 :名無しさん@英語勉強中 :2022/07/04(月) 05:39:45.61 ID:es6H4YfTM.net
普通に訳したら「私はすぐにここに来ます」と思うのですがおかしいでしょうか?

319 :名無しさん@英語勉強中 :2022/07/04(月) 06:09:39.85 ID:sBI8Y4gb0.net
ネイティブがそのセリフを言う時に、rightとhereそれぞれに人差し指を下に向けて床に指すのをよく見る
(説明下手でスマン)
だからrightはこの場合、すぐにではなくて、ちょうどって意味だと覚えてた

I’ll be right here.
ちょうど(まさに)この場所で待機しています

I’ll be right there.
だと、すぐにそこに行きます、になると思う。

320 :名無しさん@英語勉強中 :2022/07/04(月) 06:47:12.67 ID:IQ2FFxk4M.net
>>319
ありがとうございます。
I'll be rightと聞くとrightをすぐにと頭で変換されますが、後に続くのがhereならrightをちょうどに変えないといけなくなりますね。
ネイティブとか英語上級者の方はどのように理解されているのでしょうか?
リーディングならいいけどリスニングだと理解が追っつかなくなります。

321 :Seeker2022 ◆iymFBv30wPKz (ワッチョイ 9322-auNL):2022/07/04(月) 11:38:11 ID:Q0coGi5y0.net
>>320
副詞のrightを「すぐに」と訳すのは、基本的に「時間」関係のみです。 right hereは時間ではなく、場所なので「すぐに」という意味にはなりません。

I'll be right back.は時間関係なので、「すぐに」と訳します。
また時間関係でも「right now」のような場合は、「まさに」という強調の働きをしています。

322 :Seeker2022 :2022/07/04(月) 11:52:44.53 ID:Q0coGi5y0.net
補足
上の説明がちょっとわかりにくいので追加説明しますね。

「right here」・・・「here」という言葉自体に時間の概念が関与する余地はありません。
「right back」・・・「back」という言葉には「(未来に)帰ってくる」という意味合いの強い単語です。

副詞(節)自体に「時間の概念」が関与する可能性があるかどうかで判断できると思います。

323 :Seeker2022 :2022/07/04(月) 11:57:59.70 ID:Q0coGi5y0.net
>>314のit takes のitに関しては、非人称のitであるとする考え方と形式主語のitであるとする考え方の2つがあります。
従って、どっちが正しいと断定することはできません。

論拠
https://sakura-paris.org/dict/%E3%83%AD%E3%82%A4%E3%83%A4%E3%83%AB%E8%8B%B1%E6%96%87%E6%B3%95/content/318_788

324 :名無しさん@英語勉強中 :2022/07/04(月) 13:13:33.99 ID:X9sy3gUC0.net
エレクトロニック
エスニック
バイオニック

では[名詞]ニックという造語作ろうと思えばできるってこと?
[名詞]的な〜という表現として

325 :名無しさん@英語勉強中 :2022/07/04(月) 13:22:45.79 ID:eCxLlel80.net
A raise concerns about B
・AはBについて懸念を示す←AはBの被害者側
・AはBについて懸念を引き起こす←AはBの加害者側
2つ意味があって何か真逆っぽいです
どちらもBをアゲアゲしてるってことで、単にAとBの関係によって(表面的な)訳が
違うだけということでしょうか?

326 :名無しさん@英語勉強中 :2022/07/04(月) 13:25:57.58 ID:IQ2FFxk4M.net
>>321
ありがとうございます。
最初にrightと言われたら「ちょうど、まさに」の意味で理解すればいいのか、判別するのが難しいです。
英語は一つの単語に複数の意味があって大変です。(日本語も一つの漢字で発音は変わるけど色んな意味がありますが)

327 :名無しさん@英語勉強中 :2022/07/04(月) 13:26:22.51 ID:IQ2FFxk4M.net
すみません、もう一つフレーズで聞きたいことがありのですが
I won't see you until Sunday.で日曜日まで会えないね
という和訳だったのですが、この文章だとあなたには日曜日まで会わない(だろう)になるのではないでしょうか?
フレーズの和訳ならI can't see you until Sundayのほうがしっくりきますがおかしいですか?

328 :😉三年英太郎🌈 :2022/07/04(月) 13:41:30.19 ID:A7AHjYcF0.net
>>113
>虎の威を借るキツネのように文献に依存し、自分の頭で考え説明することができない三年ちゃん。たまには自分の言葉で説明しようよ^^

>>323
論拠はロイヤル(ドヤ

これはどう考えたって形式主語で、ロイヤルがなぜこんな珍説を披露しているのか理解に苦しむ

329 :名無しさん@英語勉強中 :2022/07/04(月) 14:39:41.15 ID:0LVc/He90.net
>>323
ありがとうございます。
it takes three years toの例文も載っていますよね。

非認証のitという場合もある、ということですね。

330 :名無しさん@英語勉強中 :2022/07/04(月) 15:07:10.71 ID:be3vI1Vja.net
>>324
出来るんじゃないかな
そういう言葉遊び的なのは英字新聞でよく見かける気がする

331 :名無しさん@英語勉強中 :2022/07/04(月) 15:39:14.92 ID:niGjCYqu0.net
>>321
さすがにいい加減じゃない?
時間関係でしかすぐにとは訳さない
場所だからすぐにと訳さない
あ、nowは時間だけどすぐにじゃないよ
あ、時間関係の単語じゃなくても時間関係に受け取れるならすぐにだよ
i'll be right there.
i'll be right with you.
は?

大概はすぐにの訳でいいと思うけど
right→余分なことをせずまずは、
とかそんな意味だと思うよ
i'll be (right) □
□で意味が決まるわな

332 :三年英太郎 :2022/07/04(月) 19:38:17.65 ID:A7AHjYcF0.net
PEU4 §268

preparatory "it": subject

When the subject of a clause is an infinitive expression,
this does not normally come at the beginning. We usually
prefer to start with the ‘preparatory subject’ it, and to
put the infinitive expression later (long or complicated
items are often put towards the end of a sentence.)

3. It takes … + infinitive
・It took me months to get to know her.
・How long does it take to get to London from here?

333 :三年英太郎 :2022/07/04(月) 19:42:49.53 ID:A7AHjYcF0.net
It will take you an hour to complete the task.
→ To complete the task will take you an hour.

どこをどう考えたら非人称のitになるのか意味不明

334 :Seeker2022 :2022/07/04(月) 20:02:34.39 ID:Q0coGi5y0.net
>>333
形式主語だと言っている学者もいれば、非人称のitだと言ってる学者もいるってことだよ。

ジーニアスとフォレストが非人称のitの立場を取ってるようだから、あきらめて両論可能だと認めなさい ^^

ここが参考になるよ
https://detail.chiebukuro.yahoo.co.jp/qa/question_detail/q13127223873

335 :Seeker2022 :2022/07/04(月) 20:12:25.61 ID:Q0coGi5y0.net
>>331
「right here」は「すぐにここに」と単独で言えないから、「すぐに」ではない。
「right back」は、「すぐにもどって」と単独で言えるから、「すぐに」である。

じゃあ、この「すぐに」と言えるかどうかの違いは何なのかって話。

それは、副詞が「時間関係」かどうかで決まるというのがいちば理にかなってるんだよ。
他に、これより良い見分け方があったら、披露してみたまえ。

よりマシな説明もできないのに偉そうにしてはいけないよ

私の理論
「right+副詞」の副詞が時間関連でないなら「すぐに」とは訳せない。時間関連である場合は、「すぐに」と訳す場合と「まさに」と訳す場合がある。

さあ、これより良い理論があるなら聞かせてもらおうか。

336 :Seeker2022 :2022/07/04(月) 20:45:57.24 ID:Q0coGi5y0.net
>>327
I won't see you は、自分の判断で会わないという感じです。
他方、I can't see you.は、会いたくても会えないという感じです。

「本人の判断で」という意味合いがある場合は、won't seeを「会えないね」という文脈で使っても差支えないでしょう。

I won't see you until Sunday because I have a lot of things to do. 「やることが一杯あるので会えないね」(意訳)
これを文法書どおりに「やることがいっぱいあるので会わないだろう」は日本語として不自然だとは思いませんか?

337 :😉三年英太郎🌈 ◆3CZBjOt3.Y (ワッチョイW 8a86-bbxY):2022/07/04(月) 21:27:58 ID:A7AHjYcF0.net
>>334
学者不在

338 :😉三年英太郎🌈 :2022/07/04(月) 21:38:01.50 ID:A7AHjYcF0.net
>①後続のto不定詞が「~するには,~するためには」という意味の副詞的用法で使われているので,代名詞であるitは副詞的用法のto...を指すことができない。英米人もこの考え方に近い。

これは完全な目くらまし

「当該文のto 不定詞が、副詞的用法で使われているとする根拠はなにか」と聞かれているのに、その答えが「副詞的用法で使われているから」と言ってる

英米人もこの考えに近い←Swanが違うじゃん

339 :😉三年英太郎🌈 :2022/07/04(月) 21:40:10.12 ID:A7AHjYcF0.net
>>334
他人の飲用じゃなくて自分で説明しろ、というのがseekerの言ってきたことなんだから、自分で証明してみ?

340 :名無しさん@英語勉強中 :2022/07/04(月) 21:43:45.59 ID:IQ2FFxk4M.net
>>336
グーグル翻訳で日曜日まで会えないと打つとI can't see you until Sundayと出るんですが、さっき会えない「だろうね」とするとI won't see you until Sundayになったんですよ
336さんの言うとおり本人の判断でwillの意思が反映されたってことで凄く納得できました!

341 :Seeker2022 :2022/07/04(月) 21:48:03.92 ID:Q0coGi5y0.net
>>339
どちらでも解釈できるでしょ。一番偉い人がどう言ったかじゃなく、科学的に数学的に見た場合、他方を否定できないのである。

342 :Seeker2022 :2022/07/04(月) 22:02:21.00 ID:Q0coGi5y0.net
>>331
こういえば、納得してくれるかな。このrightって基本強調語なのだよ。
だから、「right here」は「here」を強調しているし、「right now」は「now」を強調している。

「right back」は「back」を強調していて、「もどって」とは「変化・推移」のことだから、強調すると「すみやかに変化する」という感じになるわけだ。
このように「変化・推移・時間の経過(after/before)」を示す語を強調すると、自然と訳は「すぐに」となる。

「now」は時間であるけど、「変化とか時間の推移」と関係のない言葉なので「すぐに」とはならない。

<最終理論>
強調のrightが副詞・前置詞・副詞節を修飾するとき、それらが「変化とか時間の推移」に関するものであれば、「すぐに」と訳す。

343 :Seeker2022 :2022/07/04(月) 22:11:39.85 ID:Q0coGi5y0.net
>>340
先ほどの補足なのですが、強調のrightが副詞・前置詞・副詞節を修飾するとき、
それらが「変化とか時間の推移」に関するものであれば、「すぐに」と訳すと覚えておくと良いでしょう。

back 位置が変化する
after/before 時間の推移

これらのまえにつくrightは「すぐに」と訳します。

hereやnowは、「変化とか時間の推移」とは無関係な語ですので、「すぐに」とは訳せません。

344 :Seeker2022 :2022/07/04(月) 22:18:11.17 ID:Q0coGi5y0.net
>>339
実際に「非人称のitと形式主語の2つの考え方がある」とする文法書がある以上、安易にこっちが正解と言えるわけがない ^^
個人的には、不定詞部を主語に持っていけるので、形式主語でいいんじゃないのとは思うけどね。

345 :名無しさん@英語勉強中 :2022/07/04(月) 22:29:03.66 ID:niGjCYqu0.net
>>342
俺が上げた例も説明しとけ

346 :名無しさん@英語勉強中 :2022/07/04(月) 22:30:55.27 ID:0LVc/He90.net
>>332
今確認しました。
4
Preparatory itのところを見ました。

swannはそう考えるんですね。

347 :Seeker2022 :2022/07/04(月) 22:32:10.07 ID:Q0coGi5y0.net
>>345
まずは自分で説明しろよ

348 :名無しさん@英語勉強中 :2022/07/04(月) 22:44:11.95 ID:niGjCYqu0.net
「right there」・・・「there」という言葉自体に時間の概念が関与する余地はありません
が!
rightはすぐにうけとっていい意味です

「right with you」・・・「with you」という言葉自体に時間の概念が関与する余地はありません
が!
rightはすぐにと受け取っていい意味です

コピペしたげたよ

349 :三年英太郎 :2022/07/04(月) 22:58:31.61 ID:A7AHjYcF0.net
>>344
ロイヤルは高校教師が書いたまとめ本で、第一級の文法書ではない。

*************************

Downing&Locke: A University Course in English Grammar, p.37 参考になった。
Prop it と言われるものの中にも程度の差があるというお話。

it takes .... が非人称の it だと最初に書いたのはGではなかろうか。
それか他の小西本。それをロイヤルが写したように思われる。
ほかにこんな奇妙なこと言ってる本が見つからないから。

350 :Seeker2022 :2022/07/04(月) 23:03:27.94 ID:Q0coGi5y0.net
>>348
そういうレスだったら、問題ないよ。

「right there」は、「まさにその場で恋に落ちる」→「即座に恋に落ちる」という意訳。本来は「まさに」

right with youは、「be right with you」というイディオム(慣用表現)。普通の文法では対処できなので、慣用表現扱いになっている。
慣用表現は例外だから仕方ない。


参考
https://idioms.thefreedictionary.com/be+right+with+you

351 :Seeker2022 :2022/07/04(月) 23:19:01.29 ID:Q0coGi5y0.net
>>349
さすがにロイヤル英文法の著者も「ネイティブの書いた原書」を参考にしてると思うけどなあ・・・

だから、私には怖くて、一方のみが絶対的に正しいとはいえないよ

352 :Seeker2022 :2022/07/04(月) 23:57:51.36 ID:Q0coGi5y0.net
>>348
補足
>>319の違いがなぜでるかと言えば、

I'll be right there.→ 「(there近くまで来ているが)すぐに"まさにそこ"まで行くよ」という意訳が可能。
しかし、所詮「まさに」のright.

I'll be right here.は、すでにhereにいるのだから場所を移動しないので、「すぐに」と意訳できない。これも、「まさに」のright.

353 :名無しさん@英語勉強中 :2022/07/05(火) 00:24:39.49 ID:zAUQP9Gu0.net
>>352

つまり、すぐにって意味だよね
すぐにと訳さない理由は?

354 :Seeker2022 :2022/07/05(火) 00:41:53.64 ID:iaFlvsQu0.net
>>353
「まさに」と訳すrightなのだから、「すぐに」というのは別になくてもよい。
I'll be right there.→ 「まさにそこまで来てるよ」と訳しても構わない。

355 :名無しさん@英語勉強中 :2022/07/05(火) 00:51:15.85 ID:zAUQP9Gu0.net
>>354
つまり、すぐに
と訳してはならないと言ってるんだよね?
そこ確定させようか

356 :Seeker2022 :2022/07/05(火) 01:13:47.91 ID:iaFlvsQu0.net
>>355
「すぐに」と意訳できると書いてるように、誰も「すぐに」と訳してはいけないなんて一言も言ってない
話を盛るのはやめなさい

自分は、「まさに」と訳して問題ないものは「まさに」に分類するのが妥当だと考える。「すぐに」と意訳できる場合もあるだろうけど。
他方、I'll be right back.のような「まさに戻ってくる」と訳すと不自然なものは、「すぐに」で解釈したい。

これはあくまでも解釈の問題なので、「すぐに」と意訳できるのなら「すぐに」に分類したいと思うのであれば、それも間違いとは言えない。
あくまでも、自分の解釈説を述べているだけ。当然、他の分類説があってもよい。賛成しないけど。

357 :Seeker2022 :2022/07/05(火) 01:31:54.34 ID:iaFlvsQu0.net
>>355
I'll be right there.のrightが「まさに」じゃないと主張するのなら、I'll be right here.のrightが「まさに」で、
I'll be right there.のrightが「すぐに」と用法が変わってしまう理由を説明してみなよ

※I'll be right here.はすでに「ここ」にいるので「すぐにここにいる」とは訳せない

358 :名無しさん@英語勉強中 :2022/07/05(火) 04:34:06.64 ID:V3pXhoNj0.net
>>349
peuはセカンドランゲージの実践をやってた人が授業でやったことをまとめて
あの本になったというのをスワンのブログか何かで見たよ。オックスフォードだからといって
だから権威があるんだと思わないほうがいいと思うんだよね。

359 :名無しさん@英語勉強中 :2022/07/05(火) 04:35:54.32 ID:V3pXhoNj0.net
語学に権威づけなんか持ち込んでもなんのためにもならないかもと思うんだ

360 :Seeker2022 :2022/07/05(火) 07:26:43.86 ID:iaFlvsQu0.net
まとめ

I'll be right there.は、「もうそこまで来ているよ」または慣用表現で「(人に呼ばれて)今行くよ」という意味である。
論拠
https://idioms.thefreedictionary.com/be+right+there

「もうそこまで来ているよ」「今行くよ」というのは、確かに「すぐにそこに行くよ」と訳せるけど、それは意訳に過ぎない。
本来の意味は、「ほぼその場所にいるけどまさにその場所に行くからね」というニュアンスである。

九州にいる人が電話で東京にいる人に「すぐにこれから電車に乗って東京にいくよ」という文脈では一般的に使わないのである。
本来は、「まさにその場所」の意味なのである。

>>355  わかったかな?

361 :Seeker2022 :2022/07/05(火) 07:35:40.52 ID:iaFlvsQu0.net
be right there.の定義↓
To be very close to arriving at the location of the listener.

使用例↓
I'm around the corner, so I'll be right there.

362 :Seeker2022 :2022/07/05(火) 08:52:40.02 ID:iaFlvsQu0.net
I'll be right with you. も、I'll be right there.と同じように、「これからまさに一緒になります」と書いて、「現在ほぼ一緒です」という意味を出すことができ、「もうすぐ対応できます」という慣用表現が出来上がったもの。

元々「すぐに」のrightではないので、横浜にいる人が電話で東京にいる人に「すぐに会えるよ」という文脈では使わないのである。

使用例
Thank you all for your patience. Someone will be right with you.
(みなさん、辛抱頂きありがとうございます。誰かがすぐに対応します。)

363 :Seeker2022 :2022/07/05(火) 10:37:13.45 ID:iaFlvsQu0.net
追記

I'll be right with you.は、「現在ほぼ一緒です」という含みがあるため、目の前にいる人たちに向けて使います。
すでに「with you」がほぼ成立している状況なのです。

カウンターサービスの係の人が待ってる客によく使う表現です。

364 :Seeker2022 :2022/07/05(火) 10:45:48.13 ID:iaFlvsQu0.net
訂正

追記

I'll be right with you.は、「現在ほぼ一緒です」という含みがあるため、目の前にいる人たちに向けて使うことが多いです。
すでに「with you」がほぼ成立している状況なのです。

カウンターサービスの係の人が待ってる客によく使う表現です。

365 :😉三年英太郎🌈 :2022/07/05(火) 11:51:19.73 ID:w6Kekh+V0.net
>>358
ワイがPEUだけしか見てないと思うか🧐
CGEL、講義、コンサイス英文法辞典も確認済み

366 :名無しさん@英語勉強中 (ワッチョイ 276a-9ZeA):2022/07/05(火) 15:51:24 ID:OaIDVCFx0.net
田中さん、貴方に野球の大会に出場してほしかった

って英語でどう書くんでしょうか?

367 :名無しさん@英語勉強中 (ワッチョイW 539d-gfLS):2022/07/05(火) 16:07:20 ID:3GKCvrDh0.net
how come you didn't entry the baseball league, mr.Tanaka?

368 :名無しさん@英語勉強中 :2022/07/05(火) 19:01:14.89 ID:yRHMoz1X0.net
TOEICの長文間に合わないんですけど取れる人はみんな「チャンク読み」とやらをやるのが常識なんですか?

369 :名無しさん@英語勉強中 :2022/07/05(火) 19:10:12.41 ID:Qxr3gpPvM.net
>>368
単純に読むスピードが足りないんだと思う。
それは単語力不足と普段から読んでる英文量か不足してる。あと数こなして広告系はどう言う構成なのか、メール系はどう言う段落構成なのかを意識しながら読むといいかと。ある程度読み込めば自然と読むスピードは早くなる筈。

あと答え合わせして解きっぱなしにしてない?解説見ながら英文と選択肢を読み直してわからない単語や表現は確認しといた方が良いよ。

370 :名無しさん@英語勉強中 :2022/07/05(火) 19:40:42.25 ID:0Bd4Ya+U0.net
>>366
Mr. Tanaka, I wish you'd entered the baseball tournament.

371 :名無しさん@英語勉強中 :2022/07/05(火) 20:15:05.45 ID:yRHMoz1X0.net
>>369
ありがとうございます
単語と読み込みやってみます

372 :名無しさん@英語勉強中 :2022/07/05(火) 22:16:27.37 ID:lTBRfR8H0.net
>>330
ありがとう

373 :名無しさん@英語勉強中 :2022/07/05(火) 22:24:21.08 ID:lTBRfR8H0.net
野生のトラなどの獰猛な強暴(凶暴ではなく)な様子を表す場合
brutal
ferocious
savage
で迷っているのですが、凶行、残忍、冷徹、無慈悲、凶悪、などのイメージから一番遠いのはどれですか?
つまり野生、野性的wild wildlyの表現に近いものを探しています。

savageかなあと思ってるのですが

374 :名無しさん@英語勉強中 :2022/07/05(火) 22:55:12.36 ID:CEf43OTJ0.net
>>373
aggressive
untamed

375 :名無しさん@英語勉強中 :2022/07/05(火) 22:59:33.75 ID:lTBRfR8H0.net
>>374
ありがとう

376 :名無しさん@英語勉強中 :2022/07/05(火) 23:39:32.08 ID:9OM7HMv10.net
I want to single out David Sacks, my research assistant, for all he did to make this book both possible and better.
The research load was heavier than usual given the wide range of subjects covered.
David has the gift of being able to produce prodigious amounts of quality work at warp speed.
All that along with the fact he was not shy about voicing his own informed views about what I had written made him an indispensable partner throughout the writing and editing process alike.
の最終文はどんな構造になっているのでしょうか?

377 :名無しさん@英語勉強中 :2022/07/06(水) 02:35:01.82 ID:QJ8PDJ3R0.net
all that made him a partner.
です

378 :名無しさん@英語勉強中 :2022/07/06(水) 06:36:56.81 ID:Qaj1gpoR0.net
(1) All that along with the fact he was not shy about voicing his own informed views about what I had written made him an indispensable partner throughout the writing and editing process alike.

こんなのは、

(1a) All that--along with the fact he was not shy about voicing his own informed views about what I had written--made him an indispensable partner throughout the writing and editing process alike.
(ただし -- は、実際には長い横棒つまり dash にする)

または

(1b) All that, along with the fact he was not shy about voicing his own informed views about what I had written, made him an indispensable partner throughout the writing and editing process alike.

というふうに筆者が書いていてくれさえすれば、迷わずに >>377 の言ってくれてる通り
だってことがわかるのに、この筆者はそういうふうに文章を遂行する時間がなかったのかもしれんな。

379 :名無しさん@英語勉強中 :2022/07/06(水) 06:38:35.28 ID:Qaj1gpoR0.net
訂正
文章を ***推敲*** する時間がなかったのかもしれんな

380 :Seeker2022 :2022/07/06(水) 08:55:49.76 ID:sl5hz2N70.net
>>365

https://blog.abaenglish.com/impersonal-sentences-with-it/

ここで「It takes 2 hours to get to Paris」を非人称のitで説明してるから、現在は、非人称のit説が結構浸透してそうな予感 ^^

381 :Seeker2022 :2022/07/06(水) 08:59:46.75 ID:sl5hz2N70.net
非人称のit説は、決して日本人の研究者が間違ってつくった理論じゃないってことだね

382 :Seeker2022 :2022/07/06(水) 09:06:32.44 ID:sl5hz2N70.net
両論併記している参考書は、ロイヤル英文法しかないってことは、やっぱりロイヤル英文法はすごい!

383 :Seeker2022 :2022/07/06(水) 10:05:29.89 ID:sl5hz2N70.net
>>333

https://engxam.com/handbook/how-we-use-impersonal-pronouns/

この英国のサイトのページも、「How long does it take to get to the airport?」を非人称のitと解釈してるね。

もちろん、形式主語のitだとするページもあるので、「2つの考え方がある」がこの場合のベストアンサーなのだよ。

384 :😉三年英太郎🌈 :2022/07/06(水) 11:44:53.86 ID:45Ai9t9+H.net
>>380
このおバカさんは、いつもどこの馬の骨か知れんやつの書いたことを探してきてドヤってるね。繰り返すが雑

ネット上にはit's easy to read the book とかでも impersonal it とか言ってるレベルのサイトがごまんとある

間違った日本語文法を教えている日本語ネイティブのサイトがごまんとあるのと同じ

385 :😉三年英太郎🌈 :2022/07/06(水) 11:46:39.35 ID:45Ai9t9+H.net
なお、わたくしはバカンスに出てるので、これから9月まではまともな文献に当たれないことを申し上げておこう😎

386 :名無しさん@英語勉強中 :2022/07/06(水) 12:54:45.22 ID:8r/cYqZ60.net
>>376
文法的な補足をすると、

・All that の that は前の文の内容を受ける代名詞。

・the fact の後に同格の接続詞の that が省略されています。

All that along with the fact (that) 〜 で、
「〜 という事実と共にそういうことすべてが 〜 」

直訳的な全訳:
「私が書いたものに関して、彼が自分の広範な知識に基づいた意見を表明
することにためらわなかった事実も合わせ、そういうことすべてが執筆の
間中また編集の過程においても、彼を欠くべからざるパートナーにした」

387 :名無しさん@英語勉強中 :2022/07/06(水) 16:40:49.13 ID:QIUmwGzp0.net
語学学習のためのMP3プレーヤーを探しています。
現在ウォークマンの古い機種で再生スピードコントロールが付いているものを使っています。
音声を早くしたり遅くしたりできるので重宝しているのですが、古いモデルのため充電容量が
小さくなってきており1時間で電池切れを起こします。
安価なMP3プレーヤーがあったら良いなと考えて数日探しているのですが、迷っています。
例えばヤフーショッピングで「MP3プレーヤー」と検索して一番最初に出てくる2,970円の
ものなどどうでしょうかね?「変速再生」ができるようです。
色々調べる中でウォークマンの再生スピードコントロールは中々優秀で、再生スピードを
変えても音声の質の変化がなく違和感がないようです。
なのでウォークマンを電池交換しようかなとも考えています。意外と高いのですけどね。
もし安価な(3~4000円くらい)MP3プレーヤーで語学学習してらっしゃる方がいらっ
しゃったら、使い勝手の感想をお聞きしたいのですが…。
よろしくお願いいたします。

388 :名無しさん@英語勉強中 (ワッチョイW 46d9-hDPK):2022/07/06(水) 17:05:20 ID:NfK2/fSn0.net
>>387
俺は取り込んだmp3をソフトで編集で変速してるから分からないな。

リスニングは倍速で聞くよりは同じ音源を文章がカンペ無しで文章が思い浮かべるようにする方が確実かと。色んな音源に触れるのも悪くないけど両方やった方が良い。

389 :名無しさん@英語勉強中 :2022/07/06(水) 18:21:09.13 ID:QIUmwGzp0.net
>>388
おぉ、早速ありがとうございます。
MP3をソフト編集しているんですね。私、結構ロートルなのでパソコンはエクセルと
ワードくらいしか触ったことがないんですよね。(照笑)
単語帳を倍速で聴いて回しているのでリスニングではないです。
単語帳で倍速でシャドウイングすると喉発音が自然とできるようになるので不思議です。
これは発見でした。
リスニングはCNNのEEを聴いているのですが、ナチュラルスピードはなかなか難しいです。
ゆっくりは2~3回聞けば理解できるんですけど。
そんな感じです。

390 :名無しさん@英語勉強中 :2022/07/06(水) 18:21:20.29 ID:ISFdyIFTM.net
スマホの音声再生アプリで探したほうが早い気もする(というか自分はそうしてる)

391 :名無しさん@英語勉強中 :2022/07/06(水) 18:46:31.17 ID:QIUmwGzp0.net
>>390
ありがとうございます。
それもありかもですね。
これもバッテリーの問題が出てくるんですよね。
外付けバッテリーを買いますかね。

392 :Seeker2022 :2022/07/06(水) 21:24:58.42 ID:sl5hz2N70.net
>>384
まーた、負けると罵倒か・・・どっちが頭の悪い対応なのかな?
IQならぬEQこそが前頭葉の真の知性を測るメルクマールだと言われている。
すぐ罵倒に走る人たちってのは一般的に知能が高くないんだよ。

https://blog.abaenglish.com/impersonal-sentences-with-it/

これってそこそこ名も知られているオンライン英語スクールだぞ
https://apps.apple.com/jp/app/aba-english/id859243872

間違っていたらすぐに反論があって訂正するはず。
だれも反論してないのだから、非人称のit説でも問題ないと多くの人が考えている証拠。
いい加減、あきらめて、ロイヤル英文法の「2つの考え方がある」という主張を認めようよ ^^

「日本人の研究者」が勝手に考えたという三年ちゃんの意見は的外れなのだ。次にそれをさらに証明していこう

393 :名無しさん@英語勉強中 :2022/07/06(水) 22:31:39.26 ID:7BRDexaa0.net
>>377
>>378
御教示感謝。
形容詞+指示代名詞なのね。
外形的に他の可能性はなさそうだけど、
こういう使い方をする英文に会った記憶がないので判断しかねましたわ。

394 :Seeker2022 :2022/07/06(水) 23:49:15.41 ID:sl5hz2N70.net
>>333
「ロイヤル英文法に載ってる『2つの考え方がある』は日本人の考えた間違った説だ」というのが3年ちゃん意見だけど、調べたところ、「非人称のit」だと思っているネイティブの英語インストラクターも複数いるのだから、そのような独自理論を披瀝するネイティブ英文法学者がいるのかもしれない。
自分は確認してないが、ロイヤル英文法以外にもジーニアスとフォレストが「非人称のit」の立場を取っているというネットの書き込みもひっかかる。
果たして、日本の研究者が英語の原文を読まずに、どっかの日本人の言うことを間に受けてそのまま書物にするだろうか? 「日本人の考えた説」というところがどうも信じられない。

前にも言ったように自分も「to以下を指すit」のほうが断然理にかなった解釈だとは思うけど、非人称 it 説を「日本人が考えた誤り」と判断するのは早計だと考える。
「とあるネイティブ学者が考えだしたマイナーな分類法」という可能性も捨てきれないからである。

395 :Seeker2022 :2022/07/07(木) 00:07:36.62 ID:3o3fcFxj0.net
>>349
誤解されると困るので補足。
私が「ネイティブの非人称it説のページ」のリンクを載せた目的は、「非人称it説」が優勢であることを言うためでは決してないのだ。
「to 以下を指すit」の解釈が圧倒的に理にかなっていると自分は思ってる。

リンクを載せた目的は、「非人称it説が日本内だけの考えではない」ことを示すためである。
3年ちゃんの「日本人の考えた」の部分に疑問を呈するためである。
そして、「そういうふうに考えるネイティブも少なからずいる」ことを紹介するためである。

396 :名無しさん@英語勉強中 :2022/07/07(木) 01:55:36.62 ID:hfMNt9dF0.net
>>365
ありがとう

397 :名無しさん@英語勉強中 :2022/07/07(木) 09:40:42.12 ID:ZgSZrSl700707.net
runner-upは「二位の選手」という意味らしいですが、この綴りがどうしてこの意味になるのかわからないのですがご存じの方教えてください

398 :名無しさん@英語勉強中 :2022/07/07(木) 14:25:49.96 ID:hkZG6YVK00707.net
>>397
1位のやつに負けるまで勝ち続けたやつやからやで

399 :Seeker2022 :2022/07/07(木) 16:44:34.43 ID:3o3fcFxj00707.net
>>394の訂正

(誤)ロイヤル英文法以外にもジーニアスとフォレストが「非人称のit」の立場を取っている

(正)ロイヤル英文法は両論併記であり、また、ジーニアス・フォレストは非人称のitという立場を取っている情報がある

400 :名無しさん@英語勉強中 :2022/07/08(金) 05:06:11.80 ID:Qx+5ZBCt0.net
(1) runner-up

(2) The dictionary says it means "a competitor or team taking second place in a contest."

(3) In other words, "runner-up" means something along the lines of
"someone who has run up to the winner," meaning "someone who has come close to the winner."

(4) come up to
He came up to me and whispered something.

(5) come running up to
He came running up to me and whispered something.

(6) run up to
A dog ran up to me.

401 :名無しさん@英語勉強中 :2022/07/09(土) 04:14:41.37 ID:E9P4lUTH0.net
wannabeみたいなもんかな?

402 :名無しさん@英語勉強中 :2022/07/09(土) 08:58:37.35 ID:DXbTlywYa.net
しかし本当に銃自作するやつがいるとは
俺も銃自作して自殺する妄想してたけどテストする場所がないから速攻で諦めたよ
わざわざ山とか行ってテストしてたんだろうか、すげーわ
職質されたら一巻の終わりだし音でバレる可能性もなきのしもあらずなのによくやるわ

403 :名無しさん@英語勉強中 :2022/07/09(土) 12:58:44.50 ID:sOFcbg3cM.net
It took my son just 15 seconds to solve this puzzle.
私の息子はこのパズルを解くのに15秒しかかかりませんでした。

この英文をMy son took just 15 seconds to solve this puzzle.
にしたら同じ意味になりますか?

404 :名無しさん@英語勉強中 :2022/07/09(土) 13:09:51.83 ID:uxijhOs00.net
なるよ

405 :名無しさん@英語勉強中 :2022/07/09(土) 16:06:45.47 ID:WERVPMqb0.net
>>403
(1) It took my son just 15 seconds to solve this puzzle.
15秒しか ***かかり** ませんでした。

(2) My son took just 15 seconds to solve this puzzle.
15秒しか ***かけ*** なかった。

took (take) の主語を人間にすると、「(意識的に)それだけの時間しか ***かけなかった***」
とか「(意識的に)それくらいの長い時間を ***かけた***」という意味になるんじゃないだろうか?

took (take) の主語を it (= to do something) にして、たとえば
It took me two hours to do this. にすると、「2時間がかかった」つまり自分が2時間という
長い時間をかけたい(あるいは2時間しかかけたくない)かどうかに関わらず、
自然とそれだけの時間がかかってしまった(あるいは自然にそれだけの時間しか
かからなかった)という意味になるんじゃないだろうか?

と俺には感じられる。ただし俺はこれについてネイティブに尋ねたことはない。

406 :名無しさん@英語勉強中 :2022/07/09(土) 18:59:01.88 ID:T/zlLwbu0.net
↑それは文脈で判断することです。違いはありません。

407 :名無しさん@英語勉強中 :2022/07/09(土) 21:11:13.39 ID:yABFhFDR0.net
よくある願望実現法の内容みたいな内容ですが
自分の中の何か「得意だ」と思えるものに意識の焦点を
合わせるとその(得意なもの。焦点を合わせたもの)が
拡大していくと思う」っていう内容ですが

I believe what you find something being right up your street will continue to expand.

これw 文体が無茶苦茶になってきてしまってw
もっとシンプルな構文にしたほうがいいと思うんですが
二文になってもいいので
何かアドバイスくださいますか?

408 :名無しさん@英語勉強中 :2022/07/09(土) 21:19:29.59 ID:yABFhFDR0.net
ついでにw
「君がもし手加減なしの本気を出せば世界一になるよ」ってのを

pull puch を使ってかきたいんですが

If you are serious, with no punches pulled,you will be the best in the world .

これ変? 翻訳ソフトにかけたら逆の意味に翻訳されてしまう
(まあ良くあることですが)

409 :名無しさん@英語勉強中 :2022/07/09(土) 22:35:40.69 ID:T/zlLwbu0.net
本気を出せばは無理に訳す必要ないですね。なんの世界一かは分からないけど、ボクサーなら、
You'll be the top boxer if you don't pull any punches.

410 :名無しさん@英語勉強中 :2022/07/09(土) 23:31:56.52 ID:A14RmL/O0.net
トランプのdiamond模様は英語でもダイヤって略す?
dia ringとか略されてるのもあるけど
トランプに限ってはどう?

411 :名無しさん@英語勉強中 :2022/07/09(土) 23:56:03.79 ID:OHm7BxFea.net
>>410
Playing card suit - Wikipedia
https://en.m.wikipedia.org/wiki/Playing_card_suit

412 :名無しさん@英語勉強中 :2022/07/10(日) 00:09:53.71 ID:M43TLfOA0.net
>>411
少なくともその英語版のページにはdiaの略語は使われてないことしかわからない

413 :名無しさん@英語勉強中 :2022/07/10(日) 04:16:55.69 ID:oA5ZlQ2/0.net
>>407
>>自分の中の何か「得意だ」と思えるものに意識の焦点を
合わせるとその(得意なもの。焦点を合わせたもの)が
拡大していくと思う

(1) I believe that if you focus on your strength, it will expand.
(2) I believe that if you devote yourself to what you're good at, you'll be better and better at it.

414 :名無しさん@英語勉強中 :2022/07/10(日) 07:48:30.42 ID:oA5ZlQ2/0.net
>>408
not pull your punches
not pull any punches

この2つの言い回しを「手加減しない」というふうに辞書に書いてあるからと言って、
どういう場合にでも「手加減しない」という日本語にぴったりなんだと思ったら
間違いであるらしいことは、辞書の解説や例文をチラッと見ればわかるだろう?

「(言論において)手加減しない」つまり「言いたいことをズバリと言う」って
いう文脈で使うんだよ。

しかし本来はボクシングで「(相手にパンチをかますときに)手加減しない」
という意味で使う言葉だから、文字通りその意味で使うことももちろんあるし、
その延長線上でもう少し広い意味で使うこともあるだろうけど、たとえば
サッカーとか剣道とかチェスとかビジネスとか政治の面で
「手加減しないで手加減しないで相手をどんどん打ち負かして
(その結果として世界一になる)」という文脈でも使えるかどうかはわからない。
少なくとも、そういう例文を探してもなかなか見つからない。

415 :名無しさん@英語勉強中 :2022/07/10(日) 07:58:10.20 ID:oA5ZlQ2/0.net
>>408
not pull your punches とか not pull any punches を使った例文をあちこちで探すと、
次のようなものが見つかる。すべてはオンライン辞書より。

(1) "a sharp-tongued critic who doesn't pull his punches"
(2) She doesn't pull her punches when talking about lazy people.
(3) Her image is that of an investigative reporter who doesn't pull any punches.
(4) My father is very direct and pulls no punches.
(5) She has a reputation for getting at the guts of a subject and never pulling her punches.
(6) Her articles certainly don't pull any punches.

だから、言論面で「手加減しないで何でもズバズバ発言する」という意味でなら、
大いに使えるってことだ。新聞記者とか評論家とかで第一人者になるとかいう
文脈でなら、大いに使える。しかしサッカー選手とか重量挙げの選手として
成功するだろうという文脈でこれが使えるかどうかは、今のところは疑問。

416 :Seeker2022 :2022/07/10(日) 10:04:01.24 ID:NAWBvu560.net
「手加減なしの本気を出せば」は、「可能な限り本気を出せば」に変換して英訳すると良い。


You can be the best in the world if you put your mind to it as much as possible.

417 :名無しさん@英語勉強中 :2022/07/10(日) 10:20:46.77 ID:oA5ZlQ2/0.net
>>408
>>君がもし手加減なしの本気を出せば世界一になるよ

Devote yourself to it, and you'll top the world.

418 :名無しさん@英語勉強中 :2022/07/10(日) 10:39:18.55 ID:z8+Pz8W20.net
pull punch

419 :名無しさん@英語勉強中 :2022/07/10(日) 10:39:40.43 ID:z8+Pz8W20.net
pull any punchを使うのが条件なのに・・・・

420 :Seeker2022 :2022/07/10(日) 10:40:00.88 ID:NAWBvu560.net
「手加減なしの本気を出せば」が、もしボクシングの試合とかなら、「ためらうことなく全力で戦えば」に変換すると良い。
状況次第で英語も変わる。


You can be the best in the world if you fight with all your might and without hesitation.

421 :Seeker2022 :2022/07/10(日) 10:50:35.86 ID:NAWBvu560.net
>>419
pull one's punchesだったらは、ボクシングの試合なら使える。
論拠
https://eow.alc.co.jp/search?q=pull+one%27s+punches

もし、>>408がボクシングの話なら、
You can be the best in the world if you fight with all your might and withou pulling your punches.も使える。

422 :Seeker2022 :2022/07/10(日) 10:52:32.70 ID:NAWBvu560.net
>>421の訂正

pull one's punchesをボクシングの試合の文脈で使うことはできる。

423 :Seeker2022 :2022/07/10(日) 11:02:17.91 ID:NAWBvu560.net
>>408

最終回答

それがボクシングやそれに類する対戦スポーツに関するものなら、
You can be the best in the world if you fight with all your might and withou pulling your punches.
You can be the best in the world if you fight with all your might and without hesitation.

「手加減なしの本気」が単に「可能な限り本気で」のニュアンスであれば、
You can be the best in the world if you put your mind to it as much as possible.

424 :名無しさん@英語勉強中 (ワッチョイ cb89-2HoA):2022/07/10(日) 15:56:09 ID:oA5ZlQ2/0.net
(1) put your mind to it as much as possible
この言い回しに違和感を感じているのは、俺だけかな?これを検索しても、その用例は
1つも見つからない。しかし

(2) really put your mind to it
(3) seriously put your mind to it
これならば、いくらでも用例が見つかる。

他の人がせっかく提示してくれた英訳例にケチをつけたくないんだけど、
あくまで相互研鑽のためだということで、許してほしい。

425 :名無しさん@英語勉強中 :2022/07/10(日) 18:56:04.58 ID:/kTYA0HJ0.net
>>424
書くんなら

put as much mind

みたいな感じだろうかね

426 :名無しさん@英語勉強中 :2022/07/10(日) 18:56:14.83 ID:fUYeGT1g0.net
かなりあいまいな質問だけど許してくれ、 数年前、国会かあるいは政治番組で誰かが使っていたんだよね
政策を主張するなら財源の裏付けもセットで、というような意味だった。 NIMBYみたいなフレーズ on the goとか 
どうしても思い出せない いずれにしても政治で使われるような言葉

427 :Seeker2022 :2022/07/10(日) 19:02:11.10 ID:NAWBvu560.net
>>424は、「pulling your punches」が>>408の文脈では使えないだろうと書いた人だね?
使用例検索を重視し過ぎるとそういう間違いを犯しやすいので要注意だ。

今回は、"可能な限り本気でやれば"を訳したわけだが、実は"可能な限り本気でやれば"って日本語をGoogleのフレーズ検索で調べると誰も使ってないんだよね。
じゃあ、"可能な限り本気でやれば"って日本語は間違いなの?正しいよね。検索に出てこないから間違いだということには必ずしもならないのだよ。

"可能な限り本気でやれば"を原文に忠実に翻訳せよ(ただしボクシングなどではない)、という問題が出題されたら、
if you put your mind to it as much as possible.と書くしかないのである。

質問にあった「手加減しない」という言葉には、「フルパワーを出して」という意味合いがあるので、「really」「seriously」では少し意味のベクトルが違う。
「手加減なしで」→「可能な限りの力を出して」と考えるほうが、表現的にマイナーになるけど、より正確にニュアンスを伝えることができるのだ。

ちなみに、(2) really put your mind to it (3) seriously put your mind to it を和訳しても、「手加減せずに」とはならない。 「手加減せずに」と訳せるようになるには、「全ての力を出しきって」「可能な限り」という意味合いの言葉がなければならないからである。

428 :名無しさん@英語勉強中 :2022/07/10(日) 21:08:40.14 ID:hZuVwibv0.net
>>408
やっぱ何に本気出すのかわからんともやっとするのう

if you do seriously without pulling punches, you can be No.1 in the world.
で要求は満たされるやいなや

429 :名無しさん@英語勉強中 :2022/07/10(日) 21:50:17.07 ID:z8+Pz8W20.net
なんか、427必死だな。。。。

430 :Seeker2022 :2022/07/10(日) 22:37:28.55 ID:NAWBvu560.net
>>429
お金にもならないのに、よくやるよなーとは思う ^^

431 :名無しさん@英語勉強中 :2022/07/10(日) 22:59:14.87 ID:0v/nVBGo0.net
muchはどの単語にかかってるの?
put your mind to it は「〜に注力することを決心する」だがそれに程度とか量があるのかな?

432 :名無しさん@英語勉強中 :2022/07/10(日) 23:11:59.85 ID:h5N2GmLC0.net
合法大麻産業が生んだ「グリーンラッシュ」とは?
北米で熱を帯びる大麻合法化の背景も解説!

「グリーンラッシュ」という言葉を聞いたことはありますか?これは北米を中心に
合法大麻を取り扱うスタートアップが急成長しており、大規模な資金調達なども
行っていることから生まれた造語です。
日本では違法薬物、という印象が強い大麻ですが、現在、アメリカの10の州で
嗜好用大麻が、33の州で医療大麻が利用できます。また昨年10月には先進国
として初めて、カナダが国家を挙げての大麻合法化に踏み出しました。
現時点の予測では、2025年までに合法マリファナ事業の市場規模は240億ドル
(約25兆円)にまで拡大するとされています。

433 :Seeker2022 :2022/07/10(日) 23:50:45.16 ID:NAWBvu560.net
>>431
「決心する」と同じではないよ。「専念する」系だね。
そこで、「dedicate oneself to it as much as possible」(可能な限り専念する)という言い方はあり得るか調べてみよう・・・

https://www.reddit.com/r/chess/comments/unbroy/should_i_pursue_chess_fulltime_made_it_to_2236_in/

ここに
「I would say you should dedicate yourself to it as much as possible without sacrificing other parts of your life, particularly your career and personal life.」とあった。
他にもあったけど、ひとつ挙げれば十分だろう。

434 :名無しさん@英語勉強中 :2022/07/10(日) 23:59:28.61 ID:mH1OEO5Ta.net
>>432
英語板なんだからせめて英語絡めろよ
質問でもねぇし

greenrush | Marijuana Delivery Service | Buy Weed Online
https://www.greenrush.com/

435 :Seeker2022 :2022/07/11(月) 00:04:18.76 ID:QXppsCni0.net
put (one's) mind to (something)
の原義は、
Direct all one's attention to (achieving something)

出典
https://www.lexico.com/definition/put_one%27s_mind_to

436 :名無しさん@英語勉強中 :2022/07/11(月) 00:19:20.12 ID:sqRy1e4e0.net
和訳で考えるな
文字通りmy mindをto itの状態にするputするからmuchなどの程度は使えないから用例も出てこない

437 :名無しさん@英語勉強中 (ワッチョイ 23f0-2HoA):2022/07/11(月) 00:23:54 ID:sqRy1e4e0.net
all one's attentionなら尚更「できる限り」をつけるのはおかしい

438 :😉三年英太郎🌈 ◆3CZBjOt3.Y (ワッチョイW 85e9-vLxR):2022/07/11(月) 00:34:14 ID:i0i2D21O0.net
大麻なんてドラッグのうちにも入らない嗜好品みたいなもんなのに、いまだ厳しく取り締まってくるこの国って何なの?

国民の自由を奪い続けて、何かいいことあった?

それでも未だに自民党に票入れるバカだらけで、うんざりしちゃう😔

439 :名無しさん@英語勉強中 (アウアウウー Sa09-XLJR):2022/07/11(月) 00:40:38 ID:SD7Rb7BLa.net
>>438
馬鹿だろうとは思ってたけど、やっぱり馬鹿だったか
こんなとこにまで政治の話持ち込むとは

440 :Seeker2022 :2022/07/11(月) 01:10:03.69 ID:QXppsCni0.net
>>436 そういう言い方やめろ。礼儀正しく議論できないなら、出てくんな。
この場合は、「可能な限り本気で」という日本語が間違いでないにせよ少し冗長表現なんだよ。なぜなら、「本気で」にはすでに「可能な限り」の意味が含まれているので。
だから、検索で調べても「可能な限り本気で」(日本語)を使っている人はいない。指摘するなら英語ではなく、まずは「可能な限り本気で」という冗長的日本語だ。日本語が冗長なら英語も冗長になってしかたがない。
まあ、気になる人がいるようなんで、betterな表現を載せておこう。

最終回答(訂正版)

それがボクシングやそれに類する対戦スポーツに関するものなら、
You can be the best in the world if you fight with all your might and withou pulling your punches.
You can be the best in the world if you fight with all your might and without hesitation.

「手加減なしの本気」がボクシングに類するものでない場合は、
You can be the best in the world if you go all out.
※ all には明らかに「可能な限り」の意味が含まれているので、この場合は、as much as possibleは明確に不要。

441 :名無しさん@英語勉強中 :2022/07/11(月) 01:25:23.81 ID:sqRy1e4e0.net
「手加減なしの本気を出せば」は、「可能な限り本気を出せば」に変換して英訳すると良い。

↑じゃあこれ間違いってことでいいの?

442 :名無しさん@英語勉強中 (ワッチョイ 23f0-2HoA):2022/07/11(月) 01:37:50 ID:sqRy1e4e0.net
検索しても出てこない日本語に言い換えて、英語でもおかしい文作ってなにがしたいんだ?

please stop spreading bullshit, okay?

443 :Seeker2022 ◆iymFBv30wPKz (ワッチョイ 5522-TkQT):2022/07/11(月) 01:40:13 ID:QXppsCni0.net
>>441
日本語を調べてみた

「本気で取り組む」の意味=「物事に力の全てを出し切って臨むこと」
出典
https://thesaurus.weblio.jp/content/%E6%9C%AC%E6%B0%97%E3%81%A7%E5%8F%96%E3%82%8A%E7%B5%84%E3%82%80

やっぱり、「可能な限り本気でやる」という日本語は、冗長なのね。国語的には「可能な限り本気を出せば」はおかしいと判明。

(ボクシングじゃない場合は)『「可能な限り力を出せば」「可能な限り注意を注げば」に変換して考えるとよい』に訂正。
その場合の英訳が You can be the best in the world if you go all out.

444 :名無しさん@英語勉強中 :2022/07/11(月) 01:45:42.87 ID:W9dizlcX0.net
pulled punch hasn't you be the best in the world

445 :Seeker2022 :2022/07/11(月) 01:47:21.46 ID:QXppsCni0.net
>>442
それでも答えが出せない君よりずっとマシだぞ・・・ ^^

自分は最終的に正答が出せる。しかし、君は出す力がない。Do you understan?

最終回答(訂正版)

それがボクシングやそれに類する対戦スポーツに関するものなら、
You can be the best in the world if you fight with all your might and withou pulling your punches.
You can be the best in the world if you fight with all your might and without hesitation.

「手加減なしの本気」がボクシングに類するものでない場合は、
You can be the best in the world if you go all out.

446 :Seeker2022 :2022/07/11(月) 01:50:16.61 ID:QXppsCni0.net
訂正

>>442
自分は最終的に正答が出せる。しかし、君は出す力がない。Do you understand?

447 :名無しさん@英語勉強中 :2022/07/11(月) 01:55:19.01 ID:sqRy1e4e0.net
I am lost for words
this dude is such...
anyway good luck

448 :名無しさん@英語勉強中 :2022/07/11(月) 06:41:36.03 ID:QqC3MTIz0.net
必死過ぎる・・・・

449 :名無しさん@英語勉強中 :2022/07/11(月) 23:05:36.62 ID:Dk4+VPfy0.net
She torments me.
By her loveliness.

With her eyes.
with her geste.

こういう「彼女の愛らしさが私を苦しめる
彼女の 瞳
彼女の仕草

とかいう場合 withって書いてしまいそうなんですが
 Byなんですか?

使い分けられる?としたらどういう感じででしょう?

450 :名無しさん@英語勉強中 :2022/07/12(火) 00:57:49.00 ID:IHYlqyKt0.net
>>449
どっちでもいいような気がする。
torment (tormented, tormenting, torments) + him + with (by) + her lovely
という string of words で例文を検索し、例文のうちなるべく信頼性の高そうな
ものを探してみても、with であろうと by であろうと、かなり信頼性の高そうな
サイトで見つかる。

(1) with の用例
Philip Hensher · 2008 · &#8206;Fiction
... Gita, who had lived above him for six months, ●tormenting him with her tight black T-shirts
and her heartbreakingly tiny mini-skirts●, tormenting him too ...
(Google Books で見つかる小説の中の一節)

(2) by の用例
Elizabeth Cartwright Penrose · 1854
... his charge by taking a little ride one day for exercise ; and Elizabeth was constantly
●tormenting him by her suspicions of his being too indulgent● .
(小説だと思われるものの中の一節)

(3) もう一つ、with の用例
Elizabeth Lennox · 2019 · &#8206;Fiction
But she clung to him, teasing him more, ●tormenting him with her mouth and her hands● even
as he tried to pull her away. “Megan!” he snapped, his body taut ...
(小説の一節)

451 :名無しさん@英語勉強中 :2022/07/12(火) 00:58:10.76 ID:IHYlqyKt0.net
>>449
torment him by と torment him with を比べる用例をさほどたくさん見比べたわけではないけど、
ざっと見た感じでは、少なくとも with でまったく問題ないし、特にこの上の
(1), (3) の with の用例を見ると、女性が男性を自分の魅力で torment するときに
with を使っているので、これでまったく問題ないらしいと俺は感じる。そういうときに
by でもよいのかどうかとか、どういう微妙な意味の違い(ニュアンス)が
出てくるかについては、俺にはわからん。

一般的に、
他動詞 + 人称代名詞 (him, me, them, etc.) + with [something]
という構文で with を使うことについては、俺は問題を感じていないし、
そういう用例を普段からたくさん見てきたので、俺は安心して使っている。
むしろこういうときに by を使う例をあまり俺自身は見かけていないような気がするので、
俺ならば by をひとまず避けている。

452 :名無しさん@英語勉強中 :2022/07/12(火) 10:29:44.28 ID:QziAT5Zb0.net
とっても丁寧で詳細な解説ありがとうございます!

わたしもwithで書いてみて
念のためDeepLにいれたらw
Byになっちゃったので「え〜?」って思いまして。
自分でももっと調べてみますね!

あと

「こういうイベントを海外の他の国のようにもっと(日本でも)
 大きなもの(大規模のものが開催できるように)にしていこう」っていうのを

「海外のように」の部分、like other countriesだけでも
意味は通じると思いますが
ほんとうは
 (海外で開催されてるように)って
英訳するのがいいのかもって思いますが
そこらへんの関係代名詞がちょい
自信ないのでお願いできますか?

453 :名無しさん@英語勉強中 :2022/07/12(火) 11:44:11.18 ID:xEjpxt6l0.net
女性だからという理由で選ばれたるのを嫌がったを訳するとき、クォーテーションのところって存命かどうかに関わらずwasであってますか?

she didn't want to be singled out because she "was" a female.

454 :名無しさん@英語勉強中 :2022/07/12(火) 11:58:11.68 ID:IHYlqyKt0.net
>>452
>>こういうイベントを海外の他の国のようにもっと(日本でも)
>>大きなもの(大規模のものが開催できるように)にしていこう

今、俺が咄嗟に思いついた英文。もっとじっくり考えなおして別の表現を
教えてくれ、というのならまた考えなおしてはみるけど、そんなに
考えなくても下の (1) で十分だという気がする。

もっと複雑な英文というものは、
そういう複雑なものでないとどうしても表現しきれない場合になって初めて
使うという態度を取った方がいいような気がする。単純な英文で事足りているときは、
単純なままでいいと俺は思う。

(1) I want us to hold these events in scales as big as in other countries.
(2) I want us to hold these events in scales as big as those they hold elsewhere in the world.
(3) I want us to develop these events into scales big enough to accommodate
greater activities just like in the rest of the world.

455 :名無しさん@英語勉強中 :2022/07/12(火) 12:07:52.63 ID:IHYlqyKt0.net
>>453
>>女性だからという理由で選ばれたるのを嫌がったを訳するとき、
クォーテーションのところって存命かどうかに関わらずwasであってますか?
>>she didn't want to be singled out because she "was" a female.

(1) こういうときは、was で正しい。今でも女性なら通常は is になるけど、
過去の話をしているときには、その過去の時点で女性「だった」のだから
was でよいのだ。それについては、wordreference.com にて英語ネイティブが
小説を書いているときに was にすべきか is にすべきか迷っているという
相談を持ち掛け、それに対して別のネイティブが was でよい、と言っていた。

そのスレッドがどこにあるのか検索したくても、うまいキーワードが見つからず、
今の俺には見つからない。なんせ、小説の書き出しにて、「彼女はこういう
女だった。そして Irish だった。彼女はそのとき、〜した、そして〜をした」という
ようなことを書く時、その彼女は今でも Irish ではあるけど、その小説に出てくる
彼女の話をしているときには、それはあくまで過去の話なんだから、She was Irish
でよいのだ、と英語ネイティブは言っていた。

456 :名無しさん@英語勉強中 :2022/07/12(火) 12:12:21.51 ID:IHYlqyKt0.net
>>女性だからという理由で選ばれたるのを嫌がった
>>she didn't want to be singled out because she "was" a female.

基本的には because だけでもいいのだが、just because にして、次のように書いた方が
わかりやすい。

She didn't want to be singled out ●just because● she was a female.

なぜ just をつけた方がわかりやすいかというと、just becaiuse にすれば
「〜だからという理由で〜されるのは嫌だった」という意味がはっきりするのだ。

もし just をつけなかったら、
(1) 女性だからという理由で〜されるのは嫌だった。
(2) 女性だったから、〜するのは嫌だった。(つまり、「私は女なんだから、
そんなことされるの嫌よ」というふうに彼女は思っていた。)

という2つの意味のうち、どちらであるのかがわかりにくくなる。just because にすれば、
読者はたぶん (1) という意味なんだろいうという解釈がしたくなってくるんだと俺は思う。

457 :名無しさん@英語勉強中 :2022/07/12(火) 12:13:00.71 ID:IHYlqyKt0.net
>>456>>453 への回答の続き。

458 :名無しさん@英語勉強中 :2022/07/12(火) 12:17:28.91 ID:xEjpxt6l0.net
>>455
ありがとう

459 :名無しさん@英語勉強中 :2022/07/12(火) 15:35:54.28 ID:kdvSYJf/0.net
「each ~」は単数扱いですが、例えば
Stones are placed at each site.

A stone is placed at each site.
ではどちらが正しいのでしょうか?
しばしば前者の方で書くのですが、でも単数扱いだしなあと迷います。

460 :名無しさん@英語勉強中 (ワッチョイ cb89-2HoA):2022/07/12(火) 16:27:47 ID:IHYlqyKt0.net
>>459
(1) Stones are placed at each site.
(2) A stone is placed at each site.

両方とも正しいよ。厳密に言うと、
(1) は「それぞれのサイトに複数の石を置く」。つまり、1つ目のサイトには
2個。二つ目のサイトには3個、という感じ。

(2) は、それぞれのサイトに1つずつ石を置く。1つのサイトには、1つしか石を置かない。

461 :名無しさん@英語勉強中 :2022/07/12(火) 19:12:22.96 ID:kdvSYJf/0.net
>>460
ありがとうございます。

> それぞれのサイトに1つずつ石を置く。1つのサイトには、1つしか石を置かない。
の意味で前者
Stones are placed at each site.
を使っても問題ないでしょうか。

462 :名無しさん@英語勉強中 :2022/07/12(火) 19:40:48.14 ID:IHYlqyKt0.net
>>461
もし仮にそういう意味にもなりうるとしても、結局は両義性を持ってしまうわけで、
そういうあいまいな言い回しはしない方がいいってことになる。俺ならそういう
変なことはしない。1通りの意味にしかならないような言い回しをなるべく使う。
ただし口語の場合は、適当に言えばいい。日常生活はどうせあいまいなもので、
気楽にしゃべっているときは人はいい加減にあいまいに考えているものだから。

463 :名無しさん@英語勉強中 :2022/07/13(水) 00:16:29.00 ID:iEjVxI+B0.net
イギリスの民謡で良い教材になりそうなものはないですか?
ちなみにグリーンスリーブス以外で、お願いします。

464 :名無しさん@英語勉強中 :2022/07/13(水) 04:49:40.44 ID:/0Bemx7O0.net
イギリスの民謡って、自分で簡単にいくらでも探せるだろう?詳しいガイドブックが
ほしいんなら、俺が持っている次の本を読めばいい。

The New Penguin Book of English Folok Songs
Edited by Steve Julia Bishop
Penguin Classics
2012

151 曲のイギリス民謡を、楽譜付き歌詞付きで詳しく解説してある。
注釈だけでも 140 ページにわたる。

そんな分厚い本を手に取りたくないからもっと簡便な方法を教えろ、というのなら、
俺は断る。どんな馬鹿でも簡単にそんなものは探せる。

465 :名無しさん@英語勉強中 :2022/07/13(水) 05:19:20.44 ID:fC/uhfNY0.net
>>464
https://www.youtube.com/watch?v=nMOyOlmBYCU
Drunken Sailor 日本でいう田植え歌の船乗り版、アイリッシュフォークソング

466 :名無しさん@英語勉強中 :2022/07/13(水) 05:48:28.11 ID:irsO9eoP0.net
>>462
いえ、ポリシーの話ではなく、
> それぞれのサイトに1つずつ石を置く。1つのサイトには、1つしか石を置かない。
場合にも
> Stones are placed at each site.
という文を使うことができるか、というのが質問です。

467 :名無しさん@英語勉強中 :2022/07/13(水) 07:21:14.72 ID:/xXtbQ5F0.net
>>466
その英文のままだと、日本語でも「(複数の)石が、それぞれのサイトに置かれる」
という意味でしかないから、文法的には文章として成立するけど、文脈次第で上記の
両方の意味にとられる余地があるので、受け取る側は誤解しやすいということだろ。

「曖昧な表現でいい状況、文脈」あるいは「その曖昧な表現が意味を持つ状況」を
敢えて意図しているのならOK。

stones を使っても、
Stones are placed, one at each site. なら「一つのサイトに石一つ・・・」になる。

468 :名無しさん@英語勉強中 :2022/07/13(水) 16:34:00.59 ID:gtp847aO0.net
非制限用法の人称ってどこに合わせるんでしたっけ?
Children rarely eat carrots, which harm their health.
というのがあったとしてchildrenだからharm?

469 :名無しさん@英語勉強中 :2022/07/13(水) 17:58:25.01 ID:mMtRTPu60.net
>>468
childrenに合わせる非制限用法ならもちろん三単現sはつけないが
今回はsつくよ
非制限用法だから一元的にここだと指摘はできぬ

470 :名無しさん@英語勉強中 :2022/07/13(水) 18:29:19.96 ID:gtp847aO0.net
>>469
なんで今回はsつくんですかね?
全体に対する非制限用法ってことだとsがつくということ?

471 :名無しさん@英語勉強中 :2022/07/13(水) 19:01:04.72 ID:mMtRTPu60.net
>>470
whichだし
なおcarrotsかもしれないだろ!と言われても答えないからな!

472 :名無しさん@英語勉強中 :2022/07/13(水) 19:13:53.44 ID:mMtRTPu60.net
>>470
そういうことです
whoじゃないとも書くべきだった

473 :名無しさん@英語勉強中 :2022/07/13(水) 19:15:13.14 ID:gtp847aO0.net
全体に対する非制限用法だとかならずsがつくんですか?

474 :名無しさん@英語勉強中 :2022/07/13(水) 19:23:26.93 ID:mMtRTPu60.net
>>473
文全体は不可算名詞扱い
これで回答になってるかな?

475 :名無しさん@英語勉強中 (ワッチョイ 7533-4Uu4):2022/07/13(水) 19:45:24 ID:gtp847aO0.net
了解です

476 :名無しさん@英語勉強中 :2022/07/14(木) 10:52:32.07 ID:qwlQlAXS0.net
We had a lot of fun at the party.

このatはいらないような気がしたんですが必要ですか?

477 :名無しさん@英語勉強中 (ワッチョイW e30b-iNQV):2022/07/14(木) 14:23:52 ID:2ND+VmyG0.net
atをなくしてから、それぞれの単語の品詞と文の構造を考えてみると、、
どうなるのかな????

478 :名無しさん@英語勉強中 :2022/07/14(木) 22:33:09.67 ID:2udMs83j0.net
はい、必要です

479 :名無しさん@英語勉強中 :2022/07/15(金) 11:19:20.83 ID:7GhHZ7UT0.net
頭わる

480 :名無しさん@英語勉強中 :2022/07/15(金) 18:59:18.77 ID:bichbkHR0.net
君に最高の幸せを感じさせることで
君をもっともっと可愛く、世界一可愛くしたい」ってw

And I wanna make you even cuter, the cutest of the world
giving you the sweetest〜〜〜〜

なんかgivingあたりからの構文が違ってる気がするんですが
英訳お願いします!

481 :名無しさん@英語勉強中 :2022/07/15(金) 19:05:55.44 ID:pDmUwPKH0.net
rの発音についてよく舌先をどこにもつけないように奥に曲げろって言われますけど、
tailorみたいなのならできますけど、文頭にrが来た場合realisticとかはどう発音するんですか?
ィアリスティックみたいな感じになって通じますか?

482 :名無しさん@英語勉強中 :2022/07/15(金) 19:27:58.14 ID:t2fB+tRG0.net
君に最高の幸せを感じさせることで
君をもっともっと可愛く、世界一可愛くしたい

(1) I want to make you as happy as can be
to make you cuter, even cuter, the cutest in the whole world.

(2) I want to make you as happy as can be.
In so doing, I want to make you cuter, even cuter, the cutest in the whole world.

(3) By making you as happy as can be,
I want to make you cuter, even cuter, the cutest in the whole world.

(4) I want to make you as happy as can be,
thereby making you cuter, even cuter, the cutest in the whole world.

483 :名無しさん@英語勉強中 :2022/07/15(金) 19:29:56.46 ID:t2fB+tRG0.net
(5) I want to make you as happy as can be,
making you cuter, even cuter, the cutest in the whole world.

484 :名無しさん@英語勉強中 :2022/07/15(金) 19:31:28.81 ID:t2fB+tRG0.net
(6) Making you as happy as can be,
I want to make you cuter, even cuter, the cutest in the whole world.

485 :名無しさん@英語勉強中 :2022/07/15(金) 19:48:14.50 ID:bichbkHR0.net
たくさんありがとうございます!

詩の感じにしたいので少しtherebyが硬くても 四番がいいかなと思うんですが

makeを二回繰り返すのを避けたい場合も、よろしかったら
お願いします 

さっきからしらべてて
同じく硬い言い回しで」by dint ofってあったんですが
これは使えないでしょうかね〜?

あと恥ずかしくて書かなかったんですが
実際は 君の中に最高の喜び(っていうかせんさちおん)を感じさせることで」
っていうのにしたいんですが(ちょいエロ気味w)

inside youと inside of youの使い分けって適当でいいんでしょうか?

後者は inside of you全体で名刺みたいに使うっていう説明も見たんですが。

486 :名無しさん@英語勉強中 :2022/07/15(金) 19:50:03.14 ID:bichbkHR0.net
↑ 変換ミスでした

君の中に最高の喜び(っていうかthe sweetest fierce sensation)

です

487 :名無しさん@英語勉強中 :2022/07/15(金) 21:36:46.19 ID:yB8b2YIV0.net
I wanna take you to heaven so that you'll be
even cuter, cuter than anybody in the world

488 :名無しさん@英語勉強中 :2022/07/16(土) 04:57:01.75 ID:4FbjQgucM.net
Why is Jenny moving to Atlanta?
なぜジェニーはアトランタに引っ越すんですか?
という意味ですが、なぜmovingという形容詞を使った疑問文なのでしょうか?
Why does Jenny move to Atlanta?ではおかしいでしょうか?
2つの疑問文の違いは何でしょうか?

489 :名無しさん@英語勉強中 :2022/07/16(土) 05:04:58.25 ID:BlZylP+a0.net
(4) I want to make you as happy as can be,
thereby making you cuter, even cuter, the cutest in the whole world.

この上の文の make の繰り返しを避けて、しかもエロチックな雰囲気も含めて、
ちょっぴり poetical な感じにしたいのなら、

I want to make you as happy as can be,
With me inside you,
Giving you the greatest sensation,
Thereby turning you into a girl cuter,
Ever more adorable,
By far the most captivating in the whole world.

490 :488 (オイコラミネオ MMad-/SbN):2022/07/16(土) 05:46:36 ID:4FbjQgucM.net
すみません、あともう一つ聞きたいんですが
Why aren't you going on Saturday?
なぜ土曜日に行かないんですか?
という訳ですが、このbe goingは計画や段取り、調整をして行う行動、まずそれをやるよ!と確信している時の未来を表す現在形ですか?(ネットでそれっぽいやつを調べてみました)

あなたは土曜日に行かないは
You don't go on saturday
それを疑問文にしてなぜを文頭に置いたら
Why don't you go on saturday?
これで良いのではないかと思ったりしますがおかしいですか?

491 :名無しさん@英語勉強中 :2022/07/16(土) 08:59:32.07 ID:BlZylP+a0.net
>>488
(1) Why is Jenny moving to Atlanta?
>>なぜジェニーはアトランタに引っ越すんですか?

(1) は正しい。「今から(たとえば明日とか一週間後に)引っ越す予定、
あるいは、いま引っ越しをしている最中」とかいうときには、
このように現在進行形にする。

(2) Why does Jenny move to Atlanta?ではおかしいでしょうか?

おかしい。このような Jenny moves とか Jenny doesn't move とか does Jenny move?
という「単純現在形」は、「ふだん(日常的に、習慣的に)引っ越す(移動する)」
という意味になる。たとえば 
I go to school by bus.
と言えば、「日常的に(習慣的に、ふだん、毎日、あるいは週に1回、あるいは月に1回)
学校にバスで行く」ということだ。

もしも
I am going to school by bus.
となると、「今からバスで登校するよ」とか「今、バスで学校に向かってる最中なんだ」
という意味になる。そして「今、一時的にバスで登校してる(でもそのうち、
また徒歩あるいは車で登校することになるかもしれん)」という意味で使うこともある。

492 :名無しさん@英語勉強中 :2022/07/16(土) 09:02:41.73 ID:BlZylP+a0.net
>>488
>>(2) Why does Jenny move to Atlanta?

このままだったら変だが、たとえば
Why does Jenny move to and come back from Atlanta once a month?
と言えば、正しい英語だ。意味は
「Jenny は、なぜ月に一回、アトランタに往復してるの?」
ということになる。月に一回のこういう習慣を何年も続けているのかもしれない。
こういう習慣を表すときに、こういう単純現在形を使う。

493 :名無しさん@英語勉強中 :2022/07/16(土) 09:09:08.38 ID:BlZylP+a0.net
>>490
>>Why aren't you going on Saturday?
>>なぜ土曜日に行かないんですか?

「今度の土曜に行く予定(行くつもり)」という意味だ。

>>Why don't you go on saturday?
>>これで良いのではないかと思ったり

この英文も場合によっては使える。意味は、
「今度の土曜に行ったらどう?」という勧誘の言い回し。
「今度の土曜に行ったらいいのに、なんで行かないのさ?」
という非難を込めた言い回しにもなる。
(相手の語調や表情でその2つのうちのいずれかがわかる。)

まあいずれにしても、あなたの今回の疑問はすべて、理屈で質問して答えてもらっても、
どうせすぐに忘れる。ともかくあなたは、今までの英文読書があまりに不足している。
今まで読んだ量の少なくとも 100 倍から 1,000 倍くらい読み切った暁には、
こんな質問なんて自分ですべて回答できるようになっている。

494 :名無しさん@英語勉強中 :2022/07/16(土) 09:12:40.94 ID:BlZylP+a0.net
>>490
「たくさん読まないとダメ」と言われて、「でも、意味も分からずにどんどん読んでも
仕方ないのでは?」と答える馬鹿がたくさんいる。

「意味も分からずに読む」のは、読んでいるのではなくて、目を走らせているだけだ。
読むというのは「理解しながら読む」という意味に決まってる。ただ、
90% から 95% くらい理解したところで次へ進めばいいと俺は思う。そして
何百ページか先まで読んだころには、最初の数ページで躓(つまづ)いていたこと
が自然とわかってくることが多い。

495 :名無しさん@英語勉強中 :2022/07/16(土) 15:09:01.24 ID:Dvi41p2J0.net
>>481
「ィアリスティック」、そんな感じで良いと思います。
私は松澤喜好氏の『英語耳』を読んで「発音ができないとリスニングができないんだ」と一念発起し、
半年くらいかけて延々とCDを聞き込みました。どうにか舌の位置は矯正できたと思います。
しかし、それでも例えばCNNのENGLISH EXPRESSのアンカーの発音は聞き取れません。
理由としては構文の認識が追い付かないというのはあります。ただ、知ってるはずの単語が
聞き取れておらず、自分の発音と少しズレがあることに気づきました。EEの中で言及されている
強勢と弱化、連結、脱落、同化とは違うものです。
ここでふと思い出したのが、英字新聞を購読していたときに知った『ザ・ピーナッツバター・
ファルコン』という映画でした。面白い映画なのですが、私の興味を引いたのは主人公が
ダウン症の青年でその発音が独特なんですね。喉からの発音で会話しているんですね。以前、黒人の
少女がかなり舌先に頼った発音をしている動画を見て、聞き取りやすい発音だな≠ニ思って
いたのですが、その青年の発音を聴いて(当然私は聞き取れないけど、)会話が成り立つの?≠ニ
思うほどの物でした。どちらかと言うと、舌の位置によって発音が決まるのではなく、喉の緊張と弛緩に
よって大勢が決まり、しいて言えば舌の位置によって仕上がる・最終的に発音が決まると感じました。
最近、単語帳を1.5~1.7倍速でシャドウイングしているのですが、舌による吐息の量の調整ではその速さに
対応できず喉を開け閉めすることによる発音の後、舌による吐息の調整で倍速に対応できることに
気づきました。
喉の開閉による発音なので腹式呼吸も必要になりましょう。これについては松澤氏も言及しているようですが、
その主張のインパクトも口コミでは『英語耳』ほど強くないらしく、その本は購入していません。
とりあえず一度『ザ・ピーナッツバター・ファルコン』をご視聴になることをおススメします。
おそらく「ィアリスティック」で良いとお感じになると思います。

496 :名無しさん@英語勉強中 :2022/07/16(土) 15:17:59.32 ID:upMg6qCd0.net
>>495
ありがとうございますm(_ _)m

497 :名無しさん@英語勉強中 (ワッチョイ 7684-Ih8j):2022/07/16(土) 16:11:33 ID:Dvi41p2J0.net
>>496
IDが変わらないうちに…。
今は”ELVIS”を見るべきですね!!!
見事な横隔膜=魂の振動を見ることができます!!!!!

498 :名無しさん@英語勉強中 :2022/07/17(日) 14:06:01.83 ID:GUy839Sy0.net
「究極のゼミPart5」の問題について初心者質問です。

Tim Wilson "receives" an award for the project on which he had spent much time and energy.

の "receives" が誤りで、"received" が正解という問題なのですが
時間と労力を費やしたのは「過去」でも
賞を受けるのが「今」なら「receives」では誤りなのでしょうか?

499 :名無しさん@英語勉強中 :2022/07/17(日) 14:08:05.41 ID:FLHdoo9L0.net
which節の大過去見て過去形余裕でした

500 :名無しさん@英語勉強中 :2022/07/17(日) 14:32:46.91 ID:i7aAXWQU0.net
>>498
(1) 原文: Tim Wilson "receives" an award for the project on which he had spent much time and energy.

本当に初心者なら、理屈を言わないで、こういう時には過去形が自然なんだと
覚えるんだよ。それが無理なら、仕方がないから少し解説する。

上の (1) という原文は、中央で二つに分けると次のようになる。

(2) Tim Wilson "receives" an award for the project
(3) on which he had spent much time and energy.

仮に (3) を無視したとしても、「Tim Wilson は、これこれこういう仕事をして
受賞した」という文脈のとき、receives なんていう現在形には「通常は」ならない。

しかし特殊な場合に、現在形の receives になることがある。たとえば新聞の見出しでは、
普通の会話とか通常の文章の中では過去形とか現在完了になりそうなときでも、
単純現在形にする。そういう現在形を見たら、「ああ、これは本当は過去または
現在完了で言いたいことを現在形で言ってるだけなんだな」と理解しないといけない。

今回の場合、現在形はありえない。「受賞した」のだから、単純現在では表せない。

次に、(2) のあとに (3) がくっついている。(3) の中に過去完了(大過去?)が
使われている。そういう過去完了(大過去?)がついているときに、その直前には
現在形なんてありえない。

501 :名無しさん@英語勉強中 :2022/07/17(日) 14:36:28.93 ID:i7aAXWQU0.net
>>498
俺がすでに言ったことの意味がもし仮にわからなかったら、それはあまりにも初心者
だからわからないのだ。だからすでに言ったように、まずは「こういう場合には
過去形になるんだな」と素直に覚える。そのまま小学生とか幼稚園児みたいに
丸暗記することだ。100回から300回くらい繰り返して暗証しながら、それを
ノートに100回かr和300回ほど書く。そうすればどんな馬鹿でも(そして
12歳の時の俺でも)覚えられる。現に俺はそのようにして中学の3年間は過ごした。

疑問を持つなら、その3年間を過ぎたあとにしたらいい。最初は何もかもわからなくて
当然だ。そして仮に質問して、答えてもらっても、その答えの意味がわからない。
だったら諦めて、ともかく暗記することだ。それが嫌なら、英語をやめるしかない。

502 :名無しさん@英語勉強中 :2022/07/17(日) 14:36:30.53 ID:cj7P23Cx0.net
>>498
普通の現在時制には、「今〜している」という意味は無い。繰り返し習慣的に行なっている意味になる
そもそも賞を受けるのが「今」という状況は基本的に無いでしょ
1分前に受賞してもそれは過去だからね
授賞式に出席して今まさに賞状みたいなのを受け取っているならHe is receiving an award になるけどね

503 :名無しさん@英語勉強中 :2022/07/17(日) 14:46:40.84 ID:XqnluSpZ0.net
写真や絵にタイトルをつけるとして
沢山の人がジャンプしている様子、跳ねてる様子を、
そして人々というよりジャンプそのものを指したい場合

また、jumping jumperという単語を使いたくない場合
写真で過去形だからと言ってjumpedを使いたくない場合


動詞jumps(三人称単数現在)
名詞: jumps(複数形)を使って

many jumpsと表現してもおかしくないでしょうか

504 :名無しさん@英語勉強中 :2022/07/17(日) 14:48:37.63 ID:GUy839Sy0.net
>>502
ありがとうございます!
よく分かりました

505 :名無しさん@英語勉強中 :2022/07/17(日) 14:57:01.17 ID:cj7P23Cx0.net
>>503
動詞じゃなくて名詞複数形としてそれでいいと思うけど、いっそjumps だけでもいいかもね

506 :名無しさん@英語勉強中 :2022/07/17(日) 15:17:27.80 ID:XqnluSpZ0.net
>>505
了解です

507 :名無しさん@英語勉強中 :2022/07/18(月) 15:14:55.33 ID:oLRM3ENq0.net
https://i.imgur.com/pxGtF58.jpg

Dが駄目な理由がよく分からないです。

508 :名無しさん@英語勉強中 :2022/07/18(月) 15:40:55.65 ID:NV7mEiyN0.net
AとDでは意味が違います。Aしか正解はないです。

509 :名無しさん@英語勉強中 :2022/07/18(月) 16:23:08.31 ID:NxQPJDRg0.net
breath
いぶき をネット検索で調べてたんだけど
春の息吹
a breath of spring と同じ意味で

A Spring Breathという歌の題名が出てくるのですが
英語圏の人から見て違和感ないのでしょうか
題名ならありな使い方なのか
Aは付けた方がいいのは分かりますが
付けなかった場合、より違和感ありますか?

というのも私自身が文章の中で、ちょっとでも語の量を減らしたくて
〇〇の息吹 を breath of 〇〇 ではなく〇〇 breathと書いてしまいたいだけなんですけど

怪獣が火を噴くとかでもなく、自然系の、
自然の とか、
森の とか、丘の とかです。

510 :名無しさん@英語勉強中 :2022/07/18(月) 20:04:11.93 ID:S56NbC7b0.net
>>507
A.この会社は、~するために対策を講じました
D.この会社は、~するために長さを測りました

511 :名無しさん@英語勉強中 :2022/07/18(月) 21:18:00.49 ID:3X8pBL/Ra.net
>>509
違和感あるかどうかは分からんけど、タイトルとか見出しのスタイルは普通の文章とはだいぶ違うから、文章中に使うなら文章で使われてるのを確認した方がいいと思う。

512 :名無しさん@英語勉強中 (ワッチョイW bd73-3+h2):2022/07/18(月) 23:39:14 ID:/I/JsImq0.net
リスニングのお勧め教材教えて下さい。

最近英語の勉強を始めたんだけど、教材とかはまだ買ってなくて、リスニングが苦手だからなんか教材買ってみようと思って。

自分の英語レベルは、たぶん日本の平均的大学生くらいかな?
目標はTOEIC 800点。受けたことないから自分が何点レベルなのか不明だけど。

513 :名無しさん@英語勉強中 :2022/07/19(火) 00:23:36.00 ID:vjTks9zPM.net
公式問題集

514 :名無しさん@英語勉強中 :2022/07/19(火) 00:25:26.20 ID:55oVZ+rV0.net
TOEICの?

515 :名無しさん@英語勉強中 :2022/07/19(火) 01:26:15.32 ID:dwLi6teya.net
買う前にネットとかアプリとかのリスニング試してみたら?
VOA Learningあたりのゆっくり英語が聞けるなら普通のリスニング教材で良いだろうし、駄目ならかなり初級レベルからやった方がいい。

516 :名無しさん@英語勉強中 :2022/07/19(火) 02:05:39.89 ID:55oVZ+rV0.net
Netflixで英語字幕で海外ドラマ見るのと、EnglishCentralってアプリはやってみた。
どっちも一部は分かるけど、全部は聴き取れない。
VOE Learningは知らないから調べてみるよ。

そして普通のリスニング教材ってなに?そのお勧めを教えてって言ってんだけど。
Thanks.

517 :名無しさん@英語勉強中 :2022/07/19(火) 02:17:30.56 ID:dwLi6teya.net
TOEICリスニングならEnglishUpgraderって公式アプリがあるよ

518 :名無しさん@英語勉強中 :2022/07/19(火) 02:42:06.65 ID:IqGgP9iO0.net
>>511
ありがとうございます
書き忘れたけど文中で使う時というのも
タイトルとか見出しの名前として使う予定です

519 :名無しさん@英語勉強中 :2022/07/19(火) 09:17:38.51 ID:55oVZ+rV0.net
>> 517
ありがとうございます。
ちょっと試してみます。

520 :名無しさん@英語勉強中 :2022/07/19(火) 16:35:43.59 ID:2TradCfo0.net
>>509
"a breath of spring" は、その他の "a breath of [something]" という定型句を使ったものであって、
それを崩さない方がいいと思う。しかし英語ネイティブでも "a breath of air" と言いたいところを、
あえて "a spring breath" と言ってしまうときもあるだろうとは思う。それは詩の中で、
リズムとか韻を踏ませる関係でどうしても "a breath of BBB" (この BBB は名詞) を
"a BBB breath" と書かざるを得ない場合があり得るだろうと思うのだ。しかし
それは英語ネイティブが、その言い回しを奇妙だと百も承知で、rhyming とか
rhythm の関係であえて使っているだけのことだと思う。だから俺たちのように
英語ネイティブよりも百倍くらい英語の下手な人間があんまりそういうものを
真似しない方がいいと思う。

"a breath of spring" なら大いに英語ネイティブらしき人が書いた文章がずらずらと
検索で引っ掛かるが、"a spring breath" となると、確かにいくらかヒットはするけど、
日本語や韓国語がずらずら並ぶ中でときどき "a spring breath" という文字列が
混ざるだけという例があまりに多い。その中でもしも英語ネイティブらしき人が書いた
文章の中で盛んに "a spring breath" という言葉を使いまくっているということが
明らかになれば、そのときに初めて俺たちは安心してそれを使うことができる。
しかし今のところ、そういう証拠は見つからない。

したがって、"a breath of spring" はそのままの形で使った方が無難であり、
"a spring breath" を使うのはなるべくやめた方がいい、と俺は思う。

521 :名無しさん@英語勉強中 :2022/07/19(火) 16:39:23.07 ID:2TradCfo0.net
>>509
さらに、a を省きたいなどと考えるのも、すでに言ったような理由で
やめた方がいいと俺は思う。せっかく "a breath of spring" というふうに
a をつけた用例を使いながらきれいな英文がジャンジャン出てくるのに、
a を省いたり "a spring breath" などという言葉を使ったサイトは日本とか韓国ばかり
なので、避けた方が無難。

ただ、英語ネイティブがリズムなどの関係であえて a を省くのは、彼らが
自分でリスクを負いながら省いているのだから、それを英語ノンネイティブである
俺たちはそのまま受け入れるしかないわな。

522 :名無しさん@英語勉強中 :2022/07/19(火) 17:32:37.19 ID:/Yf7ww+dM.net
お客様などの部屋に入る時の失礼しますは何と言う?
入って良いですか?の意訳はたくさんあるけどお伺いしてYESを貰った後に言う言葉が探しても見つからない

523 :名無しさん@英語勉強中 :2022/07/19(火) 19:45:15.25 ID:fOqSBUx30.net
普通にHellow とかgood morninとかじゃねの

524 :名無しさん@英語勉強中 :2022/07/19(火) 19:48:00.91 ID:YqfMrbfP0.net
I'm coming!(ガチャ

525 :名無しさん@英語勉強中 :2022/07/19(火) 19:49:54.96 ID:fOqSBUx30.net
>>512
リトルチャロ

526 :名無しさん@英語勉強中 :2022/07/19(火) 19:52:13.87 ID:fOqSBUx30.net
>>522
sorry to disturbing you

527 :名無しさん@英語勉強中 :2022/07/19(火) 19:57:11.53 ID:JWoFoUKQ0.net
>>522
既に入っていいですか?と言ってるんだから、イエス貰った後は何も言う必要はない
黙って入って用事済ませて黙って出て行け

528 :Seeker2022 :2022/07/19(火) 21:14:08.59 ID:Mwh9HtWA0.net
ネイティブの発想
「部屋に入る許可をすでに得ているんだから、なんでそれが「失礼」になるわけ?」

529 :名無しさん@英語勉強中 :2022/07/19(火) 21:49:18.48 ID:tPt29xFA0.net
>>524
それイッちゃうぅぅぅぅだろ

530 :名無しさん@英語勉強中 :2022/07/19(火) 22:07:13.32 ID:fOqSBUx30.net
may I cum inside you?

531 :名無しさん@英語勉強中 :2022/07/19(火) 22:32:49.05 ID:muQLcqbIM.net
お邪魔します の英語は?

532 :名無しさん@英語勉強中 :2022/07/19(火) 23:02:48.66 ID:fOqSBUx30.net
>>531
I''ll fuck you up

533 :Seeker2022 :2022/07/20(水) 00:12:05.00 ID:LVx9REIi0.net
>>531
ネイティブの発想 「邪魔しに来たのなら、帰ってくれ」

534 :名無しさん@英語勉強中 :2022/07/20(水) 00:51:00.64 ID:iv75mFgta.net
>>531
I'm disturbing you.

535 :名無しさん@英語勉強中 :2022/07/20(水) 00:54:26.18 ID:nXf0vPmE0.net
Could you give me a OMOTENASHI?

536 :名無しさん@英語勉強中 :2022/07/20(水) 04:51:59.03 ID:kcNpAi0z0.net
Thanks for having me.

537 :名無しさん@英語勉強中 :2022/07/20(水) 05:50:10.59 ID:LI7GRJDo0.net
[Greetings by a former French aristocrat]
Thank you very much indeed for your exquisite hospitality
by having me inside your esteemed, impossibly captivating physique
through all of your heavenly delicious openings.

538 :名無しさん@英語勉強中 :2022/07/20(水) 09:32:55.61 ID:yCSURApK0.net
conjunct
コンジャンクトというカタカナ読みらしいが
ケンジャンクトが正しい?

539 :名無しさん@英語勉強中 :2022/07/20(水) 10:00:42.22 ID:8d/GDl+pa.net
>>538
品詞で変わる

Pronunciation
IPA: /ˈkɑn.dʒʌŋkt/ (noun)
IPA: /kənˈdʒʌŋkt/ (adjective)

540 :名無しさん@英語勉強中 :2022/07/20(水) 10:22:32.93 ID:OcMfMJvQ0.net
Ex-Japan Prime Minister Shinzo Abe's funeral sees crowds in Tokyo streets - BBC News

情けないですがこういうseeの用法があるのを初めて知りました

seeってこの場合「見送る」ではなく 謁見する、」対面する的意味でしょうか?

541 :名無しさん@英語勉強中 :2022/07/20(水) 10:50:58.94 ID:8d/GDl+pa.net
>>540
無生物主語+seeで"起きる"の意味があります。
なので、"葬儀で人混みが出来る"みたいな訳になると思います。

542 :名無しさん@英語勉強中 :2022/07/20(水) 11:07:48.00 ID:yCSURApK0.net
>>539
ありがとうございます

543 :名無しさん@英語勉強中 :2022/07/20(水) 11:11:12.79 ID:LI7GRJDo0.net
>>540
日本人の書いた文法書でそれについて詳しく解説したものと言えば、
江川泰一郎「英文法解説」

544 :名無しさん@英語勉強中 :2022/07/20(水) 12:47:08.09 ID:OcMfMJvQ0.net
写真の紹介コメントで

これは太郎くん

横にいるのは彼の友人の あきらくんです」っていう場合

his good friends Akira

なんでしょうか?

あきらくんは一人だから his good friend Akiraでもいい気がするんですが。

545 :名無しさん@英語勉強中 :2022/07/20(水) 14:44:04.96 ID:LI7GRJDo0.net
>>544
>>これは太郎くん 横にいるのは彼の友人の あきらくんです

写真に写っている人たちを順番に指さしながら口頭で説明するなら

This is Taro. And this is his friend Akira.

指をさしながらではなくて、写真の下に説明書き(キャプション)を添えるのであれば、
太郎が右にいて、その左にアキラがいるなら、
On the right is Taro. On his left is his good friend Akira.

546 :名無しさん@英語勉強中 :2022/07/20(水) 14:47:49.53 ID:LI7GRJDo0.net
>>544
あるいは、左から順番に紹介する説明書きだったら、

From left: Taro and his good friend Akira.

547 :名無しさん@英語勉強中 :2022/07/20(水) 19:05:59.63 ID:OcMfMJvQ0.net
ありがとうございます!
ですよね
普通単数だと思います

念のためと思ってDeepLにかけたら
friendsになったので
気になってしまいましたw

ありがとうございます

548 :名無しさん@英語勉強中 :2022/07/21(木) 05:16:04.73 ID:8tzQXgWH0.net
>>547
いくら何でも DeepL がそんなヘマをするかな、と思って俺もやってみたら、
次のように正しく英訳してるよ。

これは太郎くん 横にいるのは彼の友人の あきらくんです
DeepL 英訳: This is Taro. Beside him is his friend, Akira.

549 :名無しさん@英語勉強中 :2022/07/21(木) 05:20:31.62 ID:8tzQXgWH0.net
>>544 の his good friends Akira は、どうやら
He's good friends with Akira.
という正しい英語表現を(間違って)縮めてしまって
書いているような感じだな。

550 :名無しさん@英語勉強中 :2022/07/21(木) 19:26:33.11 ID:wWpv2vggd.net
お願いします
よくある文ですが

Let me know if you need any help.

このknowとifですが

①knowは他動詞でif以下は名詞節→助けが必要かどうかを私に知らせてください(直訳)
②knowは自動詞でif以下は副詞節→もし助けが必要ならば私に知らせてください(直訳)

①と②、どちらと解釈すべきでしょうか?

551 :名無しさん@英語勉強中 :2022/07/21(木) 19:34:47.41 ID:wWpv2vggd.net
Let me know when you will arrive in any Tokyo.
Please let us know when you are arriving.

のような関節疑問文を見ると普通に①なんでしょうかね

552 :名無しさん@英語勉強中 :2022/07/21(木) 19:36:55.17 ID:wWpv2vggd.net
>>551
any TokyoではなくてTokyoでした
すみません

553 :名無しさん@英語勉強中 :2022/07/21(木) 20:07:17.63 ID:VacMHCxK0.net
take out a book は「本を取り出す」ですが、「本を鞄から取り出す」の場合
take out a book out of my bag とはならず take a book out of my bag と
なるのはなぜなんでしょうか?

554 :名無しさん@英語勉強中 :2022/07/21(木) 22:29:29.86 ID:43lfvVvG0.net
>>550
それはどっちにもなりうるので文脈から判断するしかない

555 :名無しさん@英語勉強中 :2022/07/21(木) 22:30:05.44 ID:43lfvVvG0.net
>>553
別にout2回言っても問題は無い

556 :名無しさん@英語勉強中 :2022/07/22(金) 01:59:53.76 ID:F2UHD3b50.net
インスタなんかの画像にコメつける時

この画像は〜〜の時のですよね!ってコメント書きたいとき


This image is

That Imageか>なんですが。どうでしょう

557 :名無しさん@英語勉強中 :2022/07/22(金) 07:50:20.61 ID:XF6Z0BOI0.net
>>556
最初のコメならThis~で、話の中で出てきたらThat~にすると良いよ。
でもimageよりpicとかphotoのほうが誤解がないと思う。

558 :名無しさん@英語勉強中 (ワッチョイ 7689-qysg):2022/07/22(金) 10:08:51 ID:4oBhIhjU0.net
>>551
wordreference.com という英語質問サイトでの
Let me know when you will be/you are available
というタイトルの質疑応答を見ればわかるように、

(1) Let me know when you will arrive in Tokyo.
(2) Let me know when you arrive in Tokyo.

この二つの意味はまるで違う。
(1) 東京に着く時刻を教えてください。
(2) 東京に着いたら(着いたときに)知らせてください。

559 :名無しさん@英語勉強中 (ワッチョイ 7689-qysg):2022/07/22(金) 10:11:23 ID:4oBhIhjU0.net
>>553
(1) take out a book out of my bag
(2) take a book out of my bag

(1) のように out を繰り返さなくても、(2) だけで言いたいことは十二分に伝わっているのだから、
まともな人は (1) なんていう英文を口にしないし、書きもしない。

560 :名無しさん@英語勉強中 (ワッチョイ 7689-qysg):2022/07/22(金) 10:15:23 ID:4oBhIhjU0.net
>>553
ただし
Take out a book--out of my bag.
(-- は、正確には -- を dash つまり長い横棒にする。)
と書いて、しゃべっているときも -- の部分には0.5 秒くらい間を置けば、
このように out を繰り返しても正しく、しかも正確な英文になるだろうな。

まずは Take out a book. と言っておき、それで終わろうと思ったけど、
考え直してその out の正確な意味を言い添えるために
out of my bag をあとに添えるのだ。

しかし、あいだにまったく間を置かずに、dash もつけずに
Take out a book out of my bag.
なんて言ったり書いたりする奴はいないだろうな。馬鹿でない限り。

561 :名無しさん@英語勉強中 :2022/07/22(金) 11:32:07.15 ID:Or9b81/v0.net
(1) (V)Let (O)me (C){know when you will arrive in Tokyo}. when は疑問副詞, when~ は know の目的語
(2) 主節{(V)Let (O)me (C){know}} 従節{when you arrive in Tokyo}. when は従属接続詞

(1) の know は他動詞なので「(あなたが到着する時間)“~を知る”」
(2) の know は自動詞なので「知る」、従節→主節でならべて、あなたが来る時に→知る , "時に"が従節詞っぽい訳し方かな

みたいな感じになるのか。(2)は疑問副詞では無いので当然のことながら間接疑問文でもない、みたいな。
勉強になるなぁ。

562 :名無しさん@英語勉強中 :2022/07/22(金) 11:42:47.68 ID:Or9b81/v0.net
I don't know if it will rain tomorrow.「私は、明日雨が降るかどうか知りません。」

なるほど、副詞節の中では will 等の法助動詞は使えないから if 〜 は名詞節で確定なのか。
that, if, whether の 3 つは名詞節(主語・補語・目的語)を導く従位接続詞

従属接続詞なのに名詞節を導くのね。うーん、全然覚えられてない・・・、

563 :名無しさん@英語勉強中 :2022/07/22(金) 11:55:05.12 ID:iaDY9In50.net
トンチンカンな解釈だな

564 :名無しさん@英語勉強中 :2022/07/22(金) 12:27:07.60 ID:Or9b81/v0.net
>>563
英語を知らない人に言われても・・・、
英語を知っているなら、あなたの言う解釈をちゃんと書き記そうね?
あと、日本国内限定の英語もパスね。

565 :名無しさん@英語勉強中 :2022/07/22(金) 12:30:49.15 ID:p9j9YXf50.net
>>558
横レスだがarrive atを脳死で暗記してたから時々出てくるarrive inが怖くて怖くて
国とかでかいからでいいんかなinにする理由

566 :名無しさん@英語勉強中 :2022/07/22(金) 13:12:23.81 ID:4oBhIhjU0.net
>>565
そういうときには、まずは検索だよ。

"arrive in" "arrive at"

これをこのまま Google 先生の間抜けな口にぶち込んでやれば、
いろんなサイトを吐き出してくれるから、いろいろと解説を読めばいいんだよ。

567 :名無しさん@英語勉強中 :2022/07/22(金) 16:32:12.60 ID:pEt8107id.net
>>558
ご紹介ありがとうございます
そのパートを読むことができました
以下自分なりのまとめ

568 :名無しさん@英語勉強中 :2022/07/22(金) 16:32:30.48 ID:pEt8107id.net
(1) when以下の述語動詞が現在時制とwillを伴う場合の意味は異なります
(2) whenが時の前置詞に置き換えられる場合は相対時間が明確なので法助動詞であるwillは使うべきではありません
(3) またwhenの他の用法として関係副詞が考えられます(the time when)
※引用者(私)注
関係副詞と考えられるということは関節疑問文とも考えられる場合がほとんど

以上より
(4) Let me know when you will be available. は
利用可能になる特定の時間時刻を教えてください
(5) Let me know when you are available. については面白いことに二通りの解釈が可能です
(6) Let me know when you are available. = When you are available, let me know. (時の従属副詞節)
(7) Let me know when you are available. = Let me know the time when (= at which) you are (scheduled to be) available.(the time省略の関係副詞節=名詞節=関節疑問文)

569 :名無しさん@英語勉強中 :2022/07/22(金) 16:34:36.39 ID:pEt8107id.net
以上より>>550に対しては
>>554氏のように考えるべき

570 :名無しさん@英語勉強中 :2022/07/22(金) 16:35:02.60 ID:pEt8107id.net
>>558
面白いサイトを教えてくださりどうもありがとうございました

571 :名無しさん@英語勉強中 :2022/07/22(金) 16:36:19.71 ID:pEt8107id.net
当該パートのURL

https://forum.wordreference.com/threads/let-me-know-when-you-will-be-you-are-available.3031698/

572 :名無しさん@英語勉強中 :2022/07/22(金) 16:40:21.46 ID:MIm/oDiS0.net
>>562
その辺は大学受験の初歩レベルだよ
疑問にするところではない感じ
わかりやすい文法というのかな

573 :名無しさん@英語勉強中 :2022/07/22(金) 16:54:42.96 ID:pEt8107id.net
>>561
勉強になりますよね
面白いし
お互いに頑張りましょう

>>562
名詞節を導く従位接続詞はそれだけですから今回覚えてしまえば無問題

574 :名無しさん@英語勉強中 :2022/07/22(金) 17:04:40.69 ID:MbuwilVyH.net


575 :名無しさん@英語勉強中 :2022/07/22(金) 17:04:53.96 ID:MbuwilVyH.net


576 :名無しさん@英語勉強中 :2022/07/22(金) 17:05:12.33 ID:XhdY1F7HH.net


577 :名無しさん@英語勉強中 :2022/07/22(金) 17:40:35.37 ID:6eNAglCX0.net
>>550
>助けが必要かどうかを私に知らせてください
>もし助けが必要ならば私に知らせてください

この2つの違いがわからんは

578 :名無しさん@英語勉強中 :2022/07/22(金) 18:33:12.99 ID:PNFxePVya.net
「着いたぞ。ここが例の家だ」
「へぇ〜、こんなところにあるんだ」←これどう訳せば良いですか?
微妙な驚きを含んだ、あんまり意味がなくて相槌とつぶやきの間のような言葉です。
直訳で "it exists at such place" だと意味が通じなそうだし、
かといって"i didn't think it exists at such place" というほど意外に思ってるわけではないです。

579 :Seeker2022 :2022/07/22(金) 18:36:02.37 ID:y2svOAYe0.net
>>577
違いは、

助けが必要かどうかを私に知らせてください←助けが必要でなくても知らせてください

もし助けが必要ならば私に知らせてください←助けが不必要なら知らせなくてもいいです

580 :Seeker2022 :2022/07/22(金) 18:45:38.23 ID:y2svOAYe0.net
>>578
自分なら、こう言います。
Wow! I didn't expect to find it in a place like this.

581 :名無しさん@英語勉強中 :2022/07/22(金) 20:17:36.71 ID:iaDY9In50.net
>>564
お馬鹿さんにはトンチンカンさ加減がわからないんだろうな

582 :名無しさん@英語勉強中 :2022/07/22(金) 20:19:24.44 ID:iaDY9In50.net
>>573
こんな回答しているようじゃ知れてるな

583 :名無しさん@英語勉強中 :2022/07/22(金) 20:21:57.78 ID:af30qSI50.net
thereとかthere areをザァって言っても大丈夫?
苦手なんですよ。

584 :名無しさん@英語勉強中 :2022/07/22(金) 20:23:38.58 ID:iaDY9In50.net
>>583
カタカナで言われてもなあ

585 :名無しさん@英語勉強中 :2022/07/22(金) 20:25:35.67 ID:iaDY9In50.net
>>564
それじゃ一生英語ができるようにはならないよw

586 :名無しさん@英語勉強中 :2022/07/22(金) 20:34:29.42 ID:bfYtiBY4d.net
なかなかのキチガイっぷりw

587 :名無しさん@英語勉強中 :2022/07/23(土) 04:36:34.04 ID:hY2KALVO0.net
>>578
着いたぞ。ここが例の家だ
Here we are. This is the house.
(this を強く発音)

へぇ〜、こんなところにあるんだ
Hmm, so this is where it is, huh?
(ここでも、this を強く発音)

588 :名無しさん@英語勉強中 :2022/07/23(土) 05:01:31.02 ID:BCC/h3LC0.net


589 :名無しさん@英語勉強中 :2022/07/23(土) 05:34:15.13 ID:xnw4DsaA0.net


590 :名無しさん@英語勉強中 :2022/07/23(土) 07:33:41.26 ID:/yd3xIDzM.net
i

591 :名無しさん@英語勉強中 (ワッチョイ 499d-IdmX):2022/07/23(土) 07:54:52 ID:O0o+/3Ys0.net
>>581,582,585
日本語すら理解できていないことを
そんなに何度も自己申告されなくてもちゃんと皆さんも理解されてますよ。
ひとつご忠告しておきますが、その類まれな粘着力はここではなくて別の方面で発揮されたほうが良いのでは?

592 :名無しさん@英語勉強中 :2022/07/23(土) 08:40:12.69 ID:XC458cVq0.net


593 :名無しさん@英語勉強中 :2022/07/23(土) 08:58:01.58 ID:s6620v1b0.net


594 :名無しさん@英語勉強中 :2022/07/23(土) 09:45:58.61 ID:O0o+/3Ys0.net
>>583
there の発音は ðɛr です。カタカナ発音であればどう変えようと英語耳の人には同じに聞こえるので、その辺で違いを気にしても意味が無いと思います。
英語は発音記号通りに発音することで相手が聞き取れるというところから皆さんも理解しましょう。もちろん私もなんだけど。

"ð" は、日本語の「ザ・ジ・ズ・ゼ・ゾ」の音とは違い、舌先を上の前歯の裏に触れさせてから発音する音です。[θ]の音で声帯(と舌先)を震わせる声です。
https://eigonokai.jp/phonetics/36-%E6%9C%89%E5%A3%B0%E5%AD%90%E9%9F%B3%EF%BC%9Ad%E3%81%AE%E7%99%BA%E9%9F%B3/

595 :名無しさん@英語勉強中 :2022/07/23(土) 11:56:46.01 ID:bc1aLna0H.net


596 :名無しさん@英語勉強中 :2022/07/23(土) 11:57:03.17 ID:O0mz8DlgH.net


597 :名無しさん@英語勉強中 :2022/07/23(土) 11:57:29.52 ID:0y1ocf3BH.net


598 :名無しさん@英語勉強中 :2022/07/23(土) 11:57:48.60 ID:0OjdZMW6H.net
rt

599 :名無しさん@英語勉強中 :2022/07/23(土) 11:58:02.66 ID:R8HhxQTT0.net


600 :名無しさん@英語勉強中 :2022/07/23(土) 18:33:08.92 ID:Ykn8fUGk0.net
>>583
there, their, they're これらは全て同じ音でカタカナ表記するなら「デア」が近い。
dare と違うのは舌の位置とわずかな振動音だけ。
https://www.youtube.com/watch?v=VxFJpXVk1fs

them と dem なんかは同じ音
https://mayonez.jp/topic/1088766

601 :名無しさん@英語勉強中 :2022/07/23(土) 19:01:14.05 ID:Ykn8fUGk0.net
>>578
I didn't know it's located here

602 :名無しさん@英語勉強中 :2022/07/23(土) 19:09:23.35 ID:6Mq2y9Wg0.net


603 :名無しさん@英語勉強中 :2022/07/23(土) 19:13:00.73 ID:Ue1rd1Tc0.net
dj

604 :名無しさん@英語勉強中 (ワッチョイW d3e9-wcSV):2022/07/23(土) 19:26:12 ID:/IW/dM3S0.net


605 :名無しさん@英語勉強中 (ワッチョイW 499d-dfGR):2022/07/23(土) 19:33:24 ID:Vw8h2BdM0.net
what a fuckin house, shit!

606 :名無しさん@英語勉強中 :2022/07/23(土) 20:27:59.60 ID:arWn8dL40.net


607 :名無しさん@英語勉強中 :2022/07/24(日) 09:05:21.28 ID:pt7SegQ40.net
>>578
So this is where it is, huh?
を検索してその前後の文脈を読み取れば、今回の状況にはこの英語表現がかなり
ピッタリだと感じられると思うけど、この this を that に変えて
So that's where it is huh?
という表現もよく使われる。小説にも出てくるので、ネット上の小説の中に
出てくるこの表現の前後関係を読み取ってみてほしい。

608 :名無しさん@英語勉強中 :2022/07/24(日) 09:09:58.86 ID:pt7SegQ40.net
So this is where it is, huh?
So that's where it is, huh?

これ以外にも、so や huh が消えた形である

This is where it is, huh?
This is where it is.
So this is where it is.

That's where it is, huh?
That's where it is.
That's where it is, huh?

のような表現、そして that's ではなくて that is の形で出てくることもある。

So that is where it is, huh?
So that is where it is.
That is where it is, huh?

609 :名無しさん@英語勉強中 :2022/07/24(日) 20:42:00.50 ID:e2UPu+Bp0.net


610 :名無しさん@英語勉強中 :2022/07/24(日) 20:43:50.81 ID:HXaF2kxV0.net


611 :名無しさん@英語勉強中 :2022/07/26(火) 20:46:10.67 ID:7gXHEwbA0.net
アメリカ人の若者が書いてた言葉ですが
Me knowing I can probably hip thrust your man!
これそういう意味?

彼はジムに通っててジムのマシンに ヒップスラストっていう
ヒップアップのマシンがあるんです。
その画像とともにあったコメントなんですが。

612 :名無しさん@英語勉強中 :2022/07/26(火) 22:33:00.53 ID:8OrNdo+nd.net
This is the first time (that) I have visited London.
= This is my first time visiting London.

613 :名無しさん@英語勉強中 :2022/07/27(水) 05:28:18.29 ID:9iGfLj6Z0.net
>>611
>>Me knowing I can probably hip thrust your man!

「(俺は思いっきり体を鍛えてるから)ヒップスラストで
君のボーイフレンドを持ち上げられるだろうよ。」

だいたいこういう意味だと思うんだが。

614 :名無しさん@英語勉強中 (ワッチョイ fb91-SXL5):2022/07/27(水) 06:41:23 ID:y2fSNolL0.net
英単語を学習したいのですが、良いワードファミリーはありますか?
どのくらいの単語数を覚えればいいのか分かりません。

目的はredditなどの掲示板をスラスラ読めるようになることです。

target 1900は手元にあります。
ngslはやったのですが、少し簡単すぎるような気がするのでレベルアップしたいです。

無料で何かあれば、、、

615 :名無しさん@英語勉強中 :2022/07/27(水) 07:11:01.08 ID:U1RJh0hn0.net
https://simple.wikipedia.org/wiki/Wikipedia:VOA_Special_English_Word_Book

616 :名無しさん@英語勉強中 :2022/07/27(水) 12:34:49.31 ID:Q8lg87Bn0.net
プログレッシブ英和中辞典「基本単語レベル」1819
https://dictionary.goo.ne.jp/ej/level/%E5%9F%BA%E6%9C%AC%E5%8D%98%E8%AA%9E/
プログレッシブ英和中辞典「大学入試レベル」4666
https://dictionary.goo.ne.jp/ej/level/%E5%A4%A7%E5%AD%A6%E5%85%A5%E8%A9%A6/
プログレッシブ英和中辞典「社会人必須レベル」8285
https://dictionary.goo.ne.jp/ej/level/%E7%A4%BE%E4%BC%9A%E4%BA%BA%E5%BF%85%E9%A0%88/

617 :名無しさん@英語勉強中 :2022/07/27(水) 12:37:31.27 ID:Q8lg87Bn0.net
https://dictionary.goo.ne.jp/ej/
プログレッシブ英和中辞典のレベル情報

618 :名無しさん@英語勉強中 :2022/07/27(水) 18:28:44.91 ID:hmvh1OKRa.net
主語が単数の場合と複数の場合を合わせて書くにはどう書けばいいでしょうか?動詞を単複どちらで書くとかルール的なものありますか?

The item (s)  と書き出した場合、is なのかare なのか、、、

619 :名無しさん@英語勉強中 :2022/07/27(水) 18:39:23.85 ID:y2fSNolL0.net
>>616
ありがとうございます
社会人必須レベルというのは、ネイティブのですよね

大学生と社会人でそれほど違うものですか?

掲示板読む知恵度ならどれになるでしょうか

620 :名無しさん@英語勉強中 :2022/07/27(水) 19:02:22.19 ID:MmbC0D5E0.net
▶They couldn’t care less about the law.
奴らは法律などお構いなしだ
なんですが、どうしてそうなるんでしょう

621 :名無しさん@英語勉強中 (ワッチョイ d3c7-R4TS):2022/07/27(水) 21:08:03 ID:iL4w+rOU0.net
couldn't ... less でこれ以上、気にしない ⇒ お構いなし

I can't thank you enough.と同じ使い方です。

622 :名無しさん@英語勉強中 :2022/07/27(水) 21:48:59.86 ID:HA2JFGqx0.net


623 :名無しさん@英語勉強中 :2022/07/27(水) 23:05:03.61 ID:GW0YsdZ20.net
That so-called “bad paper” discharge is issued for misconduct,
and means the recipient is ineligible for certain veterans’
benefits and cannot reenlist.

“bad paper”とはどういう意味でしょうか?

624 :名無しさん@英語勉強中 (ワッチョイW 53e6-GNiG):2022/07/28(木) 05:25:30 ID:lG9Wh5Vi0.net
>>621

1 (I) could [OR còuldn’t] càre léss.
どうでもいいよ;関係ないね(◆ could でも couldn’t でも同じ意味になるが,could を用いる場合には強勢を置かずに発音する.強勢を置くと「少しは気になるが」という別の意味になる)

と書いてあるのですが、その、couldでもcouldn’tでも同じ意味になる、というのはどういう心の働き
でそうなるんだろう、という意味での疑問を持って尋ねたんです。

625 :名無しさん@英語勉強中 :2022/07/28(木) 06:45:51.29 ID:rpavkjUv0.net
回答ではなく
日本語だけど、嘘つけ と 嘘つくな が
同じ意味になるのはなぜなんだろう

626 :名無しさん@英語勉強中 :2022/07/28(木) 07:29:24.12 ID:1ijMiIz+0.net
You gotta be joking.
Stop kidding me.

627 :名無しさん@英語勉強中 :2022/07/28(木) 08:06:22.00 ID:hDbe4/5u0.net
>>624
>>625
そのへんは、会話してる人同士に共通認識があるっていうのが前提だろうね。

could の強勢も、嘘をつけつくな、も、
こちらがどう思っているのかのニュアンスを相手に伝えているのだと思う。
ニュアンスで伝えているから、言葉自体の意味の比重が低くなってるとかそんな感じ。

こういうのは、実際に体験してみて始めて納得の出来る物なんだけど、
知識としてそういうことがあるというのが予め分かってれば納得しやすいと思うな。

628 :名無しさん@英語勉強中 :2022/07/28(木) 08:24:32.76 ID:gaz/tkQ80.net
単にまちがった言い方が許されてるだけだよ
そういうのいっぱいある

629 :名無しさん@英語勉強中 :2022/07/28(木) 10:05:38.55 ID:JtI+i2W00.net
>>623
質問者が提示している一節と、その直前に出てくる1文とを一緒にここに引用してみる。

Belanger was part of the Marine Corps from 2019 until May 2021,
when the Marine Corps discharged him with ★an Other Than Honorable Discharge★.
That so-called ●“bad paper” discharge● is issued for misconduct, and means the recipient
is ineligible for certain veterans’ benefits and cannot reenlist.

このように並べて、俺が印をつけたところに注目すれば、質問者の疑問は氷解するだろ?
要は、
a bad paper
= an Other Than Honorable Discharge
ということだよ。俺の下手な和訳をつけると、
「不名誉な除隊命令書」というような名称のついた書類が
bad paper (まずい、よくない紙) と呼ばれてるわけだ。

こんなのは、例えば日本で「赤紙」ってものがあっただろ?
それは決して本当に「赤紙」という専門用語で呼ばれていたわけじゃなくて、
本当はたぶん「召集令状」と呼ばれ、そのようにその書類には書いてあったんだろうと
思う。それを一般に日本人は「赤紙」って言ってただけのことだ。

同じように、この Rolling Stones 誌に出てくる bad paper は、本当に bad paper という
専門用語で呼ばれていたのではなくて、あくまで
an Other than Honorable Discharage
と呼ばれ、そのようにその書類に表記してあったんだろうよ。

これくらいのことは、きちんとその Rolling Stones の記事の前後を読めばわかるだろうに。

630 :名無しさん@英語勉強中 :2022/07/28(木) 13:06:49.17 ID:lG9Wh5Vi0.net
>>625,627
ありがとう

631 :名無しさん@英語勉強中 :2022/07/28(木) 13:07:06.92 ID:lG9Wh5Vi0.net
>>628
ありがとう

632 :名無しさん@英語勉強中 :2022/07/28(木) 13:16:57.18 ID:lG9Wh5Vi0.net
イギリスの食事が美味しくないのは産業革命で主婦まで働きに出てしまって
食事を家で作らなくなって食文化が失われたからというよね。
イギリスも昔は食べ物が美味しかったとか。
イギリスの方がドイツよりも寒いんじゃない?

633 :名無しさん@英語勉強中 :2022/07/28(木) 13:18:16.00 ID:lG9Wh5Vi0.net
>>632は誤爆です

634 :名無しさん@英語勉強中 :2022/07/28(木) 13:49:42.92 ID:O9DiEWlZ0.net
care lessの場合
could ~
仮定法で、気にすることができる→実際は気にしない
couldn't ~
否定+比較級=最上級の肯定→全く気にしない

635 :名無しさん@英語勉強中 :2022/07/28(木) 14:39:21.90 ID:Q0W8MR+P0.net
今より更にケアしないこともできるで = 既に興味ないけどもっと興味無くなっても問題ないんやで

今より更にケアしないなんてできんわ = 全然興味ねぇ、これ以上興味無くすなんてムリポ

636 :名無しさん@英語勉強中 (ワッチョイ 2908-BDMY):2022/07/28(木) 15:10:03 ID:gd6w9L2L0.net
tip (someone) over the edgeって
だいたいが 限界に追い込むとかネガティブな例文だらけなんですが

一個でもポジティブな例文ないでしょうか?
たぶんとても文学的な表現になると思うんです

たとえば maddeningって気が狂うほどっていう意味ですが
maddening pleasureっていう表現もありますよね
そんな感じで
お願いします!

637 :名無しさん@英語勉強中 (ワッチョイW 53e6-GNiG):2022/07/28(木) 15:28:03 ID:lG9Wh5Vi0.net
>>634
ありがとう。助かります

638 :名無しさん@英語勉強中 (ワッチョイW 53e6-GNiG):2022/07/28(木) 15:39:37 ID:lG9Wh5Vi0.net
>>636

Don’t trip over the edge of the sidewalk.
歩道の縁に躓く、、、

639 :名無しさん@英語勉強中 (ワッチョイ 8b89-R4TS):2022/07/28(木) 19:32:08 ID:JtI+i2W00.net
>>636
tips me over the edge
という string of words を検索してみると、エロ小説の一節の中で、凄まじいほどの
性的興奮のゆえに「私は苦しいくらいに興奮する」みたいな意味合いで使われている
一節がすぐに見つかったよ。

Andrea Martin · 2021 · &#8206;Fiction
The sound of his passion almost ★tips me over the edge★.
I wrap my fingers around the base of his cock, enjoying how wide and thick he feels, ...

相手の男の cock を刺激してあげると、相手は快感のあまり呻くんだけど、
そのうめき声を聞いてると、「私は苦しいくらいに興奮する」という
ような意味で上のような文章を書いている。この小説は
Playing Away というタイトルの小説らしい。

いずれにしても、もともと negative な意味合いで使われている表現を、
相手の魅力の凄まじさによって自分が苦しいくらいに興奮したり
気が狂いそうなほどに相手がきれいだとかいうふうな表現をしている
例をよく見る。

640 :名無しさん@英語勉強中 (ワッチョイ 8b89-R4TS):2022/07/28(木) 19:36:58 ID:JtI+i2W00.net
>>636
こんなもん、いくらでも見つかるよ。またもや見つかった。

(1) The show her husband is giving sucking my cock ★tips her over the edge★
and she climaxes, too bad I don't get it on camera. In the mean time Jordan is sucking ...

(2) During the massage Michelle tries to stay professional
but Will's huge hard cock ★tips her over the edge★. Before long her hard shaft glides inside his ...

641 :名無しさん@英語勉強中 (ワッチョイ 8b89-R4TS):2022/07/28(木) 19:38:12 ID:JtI+i2W00.net
>>636
(3) She moans, squirms, and writhes
until just the sight of you stroking your hard cock ★tips her over the edge★ into a HUGE orgasm!
☆This was a custom video!

642 :名無しさん@英語勉強中 (ワッチョイ 8b89-R4TS):2022/07/28(木) 19:41:32 ID:JtI+i2W00.net
>>636
(4) Stephanie's shriek of satisfaction as my tongue ★tips her over the edge of orgasm★
harmonises with Joshua's grunted affirmation. He hurriedly pulls out, ...

(5) Another deep chuckle before he returns his mouth to me and,
with a few expert strokes of his tongue, ★tips me over the edge★.
Heat floods through my body as ...

643 :名無しさん@英語勉強中 :2022/07/28(木) 19:45:14.36 ID:JtI+i2W00.net
>>636
(6) A sudden compression of my dick ★tipped me over the edge★, strong enough
for me to lift her ass away from my cock. She hastily replaced her ...

(7) The phantom sound of Kurt gagging on my dick ★tipped me over the edge★.
Hips jerking, I spilt hot come across my stomach and moaned Kurt's ...

まあともかく、エロ小説みたいな文章からだったら、いくらでも簡単に
続々とこういう例文が見つかる。

644 :名無しさん@英語勉強中 :2022/07/28(木) 19:58:08.17 ID:JtI+i2W00.net
>>636
今度は、エロチックな文章じゃなくて、普通の美しさのようなものによって
"tip someone over the edge" という表現が使われている例文を探してみる。

(1) This is such a beautiful display and you have ★tipped me over the edge★.

上の (1) は、きれいな展示物を見せてくれたから「すごく感動したよ」みたいな
意味合いで使っているように思える。

(2) Sue MacKay · 2014 · &#8206;Fiction
Sarah's world exploded into a trillion beautiful fragments as Dan ★tipped her over the edge★
into that place where there was no beginning and no end, ...

この上の (2) は、またもや小説の一節だけど、エロ小説ではなくかなり真面目な小説の
中で、恋人同士が愛し合っていて、そのおかげで Dan tipped her over the edge と書いている。
あんまりスケベな言葉を使わないで、きれいな文章が続く中でこういう表現も出てくる。

645 :名無しさん@英語勉強中 :2022/07/29(金) 12:40:15.49 ID:warZR/A00NIKU.net
>>636
tip someone over the edge だけじゃなく、もともと negative な意味であったはずの
言葉が大いに positive な意味で使われることも多い。

insanely beautiful, insanely sexy, insanely handsome, insanely good

maddeningly good, maddeningly beautiful, maddeningly sexy, maddeningly handsome

unbearably handsome, unbearably beautiful, unbearably sexy

intolerably sexy, intolerably beautiful

こういうふうに見ていくと、そもそもこういう副詞(そして形容詞)は、
negative にも positive にも強引に使っているうちに、それが流行っていくもんだろう
という気がする。

646 :名無しさん@英語勉強中 :2022/07/29(金) 12:44:07.01 ID:warZR/A00NIKU.net
>>636
devastating という形容詞にしても devastatingly という副詞にしても、やはり
もともとは negative にしか使えなかったんだろうけど、みんながそれを
positive な意味に使っているうちに定着してしまったようだ。

devastating beauty, devastatingly beautiful, devastatingly handsome, devastatingly sexy, etc.

647 :名無しさん@英語勉強中 :2022/07/29(金) 16:22:51.59 ID:k2z2QN1X0NIKU.net
What am I doing that is rude?
これって疑問文の倒置部分がamってことですかね?
肯定文だと、
What I am doing that is rude.
となるはずですよね?
これを疑問文にすると、
Is what I am doing that rude?
と疑問詞が先頭に来ないのでダメですね。
これを防ぐために冒頭の語順になっていると解釈すれば良いでしょうか?
どなたかご教授を。

648 :名無しさん@英語勉強中 :2022/07/29(金) 16:57:32.29 ID:lySqbYJn0NIKU.net
whenって「その時はこう思ってた」って使い方出来る?

649 :名無しさん@英語勉強中 :2022/07/29(金) 17:43:58.72 ID:warZR/A00NIKU.net
>>647
(1) 原文: What am I doing that is rude?

質問の趣旨がよくわからんが、ともかく上の原文 (1) は、

(2) What that is rude am I doing?

と言い換えてもいい。しかし普通はそんな言い方をせず、that is rude が語尾に追いやられる。
that は関係代名詞であり、その先行詞が what だ。

(3) What am I doing? (私は、何をやってるんでしょうか?)

(4) What that is rude am I doing?
  (私は、失礼に当たるようなどういうことをやっているんでしょうか?)
   (私って、何か失礼なことをやってますでしょうか?もしやってるなら、
   教えてくだしゃんせ。私にはわからんもんで。)

そしてもちろん (4) のような言い方は、普通はしないで、次のように言う。

(5) What am I doing that is rude?

650 :名無しさん@英語勉強中 :2022/07/29(金) 17:44:49.38 ID:warZR/A00NIKU.net
>>648
質問の意味がわからないよ。

651 :名無しさん@英語勉強中 :2022/07/29(金) 17:50:16.70 ID:warZR/A00NIKU.net
>>647
よく似た例を探したければ、たとえば
What did I do that
という string of words を使って検索してみて、たとえば新聞に出てくる用例を
見てみるといい。

★What did I do that★ helped me along my road to achieve success and a good life?

この上の用例は、確かスポーツ新聞に出てくる用例だったと思うが、
that helped me... の that はもちろん、そのずっと前の what という先行詞にかかる
関係代名詞だ。こういう用例は腐るほど出てくるから、探してみるといいよ。

652 :名無しさん@英語勉強中 :2022/07/29(金) 18:01:06.27 ID:warZR/A00NIKU.net
>>647
先行詞の
what と関係代名詞の that とが大きく離れている他の例。すべてネットから
引き出した。

(1) What can I do that will really make an impact?

(2) What do you advise me to do that is not listed on any form by your company?

(3) What does the book say that lungs do in our bodies?

(4) What is he saying that is new?

653 :名無しさん@英語勉強中 :2022/07/29(金) 18:26:00.01 ID:k2z2QN1X0NIKU.net
ID:warZR/A00NIKUさん
ありがとうございます!!
理解出来ました。thatを漠然と指示代名詞と理解してました。
関係代名詞のthatなら離れていても形容している感じがしますね。
この元ネタはラダーシリーズの『English Made Funny 何がヘン?この英語』の
P.84からのものです。これでレベル3です。語彙数は少なくても文法に手こずるので、
嫌になっちゃいます。

654 :名無しさん@英語勉強中 :2022/07/29(金) 21:54:18.23 ID:VPTrLUH/0NIKU.net
すみません質問させて下さい

英語で「TITTI」というワードは卑猥な意味ですか?
今度展開するブランドの名前にしようと思っていたのですが、卑猥に聞こえるからやめた方がいいかもしれないと言われました。
tittyやtittiesは胸だと思うのですが、tittiも胸ですか?
そもそもこの単語は存在しない単語なのでセーフだと思っていたのですが....

655 :名無しさん@英語勉強中 :2022/07/29(金) 22:33:46.65 ID:ZBWAYoCJaNIKU.net
昔、ネットで話題になった日本食レストランがあってだな、
おそらく本来はFukimiという店名だったと思われるが、一部文字が脱落して、Fuk miと表記されていて
Japanese viagra alternativeなんて言われてた

彼らにとっては多少の綴りの違いなんかどうでもいいんだよ

656 :名無しさん@英語勉強中 :2022/07/29(金) 22:42:02.75 ID:wIBhr2V90NIKU.net
>>643>>645

ありがとうございます。
そう!まさに私もエロの場面で使おうと思ってたんです
brains outみたいな限界超えたS**みたいなかんじに
でも
エロくない真面目な文にもあるんですね!
コーパスでも米国googleやset phraseのサイトなんかでも調べたけど
調べ方が下手なんで見つけられなかったんです

657 :名無しさん@英語勉強中 :2022/07/29(金) 22:51:16.06 ID:wIBhr2V90NIKU.net
>>644

1のほうは mind-blowingみたいな使い方ですよね
でも2のほうは
露骨なポルノチックな描写を一切使わずに
そういう暗喩された性的な絶頂を示してるようにも見えます
まさにわたしが求めてた使い方でした
ありがとうございます

658 :名無しさん@英語勉強中 :2022/07/29(金) 22:59:50.13 ID:wIBhr2V90NIKU.net
1はover the edgeで一旦終了してますが
2は、edgeからさらにinto 〜〜っていけるんですね。ちょっと高度な文学チックなのかな
ロマンスノベルっぽいやつかも。だから下品にしすぎないで
でも官能を暗示させてるんですね

やっぱコーパスでお調べなんですか?
もしよかったらどのコーパスかおおしえいただけないでしゅか?w

659 :名無しさん@英語勉強中 :2022/07/30(土) 04:18:42.18 ID:28eysuYT0.net
質問です
tittyとtittiは英語では同じ音ですか?

660 :名無しさん@英語勉強中 :2022/07/30(土) 09:00:03.98 ID:Lo/nZn/10.net
>>658
別に特別な corpus なんて使ってなくて、ただの Google だよ。
The Corpus of Contemporary American English (COCA) なんていうコーパスを
昔は使ってたけど、大したことはなくて、Google の延長みたいなもんでしかない
と最近は感じている。だから今は、通常の Google ばかりを使ってる。

どういう連語 (string of words) を Google の間抜けな口に放り込むかによって、
自分の必要とする例文を探し出す工夫を何十年も繰り返していくしかないと思う。
 

661 :名無しさん@英語勉強中 :2022/07/30(土) 09:03:35.39 ID:Lo/nZn/10.net
>>654
TITTI なんて書いてあったら、英語ネイティブなら誰でも titty, titties, tits を連想する
だろうよ。ろくに英語のできない俺でもそれを連想する。
だから最初からそういうのは避ければいいんだよ。

662 :名無しさん@英語勉強中 :2022/07/30(土) 09:12:16.04 ID:Lo/nZn/10.net
>>656
調べ方が実にうまい人もいるだろうけど、俺なんかは不器用で時間のかかる効率の
悪い方法で、通常の Google を使って検索しただけだよ。

(1) tip someone over the edge
こういう連語 (string of words) の出てくる例文をあなたは探してたんだろ?だから俺は、
まずはスケベな言葉を添えて、まずは
(2) cock tips me over the edge
(3) pussy tips me over the edge
(4) kiss tips me over the edge
(5) tits tip me over the edge

こういう変な言葉を主語に入れた例文がないかどうか探した。次に、
tip(s) という現在形ではなくて過去形の tipped にしたり、me の代わりに
you, her, him などを目的語にしてみたりもした。

さらに、
"tipped her over the edge" site:google.books
とかいうふうな言葉を Google の口に入れて検索したりして、
google.books サイトにある小説の一節を探したりもした。あるいは
site:news
と指定して、新聞記事にそういうものがないかと探すこともある。

まあ、そういう不器用で(誰でも知っている)方法しか俺は使っていないよ。
あとは、何十・何百種類もの例文を斜め読みしていき、自分の好みの例文を
素早く見つける工夫をするしかないわな。そのためにも、英文を斜め読みしても
なるべく早く必要な情報をつかみ取る訓練はいつもしておかないといけないわな。

663 :名無しさん@英語勉強中 :2022/07/30(土) 11:21:04.94 ID:JwTGLrfO0.net
初歩的な質問で申し訳ないのですが聞きたいことがあります。

Whether it be chemical or radioactive, biological or nuclear, Echo-8 is always ready to deploy and fight through the most biohazardous situations known to man.
エコー-8は化学物質か放射性物質かを問わず、核燃料物質や生物などの人類が知る限り、最も危険の状態にある場合でも常時に展開して戦い抜く準備が出来ています。

この部分で「chemical or radioactive」「biological or nuclear,」は並列表現であると言われたのですが、
ちょっとピンと来ません。どういうことなのか教えて頂けると助かります。

664 :名無しさん@英語勉強中 :2022/07/30(土) 12:01:30.78 ID:hkV2oohU0.net
alexaやsiriに聞くときはdo you knowですか?

665 :名無しさん@英語勉強中 :2022/07/30(土) 13:36:35.19 ID:Lo/nZn/10.net
>>663
「並列表現」という日本語や、それに対応する英語表現である
parallel structure (parallelism) という言葉を検索すると、その解説が出てくるから
参考にしてくれ。

もしもそこで出てくる解説を読んでもわからなかったら、単純に
「2つ以上の似たような言葉を並べること」というような意味だと解釈しておけばいいと思う。

666 :名無しさん@英語勉強中 :2022/07/30(土) 15:41:07.73 ID:JwTGLrfO0.net
>>665
なるほど、ありがとうございます!

667 :名無しさん@英語勉強中 :2022/07/30(土) 15:54:05.05 ID:7ZmyXC680.net
貴方にはもっと大会に出て欲しいです
って英語でなんていうんですか?

668 :名無しさん@英語勉強中 :2022/07/30(土) 16:49:36.74 ID:gieUphwT0.net
>>661
ありがとうございます
でもtittiという単語自体は存在しないんですよね、、?
これって日本語で言うと
「おっぱ」「ぱいぱい」みたいな感じですか?
ニュアンスがわからないです。

669 :名無しさん@英語勉強中 :2022/07/30(土) 16:56:07.07 ID:Lo/nZn/10.net
>>667
>>貴方にはもっと大会に出て欲しいです

もしも「大会」を tournaments と訳していいんなら、

I wish you would join many more tournaments.

670 :名無しさん@英語勉強中 :2022/07/30(土) 18:23:33.88 ID:w5U5/GMyM.net
>>668
tittyのyをiに変えただけで、発音同じだから
「おっぱイ」じゃないかな

671 :名無しさん@英語勉強中 :2022/07/30(土) 18:55:50.40 ID:lw+aZmz1d.net
This is the first time (that) I have visited London.
≒ This is my first time visiting London.
≒ This is my first visit to[in] London.

ロンドンを訪れるのはこれが初めてだ

この文はウィズダム英和のtimeの項からの抜き書きです
一番上の文の(that)はネットで検索した似たような例文にあったもので、私が補ってみました

この文の構造はどういうものでしょうか?
thatが省略されているとしてそのthatは関係副詞なのか同格のthatなのか
ちなみに他の複数の英和辞書ではthe first time等でtimeは接続詞的に用いられるとの説明が見られます
よろしくお願いします

672 :名無しさん@英語勉強中 :2022/07/30(土) 19:53:44.77 ID:28eysuYT0.net
>>670
それだ

673 :名無しさん@英語勉強中 :2022/07/30(土) 20:06:07.74 ID:DD0ClIRp0.net
同格は普通に可能性として無いやろ
普通に関係副詞

674 :名無しさん@英語勉強中 :2022/07/30(土) 20:06:21.78 ID:DD0ClIRp0.net
あーーやっと書き込めたわ

675 :名無しさん@英語勉強中 :2022/07/30(土) 20:20:05.44 ID:gieUphwT0.net
>>670
ありがとうございます
ということは、例えばTitti Clothingという服屋があったり、TITTIというスノボ商品があるのですが
これは英語圏の人が見たら驚愕したりするのでしょうか

676 :名無しさん@英語勉強中 :2022/07/30(土) 20:32:51.60 ID:IDe7w+X30.net
riding on the boobs

677 :名無しさん@英語勉強中 :2022/07/30(土) 20:53:33.70 ID:w5U5/GMyM.net
>>675
日本にも沖縄に漫湖公園なんてものがあるから、ちゃんと由緒あるものならば偶然同じ発音なのは仕方ないってことでしょう
問題は偶然ではなくわざとの場合で、これから新しく作るものの名前の場合はわざとであると思われがちだということ
fcukが叩かれたのもわざとだと思われたから

678 :名無しさん@英語勉強中 :2022/07/30(土) 21:37:38.89 ID:gieUphwT0.net
>>677
なるほど、ちなみにticchiやticchiであればセーフなのでしょうか?

679 :😉三年英太郎🌈 :2022/07/31(日) 01:54:13.21 ID:9MEJll0e0.net
tittiが良いと思ったなら、それでええやん

680 :名無しさん@英語勉強中 :2022/07/31(日) 03:37:26.09 ID:wJ8vzMSq0.net
>>679
いや、対外的にセーフかアウトかが知りたくて
本場のニュアンス認知ができる方がいればなと思った次第です

681 :名無しさん@英語勉強中 :2022/07/31(日) 03:54:41.62 ID:Tv3UxFPw0.net
>>680
聖光学院って、セーフかアウトどっち?
日本語学習している外国人に聞いてみるといいかもしれない

短い答えは、分かる人はいない

682 :名無しさん@英語勉強中 (ワッチョイW 9ae6-LRMP):2022/07/31(日) 07:29:15 ID:WGyw8MYg0.net
>>671の答えを僕も知りたい。
可能性としてないやろ、と答えだけ提示されてもという思い

683 :名無しさん@英語勉強中 :2022/07/31(日) 07:57:20.55 ID:rdj+cAPg0.net
My first time of visiting London.

684 :名無しさん@英語勉強中 :2022/07/31(日) 08:57:50.43 ID:dtZzNnX30.net
>>671
>>This is the first time (that) I have visited London.

俺からの回答:
that は関係副詞。

もしも that が同格なら、
the first time that I have visited London
なら
(*)The first time is that I have visited London.
などという変な英文が成り立たないといけなくなる。そんな変な英文はあり得ないから、
この the first time that I have... の that は同格の that ではない。

the fact that I love her
の場合、that は同格を示す。なぜなら、
The fact is that I love her.
という英文が成り立ち、それを縮めたような形として the fact that I love her
という英文が出来上がっているからだ(という説明の仕方をしてもいいと思う)。

the hope (news, rumor) that he passed the exam
というときも、やはり
The hope (あるいは my hope) is that he passed the exam.
The news (あるいは The news going around in the town) is that he passed the exam.
The rumor (あるいは The rumor going around) is that S V.
という言い回しがちゃんと成り立ち、それが縮まった形が
the rumor (news, hope) that S V が出来上がったんだという説明の仕方をしても、
(専門家による厳密な文法解説からはもしかしたら少し外れているかもしれんが)
あながち大きな間違いでもないと思うし、そのように一応は覚えておいて、
これからこれに似た英文の解釈のときに活用していけば、さほど大きな間違いをすることはないと思う。

685 :名無しさん@英語勉強中 :2022/07/31(日) 09:02:29.92 ID:PVFEQ67+0.net
This is the first time (that) I have visited London.
これは次のようにも書き換え可能だと思われます。
I have visited London for the first time.
これだと関係副詞と捉えた方が自然ですね。
ただ、元に文にforがないので、すなわち、
This is for the first time (that) I have visited London.
とはなっていないので自分でも少し疑問ですが、forが省略されている可能性もあるかと。

686 :名無しさん@英語勉強中 :2022/07/31(日) 09:04:18.42 ID:dtZzNnX30.net
>>671
the first time that SV の代わりに、the first time when SV と書いている例も、あることはあるな。
この下に列挙した例文は、すべて Google.books から拾った。だからすべて書籍の中に
出て来たのだ。いくらでも見つかるので、興味のある人は自分で検索してみてくれ。

(1) It was not the first time when such a split took place in the Congress.

(2) When was the first time when you were living in Dallas that you saw that Lee owned a rifle ?

(3) I saw it the first time when I got a call around midnight that one of our tenants was destroying the room , a

687 :名無しさん@英語勉強中 :2022/07/31(日) 09:08:13.66 ID:WGyw8MYg0.net
>>685
This is the first time for which I have visited London.
でfor whichがwhenに変わったというだけで、>>685さんの考えでいいのでは?

688 :名無しさん@英語勉強中 :2022/07/31(日) 09:11:43.36 ID:dtZzNnX30.net
for which が when に変わる例なんてない。
at which, in which などが when に変わることはある。

689 :名無しさん@英語勉強中 :2022/07/31(日) 09:11:49.86 ID:WGyw8MYg0.net
名詞と同格といえば,たとえば The fact that I am younger than you makes no difference at all.(私があなたより若いということはまったく関係ないことだ)というような文で,that 以下の名詞節は前の the fact という名詞と同格で,その内容を説明しているというようなときに用いる。
to不定詞が前の名詞と同格になるのは,たとえば次のような場合である。
My daughter had just announced her intention to be a nurse.
(娘が,看護師になるつもりであることを打ち明けたところだった)
前の名詞を説明しているなら形容詞用法ではないかとも言われるが,前に書いたようにこういうことにあまりこだわる必要はない

と黄色ロイヤルは、her intention to be a nurse について書いている。
今の場合もそうなのでは?

690 :名無しさん@英語勉強中 :2022/07/31(日) 09:18:24.38 ID:WGyw8MYg0.net
>>688

J2. Application for patent. Include applications for patents made for the first time for which no determination has been made on patent rights.

というのが”the first time for which”でググったらでてきた。オーストラリアのサイトだった。だから、
for whichはwhenにはならないと書いているけど、問題ないよ

691 :名無しさん@英語勉強中 :2022/07/31(日) 09:18:41.91 ID:dtZzNnX30.net
>>This is for the first time (that) I have visited London.

厳密に言うと、これも変だ。
This is the first time (that) I have visited London.
の This は This time のことであり、厳密な(くどい)言い方をすれば
This time is the first time (that) I have visited London.
という意味だ。だからこそ、the first time の前に for なんてものがついた試しがないのだ。

for the first time は
(*)I met her the first time.
なんていう言い方ができないからこそ the first time の前に for をつける。
ここでの for は「第1回目として(第1回目のために)」というときの「〜として」
に近い意味だろう。

だからと言って
(*)This is for the first time that I've visited London.
っていう言い回しも「これは、〜した第一回目としてのものだ」
と訳せるからこの考え方で正しいのだ、などというのも変だ。
それは日本語でならどんなこじつけの翻訳でも作れる。英文を考えるとき、
日本語訳がどうなるかなんてことを考えてはいけない。

あまりにくだらない意見が飛び交うので、ついつい俺もこんなくだらないことを書いて
しまったが、こんなもんはどうでもいいのだけどな。

692 :名無しさん@英語勉強中 :2022/07/31(日) 09:21:45.56 ID:dtZzNnX30.net
(1) >>My daughter had just announced her intention to be a nurse.

この to be a nurse は
Her intention is to be a nurse.
と言えるからこそ、(1) の her intention と to be a nurse とが同格と呼ばれる。

693 :名無しさん@英語勉強中 :2022/07/31(日) 09:23:48.35 ID:dtZzNnX30.net
>>J2. Application for patent. Include applications for patents made
for the first time ★for which★ no determination has been made on patent rights.

ここでの for which をそのまま when に変えても、この文は成り立つのだ、という
ことは言える。(そんなことは、当たり前だ。)

しかし、だからと言ってここでの
the first time for which S V
teh first time when S V
この二つが同じ意味だ(すなわち for which を when に変えても同じ意味になる)
なんてことは言えない。

694 :名無しさん@英語勉強中 :2022/07/31(日) 09:24:48.12 ID:dtZzNnX30.net
もうやめるね。こういうただの日本語での理屈だけでごり押しの議論がまかり通るから、
俺は文法談義が大嫌いなんだ。

以降、俺はいっさい黙る。

695 :名無しさん@英語勉強中 :2022/07/31(日) 09:31:30.06 ID:PVFEQ67+0.net
>>691
確かに少し変に感じてますよ。
This is for the first time
で検索してもヒットしませんし、私自身この表現に出会ったことが記憶にないですもん。
もちろん「初めて」をイデオムで”for the first time”と覚えてるだけですし。

696 :名無しさん@英語勉強中 :2022/07/31(日) 11:44:17.01 ID:u3u9vH7na.net
This is the first time.
これが初めてです
This is for the first time.
これは初めて(の方)/初心者向けです

こんくらいの違いでしょ
This is for the first timeはto不定詞でこそないけど、ある種の循環構文的構造
つまり*This is this for the first time.とも解釈できるんじゃない?

697 :名無しさん@英語勉強中 :2022/07/31(日) 12:27:43.52 ID:pP/+TW1/0.net
>>662
本当にご親切にありがとうございます!

698 :名無しさん@英語勉強中 :2022/07/31(日) 12:46:23.16 ID:pP/+TW1/0.net
もう一度。
あなたのお優しい書き込みで今後の
英語ライフがかなり変わると思います。
最近頭の働きが悪くてw 気にしてたのですが
本当に役に立つ情報ありがとうございます。

699 :名無しさん@英語勉強中 :2022/07/31(日) 14:39:10.67 ID:+3Cpi12lr.net
習った事を思い出しましょうではなくて本当にゼロイチから英語と言うのは動詞がありましてと教えてくれるのはどこだろう?

700 :名無しさん@英語勉強中 :2022/07/31(日) 14:58:42.32 ID:6WmzOMKid.net
>>684
どうもありがとうございました
とても勉強になります
私も関係副詞の省略なんだと思え、また納得もできました

他の皆様にもありがとうございました

701 :名無しさん@英語勉強中 :2022/07/31(日) 15:03:47.40 ID:LxhxQ8KSd.net
>>699
動詞とか名詞は日本語でもあるから、そこからとなるとあんまり無いかもしれんけど、高校向け総合英語とかはかなり基礎から書いてないか?

702 :名無しさん@英語勉強中 :2022/07/31(日) 15:06:20.94 ID:6WmzOMKid.net
また、that=whenのwhenの正体についてですが、
以下私見ですがfor whichではないような気がします

関係副詞のwhenは前置詞付きのwhichで書き換えられる場合はやはりin whichかまたはat whichかon whichであり、
先行詞が前置詞のつかない時の名詞(いわゆる副詞的目的格、副詞的対格)の場合も多々あるわけで、
その場合は前置詞+whichへの書き換えにはそもそも馴染まないものなのでは、と思います
素人なので間違った見解でしたらすみません

703 :名無しさん@英語勉強中 :2022/07/31(日) 15:30:30.87 ID:0137TUJl0.net
イングランドの民謡を教えて下さい。

704 :名無しさん@英語勉強中 :2022/07/31(日) 15:53:13.85 ID:XRRUPZya0.net
>>690
この文ってどういう意味なん?
なんか意味自体がよくわかんないんだけど

705 :名無しさん@英語勉強中 :2022/07/31(日) 15:57:34.64 ID:WGyw8MYg0.net
始めて特許を出すときには特許の応用例もつけろ

706 :名無しさん@英語勉強中 :2022/07/31(日) 16:16:29.51 ID:dtZzNnX30.net
>>703
>>464 にて俺が紹介した次の本を読みながら、YouTube で歌声を検索すればいいだろ?

The New Penguin Book of English Folk Songs
Edited by Steve Julia Bishop
Penguin Classics
2012

151 曲のイギリス民謡を、楽譜付き歌詞付きで詳しく解説してある。
注釈だけでも 140 ページにわたる。

707 :名無しさん@英語勉強中 :2022/07/31(日) 16:24:52.32 ID:dtZzNnX30.net
>>690
>>J2. Application for patent. Include applications for patents made
for the first time ★for which★ no determination has been made on patent rights.

この英文に出てくる "for which 節" は、その直前の the first time にかかっているのではない。
その "for which 節" は実は、そのずっと前の patents (あるいはもしかしたら applications)
にかかっているはずだ。すなわち、次のような構造だ。

J2. Application for patent.
Include applications for patents
(1) [which have been] made for the first time
(2) for which no determination has been made on patent rights.

この上の (1) という節と (2) という節との両方が同時に、
その前の patents (あるいはもしかしたら applications) にかかっている。

だいたいからして、こういう特許関係の文章は昔から読みにくい。
それもそのはずで、それは法律関係の文章であり、それは 400 年あるいはそれ以上も前の
ラテン語やフランス語で書かれた法律関係の文書の書き方をそのまま現代まで踏襲して
いる場合が多く、現代の文章を読むような気持で読んでいると、とんでもない誤読をする。

708 :名無しさん@英語勉強中 :2022/07/31(日) 16:43:01.78 ID:dtZzNnX30.net
>>707 のように俺は言ったが、本当にそうなのかどうか証明して見せようか?
>>690 が引用した一節はオーストラリアのサイトにて J2 という項目番号のもとに
書かれた文章だ。その直前にある J1 という番号のもとに、次のような似た一節が出てくる。
特に俺が●とか★という印をつけた部分に注目。

J1. Patent
This refers to ●patents● on specific products or processes granted ★for the first time★, either in
Australia or in an overseas country. It does not include multiple rights for the same porduct or process,
or ◆applications for which no determinatoin has been made on patents rights◆.

上の一節の少しあとで、>>690 の挙げた一節である次の文章が出てくる。

J2. Application for patent.
Include applications for patents made for the first time
★for which no determination has been made on patent rights★.

ほらな。J2 に出てくる "for which no determination,,." という部分と
その前の J1 に出てくる
"applications for which no determination has been made on patent rights"
とが呼応しているだろ?

読者はそのように読み進めているわけだから、読者は普通は
"for which no determination has been made on patent rights" という J2 の一節が
その直前の the first time にかかるのではなくて、そのずっと前の applications に
かかっているのだと理解するわけだ。

結論として、(くどいようだが一言で言えば)
>>690 が挙げた文に出てくる for which はその直前の
the first time にかかっているのではなく、あくまでそのずっと前の applications にかかっている。

709 :名無しさん@英語勉強中 :2022/07/31(日) 17:27:59.56 ID:FXAMqIiI0.net
↑、なんか偉そうに。。。と俺は思う

710 :名無しさん@英語勉強中 :2022/07/31(日) 17:30:32.72 ID:4qsUWy890.net
もっと控えめな提案のように書かないと不愉快だってさ
馬鹿なんじゃねーのこういうやつ

711 :名無しさん@英語勉強中 :2022/07/31(日) 18:52:20.20 ID:Weh1tuMqM.net
法律用語ってのは偉そうに聞こえるものだ

712 :名無しさん@英語勉強中 (ワッチョイW 4a8a-2ZMl):2022/07/31(日) 20:04:40 ID:XRRUPZya0.net
>>707, >>708
なるほどねー、乙です
特許うんぬんに馴染みが無さすぎるのが主な要因だと思うんですが、構造わかってもその上でも意味(訳)がよくわかんないんですけど、どういう訳になるんですかねこれは?
あと、J2の方の文、何で主語も無く、includesみたいに三単現のSもついてないんですかね??わかりますか?

713 :名無しさん@英語勉強中 :2022/07/31(日) 21:42:20.98 ID:NgZIop4B0.net
>>712

>>705書いたのにちゃんと読む人いなくて

714 :名無しさん@英語勉強中 :2022/07/31(日) 22:36:58.94 ID:XRRUPZya0.net
>>713
普通に全然ちゃうやろと思って飛ばしていたw
for which 以下はどうなりますか?

715 :名無しさん@英語勉強中 :2022/07/31(日) 23:28:17.84 ID:NgZIop4B0.net
自分で考えて。
the first time for which でしょ。

716 :名無しさん@英語勉強中 :2022/07/31(日) 23:42:05.68 ID:XRRUPZya0.net
やっぱ全然違うっぽいな…

717 :名無しさん@英語勉強中 :2022/08/01(月) 00:26:12.22 ID:eMjfmHH80.net
make an application (for 〜) 出願する、申請する

applications (for 〜) made for the first time 初めて申請された出願
for which no determination has been made on patent rights 特許権についてなんの決定もなされていない

初めて申請されたもので特許権についてなんの決定もなされていない出願

718 :名無しさん@英語勉強中 (ワッチョイ 7a5b-0eUa):2022/08/01(月) 03:26:20 ID:VN2CdJWU0.net
質問です
自分の馬鹿さをひけらかしてこれ以上俺たちを悲しませないでくれ
の意を英語で言いたい場合、以下で論理的に合ってますか?

Please stop making us sad by boasting about your stupidity.

それとも別の意味(次の意味)になってしまいますか?
俺たちを悲しませるのを、自分の馬鹿さをひけらかすことによって止めてくれ

719 :名無しさん@英語勉強中 :2022/08/01(月) 04:40:22.54 ID:682K83/2M.net
>>718
合ってる
please stopでマジでもうやめてくれとか本当に勘弁してくれ的な「頼むからやめろ」って感じになって
stop単独が憤りの感情に近くなるのに対して、pleaseを加えるとより真剣に行為その物を止めて欲しいシリアスさが混じるから、論理的にも原文のニュアンス的にもより正しい
もし別の意味の方を言いたい場合は、Please make us not feeling sad by とかになるかな?

720 :名無しさん@英語勉強中 :2022/08/01(月) 04:56:05.56 ID:kLUo2uQN0.net
>>718
(1) 自分の馬鹿さをひけらかしてこれ以上俺たちを悲しませないでくれ
(2) Please stop making us sad by boasting about your stupidity.

(2) の英文は、文法的に正しくて、さほど不自然でもないし、綴りも正しい。
論理的に変なわけでもない。しかし、(1) の原文と (2) の訳文とが一致していない。
どこが一致していないかは、自分で考えてほしい。日本文にある何かが
英訳には欠如している。

721 :名無しさん@英語勉強中 :2022/08/01(月) 05:02:29.42 ID:VN2CdJWU0.net
これ以上 が英文には足りないということですか?

Please stop making us sad any more by boasting about your stupidity.

これなら
自分の馬鹿さをひけらかしてこれ以上俺たちを悲しませないでくれ
と同じ意味になりますか?

聞きたかったポイントはそこではなくて、
by 以下が stop にかかる解釈もあり得るのでは、ということが聞きたかったのです

722 :名無しさん@英語勉強中 :2022/08/01(月) 05:05:31.15 ID:kLUo2uQN0.net
他人の回答にケチはつけたくないのだが、自信満々に回答しておられるので、
仕方なくダメ押しをさせてもらう。そうしないと、質問者たちが混乱する。

>>Please make us not feeling sad by....
というのは、一応は間違った英文。(あるいは、少なくとも不自然な英文だ。)
ただし、間違っている(あるいは不自然である)とは言っても、読んだ人は
書いた人の言いたいことはわかってはくれる。それから、英語ネイティブでも
これくらいの変な英文を書く人はいるとは思うけど。それを英語ネイティブの
小学校教師はダメ押しをするだろうな。

さらに、質問者の回答に対して「合ってる」などとよくも言えるよな、と思ってしまう。
「7割くらいは合ってる」と言うべきだろう。単に「合ってる」と言ってしまうと、
「それで満点だ」とか「ほぼ正解」ということになってしまう。

回答者は責任を持って回答してほしい。いくら5チャンネルという低レベルの
場所だとか無料サイトだとは言っても、無責任ではいけない。誰でもすぐに簡単に
日本文と英訳とが完全には一致していないことがわかるのだから。

723 :名無しさん@英語勉強中 :2022/08/01(月) 05:14:00.84 ID:kLUo2uQN0.net
>>721
(1) これ以上 が英文には足りないということですか?

確かにその通り。

(2) Please stop making us sad any more by boasting about your stupidity.

これは間違いだと思う。"making us sad any more" じゃなくて、
making us any sadder
making us even sadder
upset us any more
upset us even further
などとすればいい。

(3) by 以下が stop にかかる解釈もあり得るのでは、ということが聞きたかったのです

そういう解釈もありうる。英文も、日本文と同じく、(そしてどんな言語においても
同じだろうと思うが)2通り以上の解釈ができるときがある。それでも、通常は
前後の関係とか常識的に考えて、そのうちの1つの解釈を読者は選ぶことになる。
ただし、いくら考えてもやはり2通りににしか考えられない場合は、悪文という
ことになるから、1通りにしか解釈できないような英文に書き直した方がいいと
いうことになる。

724 :名無しさん@英語勉強中 :2022/08/01(月) 05:33:15.26 ID:kLUo2uQN0.net
>>718
(1) 自分の馬鹿さをひけらかしてこれ以上俺たちを悲しませないでくれ

ところで「ひけらかす」というのは show off (your stypidity) の方が厳密に言うと
日本語の趣旨に近いわな。boast about は少しだけずれる。

もしも二通りの意味に解釈されるのをどうしても避けたいのなら、どうするか?
難しいけど、俺なら思い切って意訳的に次のように訳すかな?

>>自分の馬鹿さをひけらかしてこれ以上俺たちを悲しませないでくれ
All you've been doing is showing off your stupidity to upset us. Stop it already.

ところで、ここでの already は「すでに、もう」と訳したらダメだな。
アメリカの口語英語でよく使われるこういう場合の already の意味合いを知らない人は、
辞書を見てくれ。

725 :名無しさん@英語勉強中 :2022/08/01(月) 05:33:56.25 ID:VN2CdJWU0.net
質問です
1, 2 はそれぞれどういう意味になるのでしょう?
また、この最後の did は省略可能ですか?

1. He made fun of me like you did.
2. He didn't make fun of me like you did.

1'. He made fun of me like you.
2'. He didn't make fun of me like you.


1', 2' (did の省略)が可能なら、1', 2' はどういう意味になるのでしょう?
(可能でないなら、1, 2 はどういう意味になるのでしょう?)

1. あなたが私を馬鹿にしたように、彼も私を馬鹿にした

2a. あなたは私を馬鹿にしたが、彼は私を馬鹿にしなかった
2b. あなたは私を馬鹿にし、彼はあなたとは別の仕方で私を馬鹿にした
2c. あなたも彼も私を馬鹿にしなかった

2a と言いたいときに、unlike を使って次のように言えますか?

He didn't make fun of me unlike you.

726 :名無しさん@英語勉強中 :2022/08/01(月) 05:47:04.06 ID:VN2CdJWU0.net
>>723,724
ありがとうございます
Thank you! Your answer wasn't ambiguous like the other's.

この場合、Your answer wasn't ambiguous は確定なのですが、
the other's one が ambiguous だったのか否か、正反対の解釈ができてしまうのではと気になっています

日本語でも、
あなたの回答はもう一人の回答のように曖昧ではありませんでした
だと、もう一人の回答が曖昧だったのか、そうじゃなかったのか、確定しないような…

そこで、日本語と英語で次のように言えば(もし言えるなら)、一意に確定するのでは、と思いました

あなたの回答はもう一人の回答のようには曖昧ではありませんでした
Your answer wasn't ambiguous unlike the other's.

英語でこのように unlike って言えるのでしょうか?

727 :名無しさん@英語勉強中 :2022/08/01(月) 06:08:49.53 ID:VN2CdJWU0.net
>>719
ありがとうございます

728 :名無しさん@英語勉強中 (ワッチョイ d689-G1eK):2022/08/01(月) 06:17:30 ID:kLUo2uQN0.net
>>726
(1) あなたの書いた日本文: あなたの回答はもう一人の回答のようには曖昧ではありませんでした
(2) あなたの英訳案: Your answer wasn't ambiguous unlike the other's.

俺にとっては、この (1), (2) は両方とも少し気持ちが悪い。(1) のような日本文は、
確かに正しいとされていて、そういう書き方をする例はよく見る。しかし
少しでも急いで本を読んでいる人は、ここでの「〜のようには」の「は」を
読み落としてしまって、どっちの意味なのか分からなくなって、その文を
再び読み直さないとわからなくなってしまう。

英文については、これで正しい英文なのかどうか俺にはわからんが、仮に正しいとしても、
あまり気持ちのいい英文ではないと思う。

じゃあ、どうするか?俺ならこうする。

(1) の日本文の改訂案:
(1a) 別の人の回答とは違って、あなたの回答は曖昧ではありませんでした
上の日本文よりもさらによいのは、
(1b) 別の人の回答とは違って、あなたの回答は明快でした。

(1a) の「〜ではない(ではなかった)」という否定形よりも、はるかに
「〜である、〜でした」という肯定文の方がわかりやすい。だからこそ、
「曖昧ではない」と書くよりも「明快である」と言い切ってしまった方が
いい場合がよくある。今回の場合は特にそうだと思う。

(2) あなたの英訳案である Your answer wasn't ambiguous unlike the other's.
のうち、unlike... を文頭に置くと、意味がはっきりする。
Unlike the other's, your answer wasn't ambiguous.

729 :名無しさん@英語勉強中 (ワッチョイ d689-G1eK):2022/08/01(月) 06:17:41 ID:kLUo2uQN0.net
なお、the other's は the others と勘違いしやすく、それを耳で聞くと余計に混乱する。
だから the other's という言い方はなるべく避けるというのが、まともなネイティブの
書き方だろうと思う。だからこそ、一般的にまともな洋書や新聞雑誌には
the other's などという言い方はまったく出てこないのではないか。出てくるとしたら、
YouTube とか Twitter などの、まともでない下手な英文を書く英語ネイティブや
少し英語の下手な外国人が書いた英文の中にしか出てこないと思う。

結論として、次のように俺なら書く。

(1a) Unlike the answer by the other poster, yours was clear.
(1b) Your answer was clearer than the one from the other poster.

日本文でも英文でも、できれば否定形を使わず、なるべく肯定文を使う工夫をすれば
すっきりする。

まあ、俺が言っていることは日本語の書き方についての「文章読本」とか
英語で書かれた "style book" とか言われるものを2,3冊ほど読めばよくわかると思う。

日本語の本では「理科系の作文技術」という中公新書の本が役立つ。これは
理科系の人だけでなく、一般的にあらゆる人が日本文を書く時に注意すべきことを
見事に解説しきった名著として名高い。

英語で書かれたものとして短いものとしては、Strunk and White という二人が書いた
"The Elements of Style" が有名。

730 :名無しさん@英語勉強中 (ワッチョイW 7a5b-XW1E):2022/08/01(月) 06:23:08 ID:VN2CdJWU0.net
I am not disillusioned with people like you.

この意味は、
a 私は人々に幻滅していない、あなたも人類に幻滅していない
b 私は人々に幻滅していない、あなたは人類に幻滅している
c 私は人々に幻滅していない、あなたには幻滅している
d 私は人々に幻滅していない、あなたにも幻滅していない
e 私はあなたのような人々に幻滅していない

文法的に成り立たないものはどれなのでしょう…?
c, dは成り立たないかな…?

731 :名無しさん@英語勉強中 :2022/08/01(月) 06:34:43.37 ID:VN2CdJWU0.net
>>728
ありがとうございます
確かに肯定形で書けるならそれが一番ですね
でも否定形にしたい場合や、
(そんなに危険ではない、と、そこそこ安全だ、は論理的には等価でも、危険かどうかに皆の意識がある場合は、前者を使いたかったりします)
口語で、追加情報を補足的に挿入する場合、あなたの回答は… とすでに話し始めている場合がよくあります

unlike を文頭に置く形はよく見ますね
でも後ろにつける形は全然見ません
いつも、「どちらの意味にもとれるのに、よくこんな曖昧な言い方をしていて正気を保っていられるな」
と思いながら外国人が喋るのを聞いています

732 :名無しさん@英語勉強中 :2022/08/01(月) 06:39:26.07 ID:kLUo2uQN0.net
>>725
>>1'. He made fun of me like you.
>>2'. He didn't make fun of me like you.

会話とか気楽な殴り書きの文章(つまり YouTube や Twitter などの SNS つまり
social media 上の文章)では、上のような言い回しはよくあるだろうな。
だからこそ、それは一応は正しいとされている。

しかし、
He make fun of me like you.
というと二通りの意味に解釈することができる。
(1) He made fun of me like he made fun of you.
--- これを普通は He made fun of me like he did of you. って言うんだろうなと思う。
(2) He made fun of me like you made fun of me.
そしてこれを普通は He made fun of me like you did. って言うのだ。

会話などでは、その前後の状況や相手の顔色などを見て、そのうちのいずれの
意味なのかが分かる場合が多いからこそ、like you did と言わずに like you で
済ませてしまうことが多いのだろうと思う。

>>2a. あなたは私を馬鹿にしたが、彼は私を馬鹿にしなかった
>>He didn't make fun of me unlike you.

これも二通りに解釈できる。だからこそ、こういう言い方をしないで
You made fun of me, but he didn't.
とした方が圧倒的によいということになる。

733 :名無しさん@英語勉強中 :2022/08/01(月) 06:49:10.30 ID:VN2CdJWU0.net
>>729
ありがとうございます
否定形はどんな場合でも肯定形より意味が分かりにくくなりますよね
気を抜くと、否定の否定とか、否定の否定の否定とかに発展しかねないとも言えなくもありませんし

語勢を弱めるために、(断定的に聞こえないようにするために)
日本語では否定形で書きがちなのですが、
英語では肯定形を多めにして書くようにしようと思います

734 :名無しさん@英語勉強中 :2022/08/01(月) 07:00:32.32 ID:VN2CdJWU0.net
>>732
ありがとうございます
もう、数学の記号ような、あるいはプログラミング言語のような、
もっと論理的な言語を発明してそれで会話したいような気持ちになります

人々が会話するのを見ていると、
(特に、色んなものがよく省略される日本語での会話)
文章の意味がちゃんと伝わっていないがために会話に支障が出ていることが非常に多くあるように感じていまいます
みんな、厳密な意味を考えようとせず、
自分に都合のよい解釈を採用して会話を進めてしまう

英語では英語話者は詳細に物事を述べることが多いので、
問題はそこまでひどくはありませんが

735 :名無しさん@英語勉強中 :2022/08/01(月) 07:11:23.23 ID:iujRKu8c0.net


736 :名無しさん@英語勉強中 :2022/08/01(月) 07:36:19.99 ID:ts3AJIL90.net


737 :名無しさん@英語勉強中 :2022/08/01(月) 07:39:40.57 ID:kLUo2uQN0.net
>>734
俺もよくそれに似たことを考えたことがあった。曖昧な言い回しで平気で話しかけられて、
イライラすることがよくあった。

でも、人間には論理性を追求する数学者みたいな態度と、その反対に曖昧さをどこまでも
追いかける芸術家みたいな態度とがあるんだろうなと思う。

売れなくてもいいから生涯を賭けて芸術を希求する人2人と本気で付き合ったことが
あったけど、二人とも、どんなに努力しても曖昧で非論理的な話し方(書き方)しか
できない人だった。

でも、たとえば詩とか歌では、曖昧さこそが勝負だと思う。サトウハチローの
「小さい秋、見つけた」という詩の「誰かさんがちいさい秋、見つけた」という
言葉に触れて、「小さい秋なんていう変な言葉を使うな。秋というものが
ちいさいとか大きいとか言えるはずねえだろ。秋を象徴するような小さいもの、
たとえば秋に咲く小さな花とかを見つけたんだろ?それならそうとはっきり言え」
なんてことを言うと、芸術もへったくれもなくなる。

何通りにも何十通りにも解釈できるような言葉や音や画像を見せて、それを
宗教的にも哲学的にも、あるいは政治学的にも歴史的にも、あるいは心理学的、
あるいは性的、あるいは実利的にも、そして愛とか優しさを表現しているようにも、
あるいはその逆に残虐さや恐ろしさを表現しているようにも解釈できるような
作品こそが、芸術として優れたものだと言える。

もしすべてが論理的であったら、息苦しくて、まったく面白くない世の中になるだろうな。
曖昧さこそが面白さを生み出すんだろうな。そして同時に、この曖昧さこそが
愛情や感動を生み出す一方で、争いや誤解や戦争や憎悪を生み出すんだろうな。

738 :名無しさん@英語勉強中 :2022/08/01(月) 08:51:39.61 ID:mH4nyQ250.net
むかしむかし、VOAのSpecialEnglishの
デイリーの30分ニュースを楽しんでいたんですが、
いまあんなふうに「簡単な英語で」「時事ニュースを」「30分くらい続けて」
みたいなプログラム、ネットのどこかにありませんかね。

739 :名無しさん@英語勉強中 :2022/08/01(月) 09:37:11.23 ID:xaVMCgpjd.net
>>738
まだあるけど…

Learning English Broadcast - VOA - Voice of America English News
https://learningenglish.voanews.com/z/1689

740 :名無しさん@英語勉強中 :2022/08/01(月) 09:44:58.89 ID:mH4nyQ250.net
>>739
お恥ずかしい・・・

久々に聴きに行ってみたら、短いこま切れのニュースしか
みつけられなかったよ ありがとう

741 :名無しさん@英語勉強中 :2022/08/01(月) 10:05:32.32 ID:GVvMBXVs0.net


742 :名無しさん@英語勉強中 :2022/08/01(月) 10:44:25.31 ID:gX/4ugag0.net


743 :名無しさん@英語勉強中 :2022/08/01(月) 14:12:11.17 ID:VN2CdJWU0.net
>>737
曖昧さと正確さは、同時に使おうとしないならば、
同居できるのかもしれないし、できないのかもしれない
(論理的な思考が必要な局面では正確に言葉を使うようにする、など)

私が人類を見ていて思うのは、
あまりにも言語能力が低すぎる人が多すぎるのでは、
そして、そのために本来必要でない意思疎通の問題や争いが起こっていることが非常に多いのではないか、ということ

曖昧な表現は、
議論を煙に巻くだけてなく、
自らの思考を狂わせてしまったりする
そして当人がそのことに気づかない

現状、曖昧さの不足よりも正確さの不足により
重大な問題が人類には多く出ているように思う
(そのため、平和が損なわれている)

人々がもっと言葉を正確に使えるようになれば、
数学の世界(数学に関わる人々がいる世界)がそうであるように、
世の中はずっと平和になるのではと思う

744 :名無しさん@英語勉強中 :2022/08/01(月) 14:19:08.70 ID:VN2CdJWU0.net
数学の世界では、誰が言っていることが正しいのか、
誰でもわかる(まともな思考力を持っている人ならば)

正確さが求められる局面で曖昧な言葉の使い方を良しとする世界では、
おかしなことを言っている人が、自分が正しいと思い込んで、
偉そうにしている
あまつさえ、正しいことを言っている人を攻撃したりする

745 :名無しさん@英語勉強中 :2022/08/01(月) 15:18:58.59 ID:VN2CdJWU0.net
>>730への回答まだ募集しています
(OEDさん以外の人で。OEDさんにはすでに色々回答してもらったので)

746 :名無しさん@英語勉強中 :2022/08/01(月) 15:27:38.18 ID:feGUxsy70.net
Stop embarrassing yourself by making an ass of yourself.

747 :名無しさん@英語勉強中 :2022/08/01(月) 15:30:07.01 ID:VN2CdJWU0.net
730をもう少し穏やかに、そして平易にするために、別の文章に書き直してみる

I am not dependent on people like you.

この意味は、
a 私は人々に依存していない、あなたは人々に依存していない
b 私は人々に依存していない、あなたは人々に依存している
c 私は人々に依存していない、あなたに依存している
d 私は人々に依存していない、あなたに依存していない
e 私はあなたのような人間に依存していない

この中で文法的に成り立たないものはどれかあるんだろうか?

748 :名無しさん@英語勉強中 :2022/08/01(月) 16:54:06.65 ID:VN2CdJWU0.net
I like people, you like people.
= Like you, I like people.
= I like people like you.

I like people, you don't like people.
= Unlike you, I like people.

I don't like people, you like people.
= Unlike you, I don't like people.
= I don't like people like you. (1)

I don't like people, you don't like people.
= Like you, I don't like people.
= I don't like people like you. (2)

From (1) and (2),

I don't like people like you.
= I don't like people, you like people.
= I don't like people, you don't like people.

???

749 :名無しさん@英語勉強中 :2022/08/01(月) 18:03:08.36 ID:kLUo2uQN0.net
文法的に成り立つかどうかという問題は、俺にとっては難しくて、
あんまり考えたくもないけど、俺だったらそういうことよりも、
「曖昧ではなく、1通りの意味しか持たないような文をどうすれば書けるか」
ということの方に関心がある。

people like you を使って I am not dependent on people like you. なんて書けば
両義的になるかもしれんからそれは避けて
I am not dependent on your kind of people.
と言えばしまいだと俺なんかは単純に考えてしまう。それから、
I am not dependent と言うと syllables が多くて長ったらしくなるから
I don't depend と言いたくなり、
I don't depent on your kind of people.
と言いたくなる。こうすれば短くて、しかもおそらくは1通りの意味にしかならないと思う。

I don't like people like you. なんて書くと、like が2つも出てきて、しかも
別々の意味を持つ like が使われていて、俺にとってはわかりにくく感じられるから
これもやっぱり
I don't like your kind of people.
と言いたくなる。

750 :名無しさん@英語勉強中 :2022/08/01(月) 18:35:16.30 ID:VN2CdJWU0.net
> 「曖昧ではなく、1通りの意味しか持たないような文をどうすれば書けるか」
> ということの方に関心がある。

私もそう
簡潔明瞭、過不足なく、正確に物事を記述したい
だけど世の中の文章は曖昧な場合が少なからずある、というか、
言語自体(あるいはそれを使う人々の習慣自体)が曖昧さを許容し、
曖昧な表現方法が使われ続けていることに若干の疑問や不満がある
まあ、自分の使用法だけ改めればよい問題ではあるが。。

I am not dependent on people like you. の like you は、
your kind of people の意味ではなくて「あなたと違い」の意味で使われている場合を想定して書いた
日本語で書いても、「あなたのように」は2通りの意味に取れる(ので厄介だ)

私は、あなたのように他人に依存していない
(あなたは他人に依存しているのか、いないのか)

英語の場合はこれに加えて、「あなたのような」という解釈も出てくる

私は、あなたのような人間に依存していない

まあ、言語の曖昧性をこねくり回しているだけ
OEDさんは「~のようには」という表現はあまり好きではないようだけど、>>728
私は、これで1通りの意味に限定できるので、肯定的
英語でも、unlike ~ のような表現を使いたい

I don't depend on people unlike you.

ここは、like にしても通常否定の意味で解釈されると思うのだけど、
論理的な意味が否定なので、言葉の上でも否定形を使いたい(整合性、一貫性を持たせたい)

必ず と 必ずしも のように、~のように や like にも ~のようには のような否定形がほしい(論理的整合性の観点から)

751 :名無しさん@英語勉強中 :2022/08/01(月) 18:46:32.37 ID:VN2CdJWU0.net
魔王「前回のようにうまく行かんぞ」
(前回、うまくいったのか、いかなかったのか?)

魔王「前回のようにはうまく行かんぞ」
(前回うまく行ったことが確定)

It won't go well like previous time.

It won't go well unlike previous time.

前者だと前回うまくいったのか否かがわからない
後者だと意味が明確

752 :名無しさん@英語勉強中 :2022/08/01(月) 18:53:55.56 ID:Kxa1RH770.net
There is nothing left to do.
やることが何もない

ですが文法的にどう考えるのでしょうか?
nothing left に後ろからto doが形容詞的にかかり、doの目的語にnothing left がかかっているのか

ということかleave nothing to do が元々あってdoの目的語にnothingがなっていて
nothing to doが、する何もないこと、で、
それを残す、ということなのか。

753 :名無しさん@英語勉強中 :2022/08/01(月) 18:57:18.12 ID:eMjfmHH80.net
I don't depend on people like you
あなたのような人には頼らない

I don't depend on someone as you do
あなたするみたいに他人に頼らない

754 :名無しさん@英語勉強中 :2022/08/01(月) 19:02:38.57 ID:VN2CdJWU0.net
個人的に、本来ならば、

It won't go well like previous time.
前回うまくいかなかった、今回うまくいかない

It won't go well unlike previous time.
前回うまくいった、今回うまくいかない

が自然だと思う
が、実際は前者の文の like previous time が否定の意味で使われることが多い

単語を見ると、won't で否定が使われていて、
それが残りの文全体にかかるのか、like 以降にはかからないのか
の解釈の違いなのだが、
ここはかかるべきだろうと思う

最初に使い始めた人たちが、肯定文
It went well like previous time.
のまま否定文でも like 使い始めたので、
ここが曖昧になってしまったのだろうと思う

日本語の 必ず・必ずしも のように、
あるいは英語の or と nor のように、肯定形と否定形があると意味が明確になるかと思った
(が、or と nor ももはやあまり重視されていないのかもしれない)

755 :名無しさん@英語勉強中 :2022/08/01(月) 19:19:35.48 ID:VN2CdJWU0.net
「私は、あなたのように他人に依存していない」

でも意味は通じるのだが(相手の意を推測して汲めば)、
曖昧さを許容し含有したまま論を構築していくことに脆さ・危うさを感じる

人間は、誰しもが言葉を論理的なコードに変換して意味を明確にしながら会話しているわけではなく、
多くの人は、全体の印象だけでなんとなく会話を進めている

そういう人間の性質を考えると、OEDさんの方針のように、なるべく肯定形で表現するか、
部分否定を言語に導入するか、が好ましいように思う
(否定形で表現したい場合も考えると、両方)

駄文失礼
これ以上は自らのブログにでも書くことにします
以上

756 :名無しさん@英語勉強中 (ワッチョイW 7de9-WoXG):2022/08/01(月) 19:51:01 ID:RM6S/HRr0.net


757 :名無しさん@英語勉強中 (ワッチョイW 7de9-WoXG):2022/08/01(月) 20:04:50 ID:I8/RpleN0.net


758 :名無しさん@英語勉強中 (ワッチョイW dae9-WoXG):2022/08/01(月) 20:18:37 ID:o5KOuSFH0.net
5r

759 :名無しさん@英語勉強中 :2022/08/01(月) 21:43:25.63 ID:YTvl6RHna.net
たぶん英文で曖昧さのない明確な文章を書きたい人は、OgdenのBasic Englishをやればいいと思う
辞書の整理をやってて思いついたものだそうで、すべての英単語をきっちり明確に辞書的な定義をするために必要な850語を選定して体系化している
これの最大の欠点が、あいまいな言い方ができなくて、曖昧に書かれた原文を訳すときにバイアスがかかってしまうことなんだけど、そのほうがいいという人にとっては欠点ではないだろう

760 :名無しさん@英語勉強中 (ワッチョイW 45fb-LRMP):2022/08/02(火) 00:28:22 ID:NsX+4Xp70.net
>>752の回答もよろしくお願いします

761 :名無しさん@英語勉強中 :2022/08/02(火) 05:18:37.99 ID:kB9SoKTLa.net
>>760
to doはleftにかかってるんじゃないの?
leave to doで「(人に)自由にやらせる」とか「(人の)したいようにさせる」という意味で、そうできるようなものがnothingだと思ったから、その書き込みを見てへんなとこで区切ってるなと思った

762 :名無しさん@英語勉強中 :2022/08/02(火) 06:05:49.53 ID:U8ihDUO+a.net
>>682
it thatの強調構文で副詞を強調する場合、thatは接続詞でした。トイグルさんに載ってます。でもトイグルさんはあまり気にするなと言ってます。
ちょっとスッキリした!

763 :名無しさん@英語勉強中 :2022/08/02(火) 07:03:51.46 ID:7JaRoLGW0.net
>>762
It is ... that SV は、確かに強調構文だな。でも、それはここでは関係ないのに、
なぜあなたはそのことを引き合いに出すのかな?まさか

>>671
>>This is the first time (that) I have visited London.

これが強調構文だとは思ってないよね?

764 :名無しさん@英語勉強中 (アウアウウー Sa09-8+a8):2022/08/02(火) 07:57:11 ID:61mdYkyva.net
>>763
あれー!thisだったよね。も一回考えるよ。

765 :名無しさん@英語勉強中 :2022/08/02(火) 09:56:01.12 ID:NsX+4Xp70.net
There is nothing to be done.
とせずに、
There is nothing to do.
とするのはなぜなんだろう。


I cannot find a house to live in.
とか

Dickens had a large family to support
とかなら不定詞の主語はわかる。

だけど、There are is nothing to do.
だと不定詞の意味上の主語がない。
つぶらきくらいしかこういう文章の使い方がないので主語がないと
いうことでいいんでしょうか?

766 :名無しさん@英語勉強中 :2022/08/02(火) 12:48:16.02 ID:7JaRoLGW0.net
>>765
(1) There is nothing to be done. とせずに、There is nothing to do. とするのはなぜなんだろう。

to be done であろうと to do であろうと、本質的な意味はきちんと読者に伝わるから、
どちらを人は選ぶかというと、なるべく短い方(したがって発音もしやすい方)
を選ぶわな。いわゆる economy of words の原理によって、人は短い方を選ぶ。

(2) I cannot find a house to live in. とか Dickens had a large family to support
とかなら不定詞の主語はわかる。

確かに、前者なら to live in の主語が I だろうなと思う人は多い。でもそうとは限らない。
I have a lot of refugees to support. But I can't find a house (for them) to live in. かもしれん。
あるいは妾の住まいを探してるのかもしれん。

(3) だけど、There are is nothing to do. だと不定詞の意味上の主語がない。
★つぶらき★ くらいしかこういう文章の使い方がないので主語がないと
いうことでいいんでしょうか?

「つぶらき」って、「つぶやき」つまり Twitter 上のつぶやきのことかな?もしそうなら、
こういう構文はつぶやきだけじゃなくて、純文学やお堅い文章の中にも出てくる
ごく普通の言い回しだと思うよ。

そもそも、主語を明示しなくても前後の関係でそれがえわかるときには、誰でも
大いに省略するのが人間の性だろう?

767 :名無しさん@英語勉強中 :2022/08/02(火) 12:52:57.25 ID:7JaRoLGW0.net
>>765
(3) の There is nothing to do. について言えば、それとまったく同じような構文で
(まるでその英文を直訳したかのような)文がドイツ語とかフランス語でも
使われてるだろ?

ドイツ語: Es gibt nichts zu tun.
フランス語: Il n'y a rien a faire. (ただし a には accent grave というフランス語特有の記号がつく)

その他、いろんな言語においても同じなんだろうと思う。

768 :名無しさん@英語勉強中 :2022/08/02(火) 13:12:25.73 ID:NsX+4Xp70.net
>>766
>>767
ありがとう。

I have a lot of refugees to support. But I can't find a house (for them) to live in. かもしれん。
あるいは妾の住まいを探してるのかもしれん。

表現として適切か、というのは残ると思うんだけどね。

769 :名無しさん@英語勉強中 :2022/08/02(火) 13:27:28.54 ID:7JaRoLGW0.net
>>768
確かに、たとえば I can't find a house to live in. と単に言ったときは、for me to live in
を指す場合が圧倒的に多いとみんな考えるんだろうな。日本語でも
「住む家が見つからない」と言えば「自分(あるいは自分たち)が住む家が見つからない」
という意味になるのが普通であって、「(自分の)息子とか妾に住まわせる家」を
指したいときは、ちゃんとそのように言うんだろうな。

>>Zach, ★whose parents were forced to find a house for him to live in alone★ to meet registry
guidelines, is also subject to a long list of probation-related prohibitions that Buhl says are nonsensical.

上のような例文が見つかったけど、自分以外の人が住む家を言いたいときには、やっぱり
"for [someone] to live in" といちいち言うのが普通なんだろうと思う。

しかし、形の上ではあくまで「誰が住む家なのか」は明らかにしない形なので、
それを利用してお笑い番組とか映画の中で
I need a house to live in.
と言っている友人を見て、別の人がその家に住むのはその友人に決まっていると
解釈していたら、途中でその友人が
No, no. I'm not the one to occupy the house. I want my mistress to live in there.
と言って、聴衆がそこで笑う、なんていう場面が多いにありそうだな。
日本語でもそういうギャグは多いと思う。

770 :名無しさん@英語勉強中 :2022/08/02(火) 14:17:19.13 ID:wgXI/wOj0.net


771 :名無しさん@英語勉強中 :2022/08/02(火) 14:43:23.97 ID:Uoxjxnhd0.net
浪人を持っている方、埋め立て荒らしの通報をしてもらえませんか?
私が通報しようとしたところ、浪人を持っていないと通報できないようでした
次のスレに以下の本文を投稿してもらえればOKです

https://agree.5ch.net/test/read.cgi/sec2chd/1658452250/

・埋め立て(スレッドの趣旨とは関係の無い文言)
https://lavender.5ch.net/test/read.cgi/english/1655375318/756 2022/08/01(月) 10:05:32.32 ID:GVvMBXVs0
https://lavender.5ch.net/test/read.cgi/english/1655375318/742 2022/08/01(月) 10:44:25.31 ID:gX/4ugag0
https://lavender.5ch.net/test/read.cgi/english/1655375318/756 2022/08/01(月) 19:51:01.07 ID:RM6S/HRr0
https://lavender.5ch.net/test/read.cgi/english/1655375318/757 2022/08/01(月) 20:04:50.90 ID:I8/RpleN0
https://lavender.5ch.net/test/read.cgi/english/1655375318/758 2022/08/01(月) 20:18:37.84 ID:o5KOuSFH0
https://lavender.5ch.net/test/read.cgi/english/1655375318/770 2022/08/02(火) 14:17:19.13 ID:wgXI/wOj0

772 :名無しさん@英語勉強中 (ワッチョイ d689-G1eK):2022/08/02(火) 15:08:38 ID:7JaRoLGW0.net
>>751 >>754
ここは英語のスレであり、参加者はなるべく正しい英語を使えるようになりたいと
思っているからこそここにいるんだろうから、今から俺が言う言葉を許してほしい。

previous time を無冠詞で6回ぐらいこの投稿者は使っているけど、それに少し違和感を
感じて調べてみた。

(1) Google 画面のうち、ニュースサイトのページ
(2) Genius 英和の最新版
(3) Wisdom 英和の最新版
(4 The Oxford Advanced Learner's English Dictionary (OALD), 10th Edition

以上の資料だけしか見ていないけど、そこから判断する限り、どうやら
last time, next time とは違って、previous time の場合には「前回(に)」という
意味で the をつけるのが一般的のようだ。ニュースサイトで検索してみると、
少しではあるけど like previous time のように無冠詞で使う例もみられる。でも
出どころはインドのものが目立つ。一方、the をつけた例文はそれに比べると
はるかに多く、いろんな国の、しかもある程度は信頼性の高そうな news sources
から出ているものであるように俺には見える。

俺自身は、いろんな洋書とか新聞雑誌を読んできた経験では、「前回」とか
「今から直前の」という意味での「前の」という意味では
"the previous + 名詞" という形しか見たことがなかった。

ただし、「すぐ前の」という意味ではなくて「かつての」という意味で
"a previous + 名詞" という形、つまり the ではなくて a を使っている用例がある
ということはよく知っている。しかし、単数形の名詞が後に続くのに
まったくの無冠詞(つまり a も the もつかない)用例は、俺自身は通常の読書の中では
見たことがなかったので、今回、少し調べてみた次第だった。

773 :名無しさん@英語勉強中 :2022/08/02(火) 15:34:18.61 ID:3X1ZwaIKd.net
確かに無冠詞は珍しそうね

https://netspeak.org/#q=*+previous+time+

774 :名無しさん@英語勉強中 :2022/08/02(火) 15:44:00.44 ID:Uoxjxnhd0.net
>>772
ありがとう
たしかに
勉強のために書いているので、どんどんコメントもらえるのは嬉しい限り

775 :名無しさん@英語勉強中 :2022/08/02(火) 15:44:01.59 ID:P8skYcVF0.net
英語のスペルに関して質問したい。
ネイティブにしても英語上級者にしても、英語の人名を耳で聴いただけでその名前のスペルって書けるもんなの?
例えば日本語なら口頭で「山田太郎」って言われた場合、仮に漢字がわからなくても「ヤマダ タロウ」ってカタカナでも平仮名でも書けるよね。
でも英語の場合は音で聴いただけじゃスペルがわからなくない?似たような音の名前でもスペルが微妙に違うだろうし
音でスペルを聴き分けられるということなんだろうか。
そもそも俺はスペルのルール自体をよくわかってないんだけど。

776 :名無しさん@英語勉強中 :2022/08/02(火) 15:50:10.13 ID:AA/HqtfI0.net
>>775
書けない

777 :名無しさん@英語勉強中 :2022/08/02(火) 16:06:59.98 ID:Uoxjxnhd0.net
>>775
基本的に書けないが、場合によっては書ける、じゃないかな
日本語やドイツ語は音と綴りの対応が規則的だけど、
英語は不規則なので、同じ音でも複数の綴りが可能な場合がある

778 :名無しさん@英語勉強中 :2022/08/02(火) 16:14:20.30 ID:Uoxjxnhd0.net
例えば、バイという音を聞いて、
日本語だとバイ一つだけど、
英語だと buy by bi など複数の可能性がある

779 :名無しさん@英語勉強中 :2022/08/02(火) 16:16:17.65 ID:YN6fiq2P0.net
一般的な名前や経験からある程度なら分かるけど
例外も多いので音だけでは無理だね

Steven, Stephen
Stacy, Stacey
Campbell
Saoirse

780 :名無しさん@英語勉強中 :2022/08/02(火) 16:18:16.73 ID:Uoxjxnhd0.net
英語では音からスペルが確定しないので、
口頭で名前のスペルを1文字で伝えたりする
この時専用の伝え方があるようだ(調べれば出てくる)

781 :名無しさん@英語勉強中 :2022/08/02(火) 16:35:33.70 ID:P8skYcVF0.net
>>776
>>777
なるほど、基本的に書けないのか
そうすると英語の人名は漢字での表現(中国語)に近いっていう理解でいいのかな。口頭で聴いただけじゃ相手の名前(漢字)書けないし。

ちなみに英語と日本語ではどちらの方が同音意義語が多いんだろ?日本語の方が多い印象はある。
日本語なら無理矢理作ろうと思えば一文で5つくらい同じ音の単語を並べて作ることもできるけど、しっかり内容を理解するには文脈で判断しないといけない。
そういう意味では日本語より英語のほうが勉強しやすいのかな。

782 :名無しさん@英語勉強中 :2022/08/02(火) 16:37:29.84 ID:8yrqQeCv0.net


783 :名無しさん@英語勉強中 :2022/08/02(火) 16:41:35.35 ID:Uoxjxnhd0.net
>>781
同音異義語は圧倒的に日本語が多いと思う
日本語は音の数がとても少ないので同音異義語になりやすい

784 :名無しさん@英語勉強中 :2022/08/02(火) 16:41:56.30 ID:5vQors2C0.net


785 :名無しさん@英語勉強中 :2022/08/02(火) 17:00:49.03 ID:zZbKu/6h0.net


786 :名無しさん@英語勉強中 :2022/08/02(火) 17:05:13.18 ID:TwQBXUST0.net


787 :名無しさん@英語勉強中 :2022/08/02(火) 17:20:34.64 ID:jaKWzU8FH.net


788 :名無しさん@英語勉強中 :2022/08/02(火) 17:21:03.04 ID:PR8v9R270.net


789 :名無しさん@英語勉強中 :2022/08/02(火) 18:03:21.86 ID:NsX+4Xp70.net
日本語には漢字があるからね

790 :名無しさん@英語勉強中 :2022/08/02(火) 18:10:29.81 ID:P8skYcVF0.net
>>783
なるほど
ありがとう

791 :名無しさん@英語勉強中 (ワッチョイW f5e9-WoXG):2022/08/02(火) 18:22:40 ID:bUQY2nA+0.net
tt

792 :名無しさん@英語勉強中 :2022/08/02(火) 21:04:55.39 ID:NrTIbIlfa.net
漢字で識別できるからって日本語では無頓着に同音異義語を増やしてしまって、音声による識別ではなく文脈で漢字を連想して意味を識別するという、世界でも類を見ない珍しい言語になってしまった。
この日本語でのクセを直せるかどうかが、英語をはじめとする外国語を勉強する上での最大の課題だと思う。

793 :名無しさん@英語勉強中 :2022/08/03(水) 01:39:06.77 ID:BhIhQ3Y9d.net
英語の場合、同じ単語で違う意味になるの辛くない?
日本語が文法的に文脈依存なのは分かるけど、単語としてはそれほどブレない気がする

794 :名無しさん@英語勉強中 :2022/08/03(水) 06:25:08.11 ID:xUvydYUL0.net
to be bornとborn
どちらも、生まれる、という意味だと思うんですが、両者のニュアンスの違いはなんでしょうか?

795 :名無しさん@英語勉強中 :2022/08/03(水) 14:57:52.87 ID:W5MJtMVra.net
bornはbearの過去分詞形

796 :名無しさん@英語勉強中 :2022/08/03(水) 17:47:41.72 ID:xUvydYUL0.net
どういうことでしょうか
ヒント貰ってもわからないのです

797 :名無しさん@英語勉強中 :2022/08/03(水) 17:52:21.58 ID:xHkROPhH0.net
違いは無い
気にしなくて良い

798 :名無しさん@英語勉強中 :2022/08/03(水) 18:03:43.71 ID:KFzmTNNQ0.net
>>794
あなたがどこからどこまで理解しているのか、どこからわからなくなっているのか、
それがまったく周囲の人にはわからないから、どう説明していいかわからない。
あなたのたったの1行のきわめて単純な質問の仕方に触れて、それでもなおかつ
説明しようとすると、少なくとも 800 字くらい、ひょっとしたら 2,000 字くらいの
長文を書かないといけなくなる。

でもそういう下手な説明を俺たちが試みたとしても、それはすでに市販の
500 ページから 700 ページくらいの中学・高校生向けの文法書に
すべて書いてある。

だからこそ、誰も回答してくれないんだよ。

もしそれでもどうしても質問したいのなら、「これこれこういう本には、
これこれこういう英文が書いてあり、その和訳はこれこれこうだと書いてあり、
そのうちの born はこれこれこういう意味で、to be born はこれこれこういう
意味で、その違いはこれこれこうだと書いてあります。でもその意味が
私にはわかりません。教えてください。」

そういうふうに質問してくれたら、俺たちも答えようがあるというものだ。

799 :名無しさん@英語勉強中 (ワッチョイ fa29-G1eK):2022/08/03(水) 18:55:07 ID:3tCftS270.net
badassって表現について、He is badassって言い方は聞き覚えもあるし分かるんだけど
これを例えばShe is a badass playerみたいに女性に対して使うのは違和感ある? それとも全然ある?
もっと言えば目上の人とかなんらかの尊敬を受けるべきような立場の人に対して使うのはどう?

800 :名無しさん@英語勉強中 (ワッチョイW 5d9d-S0IY):2022/08/03(水) 18:56:46 ID:r+giVAf50.net
余裕でasshole言ってくるからねあの子ら…

801 :名無しさん@英語勉強中 :2022/08/04(木) 02:32:40.82 ID:r7r2wv3BM.net
>>769
>>No, no. I'm not the one to occupy the house. I want my mistress to live in there.
>>と言って、聴衆がそこで笑う、なんていう場面が多いにありそうだな。

無い。

そして
live in there → live there
多いに→大いに

802 :名無しさん@英語勉強中 :2022/08/04(木) 06:01:06.14 ID:nKNbSOsI0.net
小説にもごく普通に出てくる英語表現

live in there
stand on there
go in there
stay in there
in front of there

超初心者は黙ってな。

803 :名無しさん@英語勉強中 :2022/08/04(木) 06:05:03.29 ID:nKNbSOsI0.net
もちろんこれもよく見聞きする。

come in here
come in there
come out of there
go out of here
stay on here

804 :名無しさん@英語勉強中 :2022/08/04(木) 06:16:44.14 ID:eIRDNyqta.net
日本人がピアスの穴のことをピアスホールというのを聞くと、Pee assholeと言われたように聞こえて身構える
英語ではピアス穴くらいに小さい穴はholeじゃなくてpointだね

805 :Seeker2022 :2022/08/04(木) 09:09:43.54 ID:vfckNEfP0.net
>>801
thereは、名詞の働きもするのである。だから、in thereは語法的に十分正しい。

ジーニアス英和辞典からの引用
there[名詞]
What're are you doing in there? そこで何をしているの?

806 :名無しさん@英語勉強中 :2022/08/04(木) 14:39:18.72 ID:6x2+T+dg0.net
I saw the sun begin to dim
太陽が沈み始めるのを見たんだ
And felt that winter wind
そして冬の風を感じた

このネタ何度か出たかもしれないですが
一行目、beginsでないのはなぜ?
I saw と時制があってないのはあり得るとして
the sun って三人称単数ですよね?

807 :名無しさん@英語勉強中 :2022/08/04(木) 14:42:31.31 ID:6x2+T+dg0.net

自己解決しましたw
知覚動詞のあとは動詞の原型(または動名詞)なんですね

808 :名無しさん@英語勉強中 :2022/08/04(木) 18:23:55.54 ID:XBqkmjat0.net
I live twice the distance from the university as she does.

この比較文の省略箇所を元に戻した文を教えてください
あと live の後にat が省略されているとしたら、そのatは 省略文を元に戻したあとでは
twice の前か後のどちらの位置になりますでしょうか?

809 :名無しさん@英語勉強中 :2022/08/04(木) 20:36:37.26 ID:46SdZJsZa.net
まったく省略があるとは思えないんだが…
「私は大学から彼女の2倍の距離に住んでます」
she doesのdoesがlivesの言い換えだってことぐらいしか言うべきことが思い浮かばない
比較文でなければ、たとえば
I live 2km away from the university.みたいな言い方になると思う

810 :名無しさん@英語勉強中 :2022/08/04(木) 21:57:04.37 ID:ihRbqyMd0.net
例文ないんだけど
what’dはdidなのかwouldなのか聞き分ける方法教えて下さい

811 :名無しさん@英語勉強中 :2022/08/04(木) 22:01:50.58 ID:nL7hhi6u0.net
>>808
the distanceの前置詞はatだから
I live at twice as long as distance from the university as she does.
でいいんじゃないかな?

double distanceとか言えればそれでもいいんだろうけど。

812 :名無しさん@英語勉強中 :2022/08/04(木) 22:05:09.59 ID:nL7hhi6u0.net
twice distanceはダメなんだわ
理由はtwiceは副詞だから。

I live twice as distantly from the university as she does.

でもいいんじゃないかな?

813 :名無しさん@英語勉強中 (ワッチョイ 45fb-oJaN):2022/08/04(木) 23:25:31 ID:nL7hhi6u0.net
>>808
twiceの後に atですよ。理由は812

814 :名無しさん@英語勉強中 :2022/08/05(金) 00:51:54.75 ID:0jrg/Q/U0.net
>>808
>>I live twice the distance from the university as she does.

この
Jenna is twice as far from the finish line as she is from the start of a 10 km race. (ネット上にある例文)

815 :名無しさん@英語勉強中 :2022/08/05(金) 00:51:54.87 ID:0jrg/Q/U0.net
>>808
>>I live twice the distance from the university as she does.

この
Jenna is twice as far from the finish line as she is from the start of a 10 km race. (ネット上にある例文)

816 :名無しさん@英語勉強中 :2022/08/05(金) 01:22:51.99 ID:0jrg/Q/U0.net
さっきの2つの書き込みは、無視してくれ。

>>808
(1) 原文: I live twice the distance from the university as she does.

この英文は、教科書によく出てくる英文としては
(2) I live twice as far from the university as she does (= lives).
という形ならみんなすぐにわかるわな。

参考 ----- 上の (2) に似た例文:
Jenna is twice as far from the finish line as she is from the start of a 10 km race.
(ネット上にある例文)

(2) が (1) のような変化球になったんだろうな。
(2) がまずは
(3) I live at twice the distance from the university as she does.
となる。この at はあってもなくてもいい。I live at two miles from .... の場合でも、
やはり at はあってもなくてもいい。

(続く)

817 :名無しさん@英語勉強中 :2022/08/05(金) 01:23:26.16 ID:0jrg/Q/U0.net
>>808
(続き)
このような as は、that でも同じような意味になるようだ。

... the Russians knew that the allied fleets were
at almost double the distance from it ★that★ they were
(ネット上にある例文だが、これは Google.books に収録されている "Russia and the War"
という歴史書か何かの書籍の中の英文らしい。ここにある from it の it は Sevastopol
という船のことであるらしい。)

一般的に、that と as はときどき入れ替わって同じような意味で使われることがある
ようだ。現に、たとえば England の Midlands あたりの方言がよく出てくる
D.H. Lawrence の "Lady Chatterley's Lover" では、I think that S V. というときの
that を片っ端から as に置き換えている。これについて詳しく知りたければ、
Penguin Books 版のその小説の巻末にある方言解説を参照してほしい。
その方言解説をここに少しだけ引用する。

(1) Not ★as★ Ah know on, the' isna = Not ●that● I know, there isn't

(2) pleece ★as'll★ du = place ●that● will do

上に挙げた小説以外でも、たとえば Thomas Hardy とか George Eliot の小説などにも
方言が頻繁に出てくるが、そこでも似た現象が大いに見られたように俺は記憶している。
詳しいことは忘れてしまったが。

818 :名無しさん@英語勉強中 :2022/08/05(金) 01:32:27.73 ID:0jrg/Q/U0.net
>>808
>>817 で述べたような、接続詞 that に似た意味の as の使い方は標準英語にも
見られるけど、方言において顕著に現れるときがあるようだ。それについて
詳しくは通常の辞書ではあまり解説がないけど、The Oxford English Dictionary (OED) の
online 版にはたくさんの例文と共に詳しく解説してあるはずだ。

ただ俺は今のところ俺はそれを参照できない。なぜなら、
その online 版に今年の分の購読料金を払い込もうとしてもうまく行かず、なぜうまく行かないかを
Oxford University Press に 2 度にわたって質問しても、いくら待っても返事が来ない。
(俺自身のクレジット会社に尋ねたら、クレジットカードにも問題はないし、
俺の操作にもまったく問題はない。どうやら、OUP の会社のサイトそのものに
問題があるとしか思えない。というのも、それ以前の 10 年間はこれとまったく同じ
操作をしていても、まったく問題が起こったことがないからだ。今年だけ変なのだ。)
最近の OUP という会社の対応は、20年前に比べると格段に悪くなった。質問しても
答えてこない。5日くらい経ってから返事が来たかと思うと、機械による自動回答で、
その回答はピント外れ。

819 :名無しさん@英語勉強中 :2022/08/05(金) 01:37:33.24 ID:0jrg/Q/U0.net
>>817 の例文である
... the Russians knew that the allied fleets were
at almost ●double● the distance from it that they were
にも示したように、他の回答者が言ったように at twice the distance の代わりに
at double the distance でも同じような意味になる。

820 :名無しさん@英語勉強中 :2022/08/05(金) 04:04:58.88 ID:34wmJKWI0.net
>>818
実質的にOED loves you NOT.
になったわけかも。

821 :名無しさん@英語勉強中 :2022/08/05(金) 04:11:30.82 ID:34wmJKWI0.net
at twice the distance だとatがいらないでしょ。
副詞的目的格だから。それで副詞的目的格
でない書き方をするとすれば
twice at the distance
とするのだろうと思ったわけです。
twiceは副詞なので、the distanceを修飾できない。
だから
at twice the distanceだとおかしいでしょう?
twice at the distanceで副詞句のat the distance
を修飾させるようにするしかない。

822 :名無しさん@英語勉強中 :2022/08/05(金) 04:58:57.96 ID:0jrg/Q/U0.net
at twice the distance
twice the distance
両方とも正しいんだって言ってるだろが。新聞サイトでの英文でも、ちゃんと
at twice the distance の用例が溢れてる。

The new radar will be able to see targets half the size ★at twice the distance★ of today’s SPY-1 radar.
(Military-Aerospace Electronics より)

823 :名無しさん@英語勉強中 :2022/08/05(金) 05:05:44.56 ID:0jrg/Q/U0.net
at twice the distance に living がついてる例文も見せようか?

The -0.096 elasticity implies that ★living at twice the distance★ from the Promenade implies 10 percent lower rent.
(academia.edu っていう、大学の学術論文サイトと思われるサイトより)

824 :名無しさん@英語勉強中 :2022/08/05(金) 05:10:02.23 ID:0jrg/Q/U0.net
The diffuse cloud is thought to have an inner edge
★at 2,000 to 5,000 times the distance★ between the Sun and the Earth.
(NASA のサイトより)

こういうのが腐るほど転がってる。

825 :名無しさん@英語勉強中 :2022/08/05(金) 07:38:57.61 ID:XsGMcY91a.net
What have I done, I wonder, that to me should have been decreed the love of two such women as Marie and that of Stella, also now long dead, to whom alone in the world I told all her tale?

全然意味がわかりません

826 :名無しさん@英語勉強中 :2022/08/05(金) 09:04:08.06 ID:0jrg/Q/U0.net
>>825
H. Rider Haggard という19世紀の作家の文章だな。この作家は、俺の記憶によると、
わざと古風な文体で文章を書いて独特の小説世界を作り出している人だったと思う。
19世紀の作家だから、俺たちにとってはただでさえすでに古風な文体が元になっているけど、
この作家は自分の生きている時代よりもさらに100年も200年も前の古い文体を駆使して
いたようだ。だから、17世紀あたりの文章を真似ていたはずだ。だから俺たちにとっては、
思いっきり難しい。

原文を5つに区切って解説する。

(1) What have I done, I wonder,
(2) that to me should have been decreed
(3) the love of two such women as Marie and that of Stella,
(4) also now long dead,
(5) to whom alone in the world I told all her tale?

(1) の I wonder はもちろん挿入句であり、それを文頭に置いて書き換えることもできる。つまり、
I wonder what I have done that.... と書いても、全体の大体の意味は変わらない。
(もちろん、語気というか雰囲気のようなものは変わる。)

(2) の that 以下の節は、What have I done の後ろにくっついて、「その結果として
こういうことが起こった」というような意味合い。should がついているから、
驚きや意外性を示しているんだろう。

(続く)

827 :名無しさん@英語勉強中 :2022/08/05(金) 09:11:13.02 ID:0jrg/Q/U0.net
>>825
(1) What have I done, I wonder,
(2) that to me should have been decreed
(3) the love of two such women as Marie and that of Stella,
(4) also now long dead,
(5) to whom alone in the world I told all her tale?

(2) から (5) は、すべて that 以下の節となっている。(5) はもちろん、関係詞をつかって
その前の (3) の Marie と Stella (あるいは two such women as Marie and (that of) Stella)
にかかっている。(4) の also now long dead は、たぶん Stella を修飾していると思う。

(2) that to me should have been decreed
(3) the love of two such women as Marie and that of Stella,

上の (2) から (3) を倒置しないで(俺たちにとってわかりやすいように)主語を
前に持ってくる語順に変えると
that the love of two such women as Marie and that of Stella should have been decreed to me
となるわな。

828 :名無しさん@英語勉強中 :2022/08/05(金) 09:16:31.96 ID:0jrg/Q/U0.net
>>825
(1) から (5) を、なるべく直訳的に訳してみる。

(1) What have I done, I wonder,
(2) that to me should have been decreed
(3) the love of two such women as Marie and that of Stella,
(4) also now long dead,
(5) to whom alone in the world I told all her tale?

Marie や Stella のような二人の女性の愛を勝ち得るなんて、私はそんな幸せを
得られるに値するようなどういうことをしたというのだろうか?
Stella もすでにずっと前に死んでしまったし、世の中でその Stella だけに私は
彼女の話をしてあげたのに。

俺はこの英文の解釈については、あまり自信がない。この英文の前に出てくるはずの
文章を何行か読めば、もっと良く事情が呑み込めるはずだと思うけど、検索しても
その文脈が俺には探し出せなかったから、仕方なく文脈を無視して質問者が
提示した1文だけを読んで前後関係を勝手に想像しながら訳してみた。

829 :名無しさん@英語勉強中 :2022/08/05(金) 09:18:55.56 ID:0jrg/Q/U0.net
>>825
おっと、Project Gutenberg 上でなら読めることに気づいた。今からその文脈を読み取ったあと、
回答しなおす。

830 :名無しさん@英語勉強中 :2022/08/05(金) 10:02:36.98 ID:nMp2ODtN0.net
>>825
What have I done, I wonder, that to me should have been decreed the love of two such women as Marie and that of Stella, also now long dead, to whom alone in the world I told all her tale?

マリーとステラに私は愛と呼べることを何かしてあげただろうか?ステラはマリーのことを話したただ一人の女性で、二人ともとうの昔に死んでしまったが。

って感じじゃないかなぁ

831 :名無しさん@英語勉強中 :2022/08/05(金) 10:06:50.55 ID:0jrg/Q/U0.net
>>825
課題文の前に出てくる文脈を読み取った後、訳し直してみる。でも今でも俺はこれを完璧には
理解していないと思う。

(1) What have I done, I wonder,
(2) that to me should have been decreed
(3) the love of two such women as Marie and that of Stella,
(4) also now long dead,
(5) to whom alone in the world I told all her tale?

和訳の試み:
Marie や Stella のような二人の女性の愛を勝ち得るなんて、私はそんな幸せを
得られるに値するようなどういうことをしたというのだろうか?
(Marie だけではなく)Stella もすでにずっと前に死んでしまったし、
Marie の話を打ち明けた相手と言えば Stella だけだったのだが。
==========

つまりこの文章を書いているのは Allan Quatermain という男であり、その男が
最初に愛した相手は Marie という女であり、その女を巡っていろんなことが
起こったようだ。そして Marie が死んだあとに Allan は Stella という女と
愛し合って(おそらくは結婚したと思うんだが)、その新しい相手である Stella に
対してしかこの Marie という昔の恋人について語ったことはなかった、というわけらしい。

(おそらくはそういうことだろうと、俺は前後の脈絡から判断しているけど、
間違っていたらすまん。あまりはっきりとは書かれていないように思う。
きちんと事情を呑み込みたければ、この本を初めとして何冊もの本を
読んで筋を追っかけるしかないと俺は思う。)

832 :名無しさん@英語勉強中 :2022/08/05(金) 10:10:13.41 ID:0jrg/Q/U0.net
Cross-posted with >>830.
>>830 と被った。)

833 :名無しさん@英語勉強中 :2022/08/05(金) 10:13:41.78 ID:Zzq9I/RR0.net
以下の文について質問です。

In any case, this printer is not the one I ordered.
いずれにしても、このプリンタは私が注文したものではありません。


上記文ですが、 anyは否定文ではnot...anyの語順でないといけないため、正しくは以下の文だと考えていますが、皆さんの意見をお聞きしたいです。
以下であればokだと考えてます。
This printer is not the one I ordered in any case.

834 :名無しさん@英語勉強中 :2022/08/05(金) 10:38:36.70 ID:0jrg/Q/U0.net
>>833
(1) 原文: In any case, this printer is not the one I ordered.

なんか難しく考えてるみたいだけど、もしかして、(1) の原文が間違っていると
思ってるの?それはそのままで正しい英文だよ。in any case は、そのあとに否定文が
来ても肯定文が来ても、それとは関係なくそのまま独自に成り立つ phrase なんだよ。

835 :名無しさん@英語勉強中 :2022/08/05(金) 10:46:18.30 ID:0jrg/Q/U0.net
>>833
もう少し正確に解説してみようか。in any case にせよ、any を含むその他のどんな
phrase であれ、その前に否定を表す副詞(not, never など)がつかない限りは、
そのまま独自で独立した意味を持つんだよ。

(1) Not in any case is this printer the one I ordered.
(2) Never in any case is this printer the one I ordered.
(3) In no case is this printer the one I ordered.

上の三つの文は、本質的な意味合いは同じ。つまり「どんな場合であれ、
このプリンターは私の注文したものではない」という意味。

しかし
(4) In any case, this printer is not the one I ordered.
これは、上の (1), (2), (3) とはまったく意味が違う。
「いずれにせよ(さっき言ったような場合であれ、その他のどんな場合においてであれ、
ともかくこのプリンターは俺が注文したもんじゃねえってんだよ。)

836 :名無しさん@英語勉強中 :2022/08/05(金) 10:54:32.48 ID:Zzq9I/RR0.net
>>834
ありがとうございます。
青ロイヤルや他の総合英語系文法書を読んでいたのですが、
anyは
① 形容詞句節に修飾されている場合
② if節の中で使用される場合
を除き、not...anyがの語順が原則であると書いてて、一方で他参考書には先の例文が出てきたので混乱してました。
in any caseは独立したフレーズと考えれば良いのですね。
ありがとうございます。

837 :名無しさん@英語勉強中 (ワッチョイ 250c-dSCr):2022/08/05(金) 11:28:14 ID:5cdDT9PR0.net
以下の文は、中国の防衛予算はアメリカの予算規模に達したかという意味だと思いますが、

Chinese defense spending is in. Has it reached US spending?

is in の意味がわかりません。

838 :Seeker2022 ◆iymFBv30wPKz (ワッチョイ 7922-dSCr):2022/08/05(金) 11:33:33 ID:5075Ex3I0.net
>>825
これは、まず次の部分だけを考えてみよう
the thing(s) which to me should have been decreed the love of two such women
これを理解するためには、まず次の用法を知らないといけない。
「decree 目的語 補語」で「目的語に補語だという決定を下す」(ウィズダム英和辞典)
また、decreeにはordain(天が運命づける)の意味もあることから、この場合は、「decree the things the love of them」で「(神・運命などが)その事を彼女たちの愛(の原因)だという決定を下す」となる。
原文の主語のニュアンスを分かりやすく言えば、「彼女たちからの愛が得られた理由として私に対して天から判断された事柄」である。

ここまでくればあとは簡単で、原文の趣旨は「マリーとステラという2人の女性から愛を得られた理由となるようなことを私はしてきただろうか?二人とも亡くなって多くの時間が経った。ステラは、マリーについて全部話したあげた世界で唯一の人だった。」

839 :名無しさん@英語勉強中 :2022/08/05(金) 11:34:31.65 ID:DzLVkqOh0.net
>>837
中国の防衛予算に関する最新情報が入りました、ってことじゃないの?

840 :名無しさん@英語勉強中 :2022/08/05(金) 11:37:00.52 ID:DzLVkqOh0.net
Our weekend weather report is in! About 2 feet of snow blanketing our beautiful camp!

The CBO's report is in about the potential impacts of the American Health Care Act

841 :名無しさん@英語勉強中 (ワッチョイ d689-G1eK):2022/08/05(金) 12:05:57 ID:0jrg/Q/U0.net
>>837 については、今回の文脈においては
>>839-840 の言う通りだと思うけど、文脈によって意味が少し違ってくるはずだ。
今回の文脈は

2022 Chinese defense spending is in. Has it reached US spending?
★This video explores the details behind the latest defense budget,
as published by China.★ How does their budget compare to the US one?
And what's needed to precisely compare the two? Watch the video to find out.

上のような YouTube video の説明書き(特に★で囲った部分)を見たらわかるように、

Chinese defense spending is in.
= China has just published its lateset defense budget.
というような意味であるらしいことがわかる。

質問者はきちんとその文脈を最初から明らかにすべきだったんだよ。
ただ、あなただけではなくて質問者たちはみな、まさか文脈によって意味が
変わるなんてことは予想してないからいつも文脈なしでいきなり特定の箇所だけを
引用して質問を投げかけてしまうんだろうけど。

842 :名無しさん@英語勉強中 :2022/08/05(金) 13:21:04.54 ID:tMzWl7LM0.net
the love of two such women as Marie and that of Stella

気取りすぎてこの箇所間違ってるよな

これ文字通り読んだら 「メアリーと他一名の愛、そしてステラの愛」になっちまう

843 :名無しさん@英語勉強中 :2022/08/05(金) 16:14:34.58 ID:0jrg/Q/U0.net
>>838
>>「decree 目的語 補語」で「目的語に補語だという決定を下す」(ウィズダム英和辞典)

悪いけど、あなたはウィズダム英和の解説を少し読み違えているんじゃないかな?

   ウィズダムの記述:
   decree A C = A (人) に C であると判決(決定)を下す(ただし、C は形容詞)

と書いてある。補語の部分は名詞であってはならず、形容詞でないといけないのだ。
ウィズダムにはその用例として、次の文を挙げている。

   The court decreed him guilty.

844 :名無しさん@英語勉強中 :2022/08/05(金) 16:24:33.46 ID:0jrg/Q/U0.net
>>825
What have I done, I wonder, that to me should have been decreed
the love of two such women as Marie and that of Stella, also now long dead,
to whom alone in the world I told all her tale?

そうなるとやはり、最初に俺が言った通り、that 節のあとは主語が後ろに追いやられた
形だと解釈せざるを得ないと俺は思う。
to me should have been decreed the love of [the two women]
という形になり、
「(二人の女性)の愛が私に対して決定された(なんて)」
という感じの意味にしかならないように、俺には思える。should が気にかかるが、
やはり俺には意外性とか驚きを表す「〜だなんて」というような意味合いにしか
考えれない。

そうなるとその直前の that は何なんだということになるが、これは昔の英語では
理由を表す that つまり because に近い意味合いを持つ that として大いに使われたらしいが、
それのような気がする。

“When we are born, we cry that we are come to this great stage of fools.”

上の文は、Shakespeare の King Lear に出てくる有名な台詞だが、この that は
because に近い意味合いで使われている。
「俺たちは生まれたばかりのとき、馬鹿どもが演じるこの大きな舞台にたどりついて
(たどりついたので)泣くのだ」(馬鹿に囲まれているからこそ、赤ん坊は泣くのだ)
という意味合いだ。

今回の英文に出てくる that も、もしかしてそういう意味合いじゃないかという気がするんだが、
違うだろうか?こういう that は、I am happy that SV とか I am sad that S V における
that として現代でも少しは生き残っている。

845 :名無しさん@英語勉強中 :2022/08/05(金) 16:28:32.43 ID:0jrg/Q/U0.net
>>825
ついついいろいろと考えているうちに、論旨がずれてしまった。この場合の
that は because というよりも、結果を表す that に近いような気がする。

What have I done that S should [do something].
というわけで、〜という結果を生み出すなんて、いったいどんな(よい)ことを
私はしたというのだろうか?
(私が二人の女性の愛を勝ち取れるような善行を行ったなんて考えられないじゃんか)

という感じではないかという気がするんだが、ちゃいまっかいな?

846 :名無しさん@英語勉強中 :2022/08/05(金) 16:33:55.69 ID:0jrg/Q/U0.net
現代英語では "so (adjective, adverb) that S V" という形で、「あまりに〜なので、〜となった」
というふうに、that 以下の節が結果を表す。そして堅い文章の中では、so が省略される
ことが多い、と現代英語の辞書にも書いてある。

その、so がなくて "that S V" だけの形で、その前の行いの結果として生じた事柄を
示しているんではないかと思うんだが、これについてはしっかりした証拠が
見つからない。The Oxford English Dictionary にてそういう用例と定義文とを
探したいところだが、すでに言ったように今の俺は OED Online にはアクセスできない。

847 :名無しさん@英語勉強中 :2022/08/05(金) 16:48:38.97 ID:0jrg/Q/U0.net
>>825
原文: What have I done, I wonder, ★that★ to me should have been decreed
the love of two such women as Marie and that of Stella, also now long dead,
to whom alone in the world I told all her tale?

どうやら結論が出たような気がする。OED を引かないと埒が明かないと思ったが、
手元にある The Shorter Oxford English Dictionary (SOED), Sixth Edition, 2007 にも
きちんとこういう場合の that が紹介されていた。

SOED の解説に基づいて結論を分かりやすく言うと、この原文に出てくる that は
that S should [do something]
という形であり、目的や意図を表す。だから「〜が〜になるように仕向けるために、
私は何をしたというのだろうか?」(二人の愛が私向けに与えられる(命令される)
ように仕向けることになるようなどういう善行を私はしたというのだろうか?)」
という意味になる。

SOED に書いてあるこのような that の、古い(archaic な)使い方についての
解説と英文を、今からこの下に長々と引用する。すべて大切なので、端折らずに
すべて引用する。

848 :名無しさん@英語勉強中 :2022/08/05(金) 16:57:38.70 ID:tMzWl7LM0.net
このthatの用法は
中級以上の英和辞書に載ってるレベルのことだから
大して難しくもない(受験では出ないがな)

849 :名無しさん@英語勉強中 :2022/08/05(金) 17:01:41.28 ID:0jrg/Q/U0.net
SOED の中の、conjunction としての that

3 a ------ Introducing a clause expressing purpose, aim, or intention: with simple subjunctive (archaic),
or with may, might, should, (rarely) shall. OE (これはつまり、この用法が Old English つまり
古英語に由来していることを示す)

3 b
Introducing an exclamatory clause expressing desire or longing: with verb in (past)
subjunctive. ME (つまりこれは Middle English での用法に由来)

3 c
Introducing a clause expressing a hypothetical desired result: with verb
in subjunctive or its equivalent. E17. (つまりこの用法が 17 世紀の英語だということ)

上記の 3 a についての用例
A. J. Christie Christ
... had prayed ★that★ Peter's faith ●should● not fail
(この上の例文の that ... should... という呼応関係に注目。今回の質問者が挙げている原文の場合と同じ。)

A.S. Tessimond
Losing battles Fought ★that★ the sons of sons may win.

上記 3 b の用例
W. Cowper
Oh ★that★ those lips had language!

G.W. Dassent
I would give all my goods ★that★ it had never happened.

これで、今回の「目的・意図」を示す conjunction としての that の古風な使い方についての
SOED による解説と用例の紹介を終わる。すべてを手でタイプしたので、ミスタイプ
したところもあるかもしれないが、許してくれ。

850 :名無しさん@英語勉強中 :2022/08/05(金) 17:05:01.21 ID:0jrg/Q/U0.net
>>848
後出しでならどんな馬鹿でもそういうことは言える。問題は、誰も正解をきちんと
細かく説明していないとき、ああでもなくこうでもないと考えて、そのうえで相手に
わかるように解説することだ。

後出しで、1行か2行で「そんなもん、小さな辞書にも載ってる」などというのは、
負け犬の捨て台詞でしかない。

悔しいなら、最初からきちんと己(おのれ)で解説してみさらせ。
できねえんなら、最後まで黙りさらせ。

851 :😉三年英太郎🌈 ◆3CZBjOt3.Y (ワッチョイW dae9-QBiA):2022/08/05(金) 18:21:18 ID:3KMxgbWa0.net
長文先生規制解けて絶好調やな😁
よほど人恋しかったんだろう

852 :名無しさん@英語勉強中 (ワッチョイ faf0-1wwR):2022/08/05(金) 18:44:06 ID:tMzWl7LM0.net
ちなみにWeblioは研究社の新英和中辞典をソースにしてる
これ見ればだいたいのことがわかる
3500円相当のものがタダで使えるスゴイ

853 :Seeker2022 :2022/08/05(金) 20:19:34.74 ID:5075Ex3I0.net
>>843
おいおい、俺のウィズダム辞書には「ただし、C は形容詞」なんて書いてないんだが。(自分のは最新版)  Cには、形容詞以外にも名詞の働きをするものが来れることは知っているよね?

辞書を引くまでもなく、decreeOCのCに名詞の働きをするものが来れるのは、小説「The Lost Letter: A Novel」著者: Jillian Cantorの中で「the Germans would decree him a criminal, too, simply for destroying their instructions. 」と使っていることからも明らかである。

843が言っているように例の文を「decree 目的語」の第三文型の受動態であると解釈すると、What との関連性が説明できなくなるばかりでなく、なぜdecreeという動詞を使うのか説明ができない。
「decree the love of them」の受動態だとするのが>>843の解釈であるが、いったいどう訳すというのだろう?「彼女たちの愛を命じる」の受動態で、「彼女たちの愛が命じられた」になってしまう。そんなわけのわからない意味になるはずがない。

ここは、decree OCで、解釈するのがもっとも理にかなっているのである。

854 :Seeker2022 :2022/08/05(金) 20:24:59.27 ID:5075Ex3I0.net
小説「The Lost Letter: A Novel」著者: Jillian Cantorの中で「the Germans would decree him a criminal, too, simply for destroying their instructions. 」と使っていることから、
>>843の「補語の部分は名詞であってはならず、形容詞でないといけないのだ」は、間違いである。

それにしても、>>843の独善的な持論は、目を見張るものがある ^^;

855 :Seeker2022 :2022/08/05(金) 21:04:24.60 ID:5075Ex3I0.net
>>844は、まず、「主語 is p.p.」の受動態で、主語が過去分詞の後ろに倒置される例を挙げて欲しい。それが見つからない以上は、>>844を信じるわけには行かない。
それと、「decree the love of them」の受動態、「The love of them was decreed.」の直訳を挙げて欲しい。

この2つの事柄への納得のいく回答が得られないうちは、>>844の説を検討することはない。

856 :Seeker2022 :2022/08/05(金) 23:46:12.84 ID:5075Ex3I0.net
「The things were decreed the love of them to me.」は、第五文型の受動態として解釈すると次のように自然に理解できる。

「(天・運命・神などによって) その事柄が私に対する彼女たちの愛だと判断された」→「その事柄が彼女たちの私への愛を運命づけた」というニュアンス。

decreeは名詞で「天の意志」「天命・運命」という意味を持つことは辞書を見れば分かると思う。動詞decreeにも「天の意志によって運命づける」という意味があることは下記リンク先で「decree=ordain」となっていることからも明らかである。
https://www.dictionary.com/browse/decree

>>825の英文は、「The things were decreed the love of them to me.」のthe thingsが先行詞になって、「The things that were decreed the love of them to me」となったものと同様である。

857 :名無しさん@英語勉強中 :2022/08/06(土) 01:33:11.44 ID:bv3eHCT+0.net
>>822
ありがとう。
The plane can fly at twice the speed of sound.
その飛行機は音速の2倍の速さで飛ぶことができる
This house is twice the size of that one.
この家はあの家の2倍の大きさだ

というのもあるものな。
なぜtwiceが形容詞じゃなくて副詞なのに
twice the size of なんだろう?

858 :名無しさん@英語勉強中 :2022/08/06(土) 03:59:23.26 ID:0rnFVnV70.net
>>855 これとかどうよ
https://i.imgur.com/I5v9h0X.jpg
Fifty Years In The Church Of Rome
By Charles Chiniquy (1809-1899), a Canadian Catholic priest

859 :名無しさん@英語勉強中 :2022/08/06(土) 04:51:18.36 ID:I1mvSdaT0.net


860 :名無しさん@英語勉強中 :2022/08/06(土) 05:25:18.22 ID:7GuugzxD0.net


861 :名無しさん@英語勉強中 :2022/08/06(土) 05:46:58.35 ID:opwCaqZB0.net


862 :名無しさん@英語勉強中 :2022/08/06(土) 06:20:48.25 ID:Z6iYGnu60.net
>>825 の英文・・・どういう小説のどういう文脈のなかの文章なのか?
「今は亡き二人の女性にこれといったことは何もしていないのに、愛されて
 しまった勘違い男の『モテ自慢』を、自分で懐かしんで分析してるのかい」

まあ、この世にはいろいろな状況があるし、いろいろな男がいるから、それでも
構わないんだが・・・・・・一般の読者がぐいぐい引き込まれる設定じゃないな。

むしろ、>>830 さんの解釈のような流れの方が・・・・・自然でいいなあ・・・
文法的に、そうはいかないのかな・・・・・・・・・・

863 :名無しさん@英語勉強中 :2022/08/06(土) 06:53:12.76 ID:0rnFVnV70.net
前妻だよ
一人目は結婚した次の日
二人目は一年足らずで死んだ

864 :名無しさん@英語勉強中 :2022/08/06(土) 07:03:24.24 ID:Z6iYGnu60.net
「二人から受けた愛に対して、十分に応えてあげただろうか」という思い、
悔恨の情なら、自然な流れもあるか

865 :名無しさん@英語勉強中 :2022/08/06(土) 07:09:00.78 ID:ZQ2Un1020.net
>>808
ちなみに
I live twice the distance from the university as she does. の
as she does の as って「接続詞の as」 でしょうか?
よろしくお願いします

866 :名無しさん@英語勉強中 :2022/08/06(土) 07:26:55.70 ID:0rnFVnV70.net
文字通りの意味にしがみつくなよ
修辞ってもんが分からないかい?

867 :名無しさん@英語勉強中 :2022/08/06(土) 07:29:02.32 ID:0rnFVnV70.net
>>865
それ出典なに?
色々混ざっちゃって妙な文
(言いたいことはわかるけど)

868 :名無しさん@英語勉強中 :2022/08/06(土) 07:34:08.40 ID:Z6iYGnu60.net
>>866

もっと具体的に説明してくれ

869 :Seeker2022 ◆iymFBv30wPKz (ワッチョイ 5922-woMg):2022/08/06(土) 09:48:14 ID:nkIuf8fq0.net
>>858
なるほど。倒置説は、検討する価値があるね。ちょっとあとでじっくり検討してみるよ。

870 :名無しさん@英語勉強中 :2022/08/06(土) 10:02:43.20 ID:Z6iYGnu60.net
>>862 以下の書き込み、出典の英文を読まずに、ここのレスだけに
反応して書いてしまった。・・・というわけで、取り消し。・・・スマン

今、元の英文を読んでいるところ。・・・なかなか面白いかも。

871 :名無しさん@英語勉強中 :2022/08/06(土) 10:19:09.40 ID:dCQzzQAiH.net


872 :名無しさん@英語勉強中 :2022/08/06(土) 10:19:27.83 ID:dCQzzQAiH.net


873 :名無しさん@英語勉強中 :2022/08/06(土) 10:19:40.52 ID:iYw8ivsK0.net


874 :Seeker2022 :2022/08/06(土) 10:32:57.88 ID:nkIuf8fq0.net
>>858
858は意図のthatと主張しているようだが、訳すと「自分は何をしてきただろうか? 彼女たちの愛を自分と運命づけた意図で?」というふうになって不自然に感じられる。時制も過去時制(should have p.p.)が使われていて、とても意図とは思えない。
この文の真意は、「自分は何をしてきただろうか? 彼女たちの自分への愛を運命づけた証となる何かを?」だと思う。そこには、意図の意味はないと思われる。

そこで、このthat節が接続詞だとした場合、疑問詞whatとthat節が同格関係にあるとするのが良いのではないかと思うのだが、それについてはどう考える?

875 :名無しさん@英語勉強中 :2022/08/06(土) 12:19:58.71 ID:U0AoXZA00.net
通りすがりにちらっと見ただけで、あまり深く考えてはいないけど、
この that は関係代名詞で、冒頭の What という疑問詞を先行詞にしてるんじゃないの?

一応、一つの考え方として提示しておきます。 (´·ω·`)

疑問詞が先行詞になる例はあまり多くはないけど、一部の文法書には載っていますし、
まれに現代の英文でも出会うことがあります。

文法書の例文
・Who that really knows him can believe him?
ほんとうに彼を知っている者は、だれが彼を信用することができようぞ
・Who is the girl that is playing in the garden?
庭で遊んでいる少女はだれか

問題の文章では、
「 〜 の愛を運命づけられるような一体どんなことを私はしただろうか」

「 〜 の愛を運命づけられるようなことは私はした覚えがない」
( 〜 に愛される(に値する)ようなことは私はしていないのに)
という感じ。

876 :名無しさん@英語勉強中 :2022/08/06(土) 12:35:09.53 ID:P876X+b50.net
というかその文章そのままググれば何もかも載ってるからw

877 :名無しさん@英語勉強中 :2022/08/06(土) 13:25:25.81 ID:U0AoXZA00.net
参考までに。

ネットで検索して、「疑問詞の what + それを先行詞とする関係代名詞の that」のパターンのわかりやすそうなものを掲示しておきます。 (´·ω·`)

1. what have i doneの意味・使い方・読み方 | Weblio英和辞書
What have I done that I should deserve such misery?
何の因果でこの憂き目 - 斎藤和英大辞典

2. ”What Have I Done That Made You Go Away” by Moon Mullican
2019/10/24 — What Have I Done That Made You Go Away; 9.I'll Sail My Ship Alone; 10.You Don't Have To Be A Baby To Cry; 11.Sweeter Than The Flowers; 12.

3. 'What have I done that requires remorse?' | CBC.ca
'What have I done that requires remorse?' 10 months ago; News; Duration 0:40. Othman (Adam) Hamdan responds to concerns that he is a threat to Canadians and ...

4. Harry Belafonte on Nate Parker: 'What Has He Done That ...
2016/08/24 — Harry Belafonte on Nate Parker: 'What Has He Done That Requires This Kind of Animus?' By. Yesha Callahan · 8/24/16 7:23AM.

878 :名無しさん@英語勉強中 :2022/08/06(土) 16:03:01.99 ID:WJ0dQOEY0.net


879 :825 :2022/08/06(土) 21:16:15.51 ID:u/hvom0Aa.net
詳しい解説めっちゃ助かりました
ありがとうございます

880 :名無しさん@英語勉強中 :2022/08/06(土) 21:33:29.37 ID:ZQ2Un1020.net
The rent of apartments in Tokyo is only one quarter as high as that in New York.
「東京のアパートの家賃はニューヨークのアパートの家賃のほんの4分の1ほどだ」
を英訳する場合、as ... as の「...」の部分は high でいいでしょうか?

881 :名無しさん@英語勉強中 (ワッチョイ d3c7-9Xv3):2022/08/06(土) 22:03:08 ID:0QFaXI7F0.net
highでよいですが、
The rent of apartments in Tokyo is only a quarter in New York
のほうが自然だと俺は思う

882 :Seeker2022 :2022/08/07(日) 00:03:10.21 ID:8QtlDNsj0.net
>>825の最終見解(まとめ) これまでの私の書き込みはすべて白紙に戻します。 >>879 惑わせてしまってごめんね 

「What have I done that to me should have been decreed the love of them?」と文章を簡略化してみる。
「主語 was given to me」は、>>858により、「to me was given 主語」という倒置が可能なので、上記文章は、「What have I done that the love of them should have been decreed to me?」と書き換えることができる。

●このthatは接続詞である。意外性を表すshouldと結びついている。
●「decree the love of them to me」で「(天意が)彼女たちの愛を私に対して定める」(彼女たちの愛を私に運命づける)
●このshouldは意外性を表すshouldで「~なんて」と訳す。例 I was surprised that you should have so spoken! (まさか君がそんなことを言うなんて驚いた!)

従って、上記英文は次のように訳すことができる。
「彼女たちの愛が私に運命づけられたなんて、いったいどういう因果なのだろうか?」

883 :名無しさん@英語勉強中 :2022/08/07(日) 05:23:14.94 ID:CzBPNrUc0.net
>>877
1. what have i doneの意味・使い方・読み方 | Weblio英和辞書
What have I done that I should deserve such misery?
何の因果でこの憂き目 - 斎藤和英大辞典


thatの先行詞ってなに?

884 :名無しさん@英語勉強中 :2022/08/07(日) 06:57:18.55 ID:CPIW7cXS0.net
>>880
それでよいと思う。代案として、次のような言い回しもあるらしい。
Apartments are rented for only a quarter of what they charge in New York.
少しでも短くしようと努めてみた。

885 :名無しさん@英語勉強中 :2022/08/07(日) 07:02:03.78 ID:CPIW7cXS0.net
>>881
>>The rent of apartments in Tokyo is only a quarter in New York

回答者同士でコメントし合うのはなるべく避けないと危険なんだが、あえて
コメントするので、どうか気を悪くしないでほしい。

... is a quarter in New York.
という書き方をすると「NY の4分の1である」という意味にはならず、
「NY では4分の1である」という意味にしかならないと思う。
だから「東京では、NY では 4分の1である」という変な意味になってしまう。

そうなるとやっぱり、
... is a quarter of the rates of New York
... is a quarter of what they charge in New York
... is a quarter of the rates prevalent in New York
... is a quarter of what you typically see in New York
みたいな書き方をしないといけなくなるような気がするんだけど、違うかな?

886 :名無しさん@英語勉強中 :2022/08/07(日) 07:15:10.17 ID:CPIW7cXS0.net
>>857
文法用語とか文法解説なんて、暇を持て余してる文法家が後付けで適当にこじつけた
解説でしかない。大事なのは、英語ネイティブがごく普通に至る所で頻繁に使っている
言い回しをそのまま真似るだけだよ。

Twice three is six.
Two times four is eight.

887 :名無しさん@英語勉強中 :2022/08/07(日) 07:21:30.96 ID:CPIW7cXS0.net
>>853
俺の Wisdom 英和、第4版、特装版、2019 年1月10日、第1刷発行
には、きちんと decree A C
A(人)にCであると判決(決定)を下す
(Cは形容詞)

とはっきり書いてある。

俺のことを独善的とあんたは言ったが、俺はあんたこそ独善的だと今までずっと思ってきた。
デタラメを言い続けてきたもんな。特に英訳で。

888 :名無しさん@英語勉強中 :2022/08/07(日) 07:29:08.13 ID:CPIW7cXS0.net
>>883
その that は関係代名詞じゃないって、いろんな人がすでにこのスレで言いつくしてるだろ?
>>882 のコメントと >>849 に示した辞書での解説を総合すれば納得するよ。

889 :名無しさん@英語勉強中 :2022/08/07(日) 07:40:04.31 ID:CPIW7cXS0.net
>>880
>>The rent of apartments in Tokyo is only one quarter as high as ★that in New York★.

>>881
>>The rent of apartments in Tokyo is only ★a quarter in New York★

勘違いしやすいので、説明を補足しよう・>>880 の that in New York は
確かに「NY のそれ(つまり NY の家賃)ということになるけど、
>>881 でも同じように「NY の 1/4」になるかというと、そうはならない。
... is a quarter in New York. となっているから、「NY では 1/4 である」という
意味にしかならない。だからこそ、"a quarter of *** in NY" だとかいうふうに
工夫する必要がある。

890 :名無しさん@英語勉強中 :2022/08/07(日) 08:04:43.68 ID:CzBPNrUc0.net
>>888
あんたに聞いてないよ( ^ω^)

891 :名無しさん@英語勉強中 :2022/08/07(日) 08:38:51.59 ID:CPIW7cXS0.net
>>890
俺だって、あんたが俺の言ってることを理解できる奴だとは思ってないよ( ^ω^)

892 :名無しさん@英語勉強中 :2022/08/07(日) 08:41:09.71 ID:CPIW7cXS0.net
ついつい豚に対して人間の言葉で語りかけてしまった。すまん。
俺はブタ語が使えないもんで。

893 :名無しさん@英語勉強中 :2022/08/07(日) 09:44:41.95 ID:CzBPNrUc0.net
訊いてないってのに

894 :名無しさん@英語勉強中 :2022/08/07(日) 11:13:48.17 ID:CPIW7cXS0.net
>>858 に出てくるような、have been p.p. の後に主語が出てくるのは、19世紀以前には
よくあったようだな。俺も、古い文献をときどき読むが、しょっちゅうそういうのが
出てきて最初はてこずった。

(1) ... of knowledge should dwell in the Son , it follows that
★to Him must have been given the key of all knowledge★ , whether it be visible or invisible .
(1857 年の文献)

(2) ... consequently ★to him must have been given the decree for the building of Jerusalem★.
(これも、19世紀半ばの文献)

(3) Had he reached that battlefield in time to have made one charge,
★to him would have been given the credit of snatching victory out of the very jaws of defeat
of saving that army★, and ●to him would have been given the crown●.
(1880年代)

これ以外にも、根気よく探せばいくらでも見つかる。

895 :877 :2022/08/07(日) 11:53:17.06 ID:xnSBBS6F0.net
>>883
なるほど。
そう言われてみれば、1.の英文の that は関係代名詞と解釈するのは難しいね。 (´·ω·`)

先行詞と I should deserve such misery を組み合わせて元の文章が出来なければ
ならないけど、それが出来ない。
I should deserve such misery で、主語も目的語もある欠けたところのない完全な
文になっている。

2.以下の英文は that を主格の関係代名詞と考えて文法的に成立するけど。

ということは、1.の英文は根拠として提示するには不適切であったか。

しかし、今問題としている文に関しては、「疑問詞の what + それを先行詞とする
関係代名詞の that」のパターンであるとの解釈は依然有効です。

896 :名無しさん@英語勉強中 :2022/08/07(日) 11:55:49.15 ID:CPIW7cXS0.net
>>858 に似た用例。どうも "has been given (あるいは 助動詞 + have been given" + 主語"
の用例が多くて、given 以外の動詞の過去分詞の出てくる例が俺にはなかなか見つけ出せない。

(4) He is Lord of all, for ★to Him has been given "a name that is above every name."★

(5) To him has been given a name under Heaven and among men by which we can all be saved.

キリスト教関係が目立っていて、それ以外の文脈の中での例文がなかなか見つからん。

897 :Seeker2022 :2022/08/07(日) 14:36:55.66 ID:8QtlDNsj0.net
>>887
じゃあ、Wisdomが間違ってるんだよ。
小説「The Lost Letter: A Novel」著者: Jillian Cantorの中で「the Germans would decree him a criminal, too, simply for destroying their instructions. 」と、補語に名詞が来てるのだから。

そういうあんたも、that節は意図を指すとかデタラメを言ってたくせに。^^

>>882の正しい解釈「意外性のshould」「自然な和訳」にたどり着いたのは、俺のほうが先なのだということを忘れないように。

898 :Seeker2022 :2022/08/07(日) 14:41:05.49 ID:8QtlDNsj0.net
>>895
今回の英文のthatは接続詞だよ。
What have I done that I should deserve such misery?
何の因果でこの憂き目 - 斎藤和英大辞典

このパターンに該当するもの。shouldは「意外性を表す」もので、「~とは」「~なんて」という意味。
I was surprised that you should have so spoken! (まさか君がそんなことを言うなんて驚いた!)
のthat節と同じ働き

参考
https://ejje.weblio.jp/content/What+have+I+done+that

899 :Seeker2022 :2022/08/07(日) 14:48:35.79 ID:8QtlDNsj0.net
>>887ってもしかして、別の人?
なら、教えてやる。 今回の英語とは無関係だけど、「decree OC」 の C には、名詞も来れる。覚えておくように。

証拠
小説「The Lost Letter: A Novel」著者: Jillian Cantorの中で「the Germans would decree him a criminal, too, simply for destroying their instructions. 」と、補語に名詞が来てる


※注意: 今回質問にあった「to me should have decreed 主語」は、第3文型の受動態である。

900 :Seeker2022 :2022/08/07(日) 14:56:05.97 ID:8QtlDNsj0.net
>>887
私はひとつ英訳のミスをしたけど、それは、日本語の意味を間違って捉えていたからで、その日本語の意味の間違いを正確に英語に反映させただけ。
他に英作のミスはひとつもしていない。もしあるなら、出してみなよ。論破してあげるから。

901 :Seeker2022 :2022/08/07(日) 15:06:22.44 ID:8QtlDNsj0.net
自分はジーニアスは本で持ってるけど、ウィズダムはアプリ版。アプリ版には、「Cは形容詞」という表現はないのだ。間違いだとわかって削除されたのだろうか?

902 :Seeker2022 :2022/08/07(日) 15:21:53.81 ID:8QtlDNsj0.net
今調べたところは、Wisdom英和辞典は、アプリ版のほうが古かった。ということは、新しい本版に「Cは形容詞」という間違った情報が追加されているということか・・・
現に小説家がCに名詞を持ってきてるのだから、「Cは形容詞」と限定するのはどうかと思う。確率的には形容詞のほうが多いとしても。

903 :名無しさん@英語勉強中 :2022/08/07(日) 15:47:56.18 ID:CPIW7cXS0.net
(1) What have I done that I should deserve such misery?
↑この上の that は、意図や目的を示す。should は意外性ではなく、
that 以下の、本来なら仮定法現在を使っていたところを、のちにイギリス英語で
should を使うようになっただけ。これについては、俺が示した SOED の長い記述を
見てもわかる。

(2) I was surprised that you should have so spoken!
これは、意外性を示す should だ。

(1), (2) は根本的に意味合いや性質の違う言葉が that の前に来ている。一方は
What have I done? で、もう一つは I was surprised だ。I was surprised なんだから、その
あとは意外性を示すに決まってる。しかし What have I done...? は別だ。

Seeker の示した英訳の間違い(というか不自然さ)については、
without の含まれるたくさんの英訳例(不自然な英訳例)をたどればわかる。
それに対して、俺だけじゃなくて他の人たちも何度もダメ押ししたのに、
あなたは屁理屈を言い続けておしまいだった。

俺はこれについてももう何も言うことがないから、
一人で勝手にこじつけの言い逃れをし続けたらいいよ。俺は黙るけど、
あんたに論破されて黙るんじゃないからな。くだらないことを書くのが嫌なんだ。
少しは見込みのあるやつに物を言うなら丁寧に論議を続けるが、
どうせあんたは言い逃れを続けるだけだろ?それが正しいって
思い続けるんだろ?それならそれで仕方がない。思い込んだ方が勝ちなんだ。

904 :名無しさん@英語勉強中 :2022/08/07(日) 15:51:04.31 ID:CPIW7cXS0.net
それから、三年馬鹿太郎も Seeker と同じく見込みのない奴だと昔に気づいたから、
ずっと「あぼーん」しているからあいつの書いてることはいっさい読んでいない。
同じように、これからは Seeker の言葉も NG 登録して完全無視するので、
よろしく。俺が黙り続けているからと言って、あんたや馬鹿太郎に論破されている
わけじゃなく、単に無視している(最初から読んでいない)だけ。

905 :名無しさん@英語勉強中 :2022/08/07(日) 16:11:23.87 ID:JT+KJhty0.net
三単現のsを忘れたり
hasにするところをhaveにするうっかりがよくあります。
アホかと自分で思うのですが
改善しないのです
同じ悩みされてる方はいますでしょうか?

906 :名無しさん@英語勉強中 :2022/08/07(日) 16:11:56.21 ID:ThbJL18ea.net
No sooner a child is out of its cradle than is it taught that it should not do certain things because it would invite laughter. The sooner this mistaken idea is cleared up the faster will the problem of foreign language learning be solved.

なぜno soonerの直後が倒置されるのではなくて, 呼応先のthan以下で倒置が起きているのでしょうか? また, the soonerの後ろでは倒置が起こっていないのに, なぜthe fasterの後ろでは倒置が起きているのでしょうか?

No soonerとthe soonerが文を超えて呼応しているから, 並びを同じようにしてみたということでしょうか。

907 :名無しさん@英語勉強中 :2022/08/07(日) 17:44:34.36 ID:CPIW7cXS0.net
>>906
(1) No sooner a child is out of its cradle
than is it taught that it should not do certain things because it would invite laughter.
>>なぜno soonerの直後が倒置されるのではなくて, 呼応先のthan以下で倒置が起きているのでしょうか?

その方が語呂がいいと著者は感じたんだろうな。

(1a) 原文のままの場合:(強く発音するところを大文字で表記する)
No SOOner a CHILD is OUT of its CRAdle
(こうすると、強いところと弱いところが交互に現れる。強い音が二回も重なるところがない)
THAN is it TAUGHT that it should NOT do CERTAIN THINGS beCAUSE it would inVITE LAUGHTER.

(1b) 別の語順にした場合:
No SOOner is a CHILD OUT of its CRAdle
(こうすると、CHILD OUT の部分で強い音が二回も続くから、気持ち悪い)
than it is taught that it should not do certain things because it would invite laughter.

このように、弱い音が2回続いてもあまり気持ちが悪いわけではないが、
強い音は目立つので、それが二回も続くよりは、強い音と弱い音とが交互に現れた方が
気持ちいいと感じる人が多いと思う。)

(2) The sooner this mistaken idea is cleared up the faster will the problem of foreign language learning be solved.
>>また, the soonerの後ろでは倒置が起こっていないのに, なぜthe fasterの後ろでは倒置が起きているのでしょうか?

これについては、俺にはわからん。語呂の良さの点では、俺には同じように感じられる。

908 :名無しさん@英語勉強中 :2022/08/07(日) 17:50:26.93 ID:CPIW7cXS0.net
>>906
文章の美しさにこだわる作家は、自分で書いた文章を必ず一人で朗読するらしい。
少なくとも、20世紀前半に活躍したイギリスの作家である Virginia Woolf は
そのように書いている。朗読してみて、できるだけ語呂のよい、滑らかな美しい
文章に書き換えていくのだ。

ただし日本人がこれを朗読してみても、ある程度は正確な発音で適切なイントネーション
で発音しない限り、それを英語ネイティブの読者が読んだときに美しく滑らかで語呂が
よく感じられるかどうかはわからない。

ただ、俺たち nonnatives でも、ある程度は
想像できるだろうなとは思う。だからこそよく「ともかく自分で朗読してみたら、
その文章がどのように歯切れがよいか、あるいは悪いか、どれくらい美しいか、
どういうリズムを持っているかが感じられる」と指導者たちは言うよね。

909 :名無しさん@英語勉強中 :2022/08/07(日) 18:00:55.83 ID:gXFSri/x0.net
>>908
大江健三郎もそんなことを歩きながらすると言ってたよ

910 :名無しさん@英語勉強中 (ワッチョイW 338a-0DAU):2022/08/07(日) 19:44:42 ID:qA1/qPwO0.net
へえーーー、俺は>>830なんだけど、まさかこんなこと>>858があるとは…全く知らなかったし考えもしなかった
ちなみに俺は関係代名詞thatで中がSVOCだと思って訳した

ただ、用例もgiveが多いみたいだけど、そもそも意味も文型も違うdecreeで同じようにやっていいのか?というのはあるよね

911 :名無しさん@英語勉強中 (ワッチョイW 338a-0DAU):2022/08/07(日) 20:04:45 ID:qA1/qPwO0.net
>>905
本人のレベルとか学習期間によるからそれだけだとなんともなぁ

912 :名無しさん@英語勉強中 :2022/08/07(日) 20:27:07.29 ID:DYMO4cM+0.net
>>907>>908

ありがとうございます!

913 :😉三年英太郎🌈 :2022/08/07(日) 21:37:26.08 ID:g+dCskcE0.net
長文先生はNG登録なんかしてないのになんで嘘つくの🤔
自分が間違えてたくせに、なんでおれがバカってことなってんの🤔

フランス留学&仏検1級取ったと豪語してたくせに、表題のle/laが省略されることも知らなかったの誰?

印欧語の比較言語学をやってるといったのに、ゲルマン語の否定語の変遷さえ知らなかったの誰?

おれが正してあげたのに、なんで恨んでるの🤔

914 :Seeker2022 :2022/08/07(日) 22:09:03.32 ID:8QtlDNsj0.net
>>904
今回のWhat have I done~で、最終的に完全な正解を出せたのは、あんたじゃなく私のほうなんだが。^^
過去に論破されたからって、逆恨みしないでね

915 :名無しさん@英語勉強中 :2022/08/07(日) 22:11:05.66 ID:nWucgvcsH.net


916 :名無しさん@英語勉強中 :2022/08/07(日) 22:11:36.74 ID:9PmkX/aC0.net


917 :Seeker2022 :2022/08/07(日) 22:18:19.20 ID:8QtlDNsj0.net
>>895
>>910
「that to me ~」と「to me」が前に出たことが、倒置のサインだと考えて良いと思う。
また、この場合の「decree」は、「decree 目的語 to me」で「目的語を私に定める」で十分意味が通る。
「decree him a criminal」の第五文型では、「him=criminal」 が成立するが、「what = the love」は無理がある。

よって、>>882の解釈がこの問題の完全正解で間違いないです。

918 :名無しさん@英語勉強中 :2022/08/07(日) 22:19:50.07 ID:6y0b3WCn0.net
What have I done?ってビートルズの I'm a loser に出てくるな
"What have I done to deserve such a fate?"
deserve が賞罰にかかわりなく使えることや fate が destiny に対して
悪い運命であることが学べる

919 :名無しさん@英語勉強中 :2022/08/08(月) 01:35:44.46 ID:D+tITzmiH.net


920 :名無しさん@英語勉強中 :2022/08/08(月) 01:35:57.57 ID:uUqrOoLTH.net


921 :名無しさん@英語勉強中 :2022/08/08(月) 01:36:11.11 ID:uUqrOoLTH.net


922 :名無しさん@英語勉強中 :2022/08/08(月) 01:36:23.11 ID:iZ9xVFS3H.net


923 :名無しさん@英語勉強中 :2022/08/08(月) 01:36:38.74 ID:uUqrOoLTH.net


924 :名無しさん@英語勉強中 :2022/08/08(月) 01:36:58.47 ID:JAf7iYDm0.net


925 :名無しさん@英語勉強中 :2022/08/08(月) 03:23:36.97 ID:1rwido1F0.net
「Whether it be Immortality or the Genie's Lamp, The Keepers are the ones to contain and hide them from pursuant parties.」
「不死であれ、魔神のランプであろうと、キーパーズは…」

「The Keepers are the ones to contain and hide them from pursuant parties」の部分がかなり苦戦しています。

@「The Keepers are the "ones" to contain 」の「ones」を「収容すべきものを」で果たしていいのか迷っています。

A「hide them from pursuant parties」この部分は色々と意味を調べたんですがよく分かりません。
「hide them from」は「〇〇から隠す/隠し通す」という部分はまだ分かるのですが…。

問題の「pursuant parties」はGoogle先生で調べてもヒットなし。
みらい翻訳先生の翻訳だと「儀式集団/団体」のような変換があり、なぜそこに行き着いてたのかも分からず、
小一時間苦戦している状態です。

どなたか教えてください。

926 :名無しさん@英語勉強中 :2022/08/08(月) 04:01:13.48 ID:7JU4jvqj0.net
>>925
> ①「The Keepers are the "ones" to contain 」の「ones」を「収容すべきものを」で果たしていいのか迷っています。

The Keepers = the ones

The Keepers (= the ones) というのは、「収容すべきもの」という容器というよりは、ゴールキーパー、ゲートキーパーのような、抑える・守る役職の人たちだろうと思われる


> ②「hide them from pursuant parties」この部分は色々と意味を調べたんですがよく分かりません。

不老不死であれ精霊のランプであれ、キーパーたちが抑え込み、追跡者(その力を求める人々)から隠す.

キーパーたちは、世界が異常な世界の禁断の力を掴まないように、その力を求める人々からその力を確実に隠すために存在する。

世の中には、禁断の力(不老不死であれ精霊のランプであれ)を得たいと思っている人達がいる.
キーパーは、そういう力をそれらの者に渡さないようにしているようだ.

927 :名無しさん@英語勉強中 :2022/08/08(月) 04:03:30.38 ID:7JU4jvqj0.net
pursuant 〔捕まえようとして〕後を追う
party 人々

pursuant parties 追い求めている人々

928 :名無しさん@英語勉強中 :2022/08/08(月) 04:22:22.54 ID:1rwido1F0.net
>>926-927
レスポンスありがとうございます。
<〔捕まえようとして〕後を追う> <party> この部分を具体的に解説して頂けると助かります。

929 :名無しさん@英語勉強中 :2022/08/08(月) 04:31:06.15 ID:7JU4jvqj0.net
>>928
ナメック星に、ドラゴンボールがある
各村の長老やナメック星人がそれを守っている
彼らが keeper

フリーザ一味やベジータがドラゴンボールを追い求めている
彼らが pursuant parties

930 :名無しさん@英語勉強中 (ワッチョイ eb89-qy/x):2022/08/08(月) 05:16:25 ID:4m/2kPIn0.net
>>925
質問者の掲げた課題文のすぐ前の2つの文も含めて、順番に検討してみる。そうすれば、
今回の課題文 (3) の意味がすんなり理解できると思う。


(1) 課題文の2行ほど前の文:
Ensuring the world does not grasp onto the forbidden powers of the anomalous world,
The Keepers are there to ensure that such powers are hidden from those who seek it.

意味合い(和訳ではない):異常な世界(the anomalous world)の「禁じられた力(魔力?)」を全世界のいろんな悪者(the world)
が掌握しようと躍起になってるんだろ?それを The Keepers という奴ら(たぶん正義の味方?)
がその魔力を隠して、悪者がその魔力を奪ってしまうことのないように頑張ってるんだ。

(2) 課題文の1行ほど前の文:
Considered as one of the most blackest task forces of the Authority,
it is alleged that The Keepers contain the most darkest and corrupting anomalies
within the Authority database.

意味合い(和訳ではない):The Keepers は当局(the Authority)の一部として、最高に邪悪で腐敗した異常勢力を
当局のデータベース内に「封じ込め」(contain)る役割を持っているのだとされている。

(3) 今回の課題文:
★Whether it be Immortality or the Genie's Lamp,
The Keepers are the ones to contain and hide them from pursuant parties.★

意味合い(和訳ではない):The Keepers は、them (つまりそういう邪悪な力) を
封じ込め(contain)、「そういう力を掌握しようと躍起になってる連中(pursuant parties)」
から隠す役割を背負った奴ら(the ones)なのだ。

こういうふうに、課題文の少し前から順番に読んでいけば、今回の課題文の意味は
明らかだろう?俺みたいにこのゲームらしきものをやったことがない無知な人間でも、
大体の意味は分かるような気がしてくる。

931 :名無しさん@英語勉強中 (ワッチョイ eb89-qy/x):2022/08/08(月) 05:21:13 ID:4m/2kPIn0.net
>>925
一言で言うと、今回の課題文 (3) は、その直前の (1) と (2) にすでに書いてあることを
少し別の言葉で言いかえているに過ぎない。

今回のものに限らず、このスレでもネット上の他のサイトでも、質問者は
いま抱えている一つの文だけを取り上げて、頭を抱えている。しかし実は、
その直後とか直前に書いてある数行を順番に理解すれば、その課題文は
どうせ同じことを別の言葉で言い換えているに過ぎない場合がほとんどだ。

932 :名無しさん@英語勉強中 (ワッチョイ eb89-qy/x):2022/08/08(月) 05:23:05 ID:4m/2kPIn0.net
>>927 >>929 の回答者は、すでにこのゲームの全体を知っているみたいで、
最初から正解を出しているようだね。

933 :名無しさん@英語勉強中 (ワッチョイ eb89-qy/x):2022/08/08(月) 05:29:03 ID:4m/2kPIn0.net
>>925
pursuant に関しては、それを辞書で引くと pursuant が "pursuant to 名詞" の形で
しか紹介されていないので、今回の文脈では意味がずれているように見えてしまう。

しかし今回の pursuant parties の場合は、そういう意味ではなくて pursuant の
本来の形である pursue という動詞の本来の意味で使われている。だから

pursuant parties = parties [who are] pursuing the powers
(そういう邪悪な力を追いかけている、掌握しようと狙っている連中)

というわけだな。こういう pursue を使った簡単な例文としては、
The police are pursuing the criminal.
というものを考えればいいよね。

934 :名無しさん@英語勉強中 :2022/08/08(月) 05:35:28.94 ID:4m/2kPIn0.net
>>925
言うまでもないかもしれんけど、
The Keepers are the ones to contain and hide them from pursuant parties.」
ここでは、
The Keepers are the ones to
   (1) contain them (ここで them を補えばわかりやすい)
     (them というのは、the most darkest and corrupting anomalies あるいは
     the forbidden powers of the anomalous world を指している)
  and
   (2) hide them from pursuant parties.
     (ここでの them も、上で書いた them と同じものを指している)

935 :名無しさん@英語勉強中 :2022/08/08(月) 05:46:14.72 ID:4m/2kPIn0.net
>>925
The Keepers are the ones to 動詞.
とくれば、ああこれは
You're the one to 動詞. (あなたこそが〜するという役割を持った人だ)
という決まり文句とと同じだな、と思えばいいよね。the one は単数だけど、
今回は the ones と複数になっているだけだ。

Even if ★you’re the one to end a relationship★, you can still find yourself heartbroken.
(だいたいの和訳: 自分から別れ話を切り出す場合でも、やっぱり意気消沈するのだ)

上の文はネット上にある新聞記事の一節だ。恋人同士が別れるとき、
どちらかが別れ話を切り出す。相手から急に「別れてくれ」と言われると、
言われた方は意気消沈する。でもこの記事では、別れ話を切り出す方も
意気消沈するのだ、と言っている。

936 :名無しさん@英語勉強中 :2022/08/08(月) 07:15:49.38 ID:4m/2kPIn0.net
>>925
pursuant parties という表現は、英語のままで理解するよりも、これをフランス語に
直訳するとわかりやすい。俺の直感では、この英語を書いた人はもしかして
フランス語を母国語としている人ではないかと思う。

フランス語の辞書(しかも古い英語ではなくてあくまで現代フランス語の辞書)で
これを確かめると、ずばりの表現がずらずらと出てくる。

(1) poursuivant, poursuivante (英語の pursuant に相当)
これはもともとは poursuivre (英語の pursue に相当) という動詞の現在分詞みたいな
形であり、その意味合いは英語に直訳すれば pursuing とか pursuant に相当する。
この単語は、名詞にも形容詞にもなる。

poursuivant = Personne qui exercise des poursuites judiciaires という意味であり、その
類義語としては demandeur がある、と書いてある。(Le Petit Robert, Editiion 2017)

形容詞として使った時の用例として、この辞書には la partie poursuivante を挙げている。
これを英語に直訳すると the pursuant party だけど、the pursuing party と訳すこともできる。

というわけで、今回の pursuant parties は pursuing parties と書いても同じ意味になる
(少なくともこの英文を書いた人は、そういう意味で使っている)。もっと
英語らしい表現にすれば、単に pursuers と言えばいいところだろうと思う。

937 :名無しさん@英語勉強中 :2022/08/08(月) 07:34:18.97 ID:4m/2kPIn0.net
>>925 の課題文の source site 内にある別のページを見ると、次のようなことが書いてある。

Designation: Delta-1
Codename: The Keepers
Role: Asset Protection, Asset Retrieval
Established: 1895-1896

Description: Delta-1 is a combined asset retrieval and protection task force of anomalies
that are considered potentially exploitable or otherwise sought after by entities of interest,
initially established as an acquisition task force retrieving items suspected to be anomalous
in nature from the Imperial Japanese Army.

つまり、19世紀の終わりごろにこの The Keepers という軍団(みたいなもの)が
設置され、それを Delta-1 と呼んでいたのだが、これはそのころに東南アジアに
軍事的に進出・侵略を続けていた大日本帝国から大切な asset (財産) を
奪い返すことを使命としていた、というようなことを書いている。

当時の日本と東アジアや東南アジアを示す地図も載せている。本当にこんな
The Keepers という名称の組織が設置されていたのかどうか知らないけど、
このゲームって、要は日本に対する negative campaign のためのゲームみたいだな。
韓国人・北朝鮮人・中国人が喜んでこのゲームに参加してるんだろうな。

それから、「有色人種のくせに」白人に盾をつき、白人による植民地支配から
アジア・アフリカ諸国が続々と独立するに至った現象の原因を作った
日本人が憎くてたまらん白人たちの怨念がここにも現れているようだ。

こんなゲームをもしも日本人までが喜んでやっているとしたら、アメリカによる
War Guilt Information Program (WGIP) が今でも強力に生き続けているということだな。
日教組がまたそれを後押ししているに違いない。

938 :名無しさん@英語勉強中 :2022/08/08(月) 07:43:21.62 ID:2kTEXF3W0.net
>>926
キーパーたちが抑え込み、追跡者(その力を求める人々)から隠す.

は間違いだよ。したがってその後に訳も違っている。
何が

> キーパーたちは、世界が異常な世界の禁断の力を掴まないように、その力を求める人々からその力を確実に隠すために存在する。
>
> 世の中には、禁断の力(不老不死であれ精霊のランプであれ)を得たいと思っている人達がいる.
> キーパーは、そういう力をそれらの者に渡さないようにしているようだ.

なんだよ?違うだろ。

containとhide them fromの両方の目的語にpursuantがなってるんじゃない?
whetherの部分が難しくて、whether due to…のことなんじゃないの?
主節は、キーパーたちは、追跡者を押しとどめたり、追跡者からそれらを隠すのが仕事だと。
従属節のitは事情を指すit で、意味は大してなく、
よくわからんけど不死であるからなのか、それともランプを使うからなのか、



又は、上に書いたこととwhether it..の部分についてちがう解釈だけど、
the gate keepersがitを守る存在で、
それが永遠の命であろうがなんでも叶えてくれる魔法のランプであろうが、
キーパーというものは、それらを追い求める者を食い止めたり、それらを
隠してしまうものだ。

でしょ。ここの皆さんの書き込みを見て思ったことは、
この人たち付和雷同さんたちだということ。
自分の頭でかんがえることと、普段からものを考えなよ。
昔はものを覚え思はざりけりの皆さん。

939 :名無しさん@英語勉強中 :2022/08/08(月) 07:47:35.30 ID:2kTEXF3W0.net
質問者もあれだね。普段から考えてない。
考えてないからここに話題を提供してくださっている
大切な貢献者なんだがね

940 :名無しさん@英語勉強中 :2022/08/08(月) 07:57:03.21 ID:4m/2kPIn0.net
>>938
>>containとhide them fromの両方の目的語にpursuantがなってるんじゃない?

俺は質問者じゃなくて回答者の一人だけど、あなたの回答を読んでなっとくした。
あなたの言う通りだな。
[contain and hide them from] pursuant parties
というわけだな。僕も間違っていた。ありがとうね。

941 :名無しさん@英語勉強中 :2022/08/08(月) 08:03:25.25 ID:4m/2kPIn00808.net
>>938
>>whetherの部分が難しくて、whether due to…のことなんじゃないの?

これにも納得した。あなたの言う通りだな。ありがとう。俺もこの部分がどうしてもわからなかったが、
そもそもこの英文を書いた人が少し変な英文を書いていたんだな。現にこの著者は別のところで、
Considered as one of the ★most blackest★ task forces of the Authority,
などという文章も書いてるからね。かなり上手な英文を書いてはいるけど、
殴り書きでどんどん書き続け、あまり推敲をする暇もなかったんだろうな。

小説家などが書いたんだったら、原作者じゃなくて編集者が後で何度も
推敲するからこそ、読者は完ぺきに近い英文が読めるんだもんな。
ブログを書く人は、自分一人で何もかも無料でやってるから、少しくらい変な
英文も書くだろうね。

942 :名無しさん@英語勉強中 :2022/08/08(月) 08:23:04.99 ID:B9IED6IF00808.net
 h

943 :名無しさん@英語勉強中 :2022/08/08(月) 08:53:53.11 ID:EE8ZSDFI00808.net


944 :名無しさん@英語勉強中 :2022/08/08(月) 09:29:10.79 ID:dGd2KO9a00808.net


945 :名無しさん@英語勉強中 :2022/08/08(月) 09:43:45.10 ID:1hO15afl00808.net
ネトウヨは妄想が激しすぎる

946 :名無しさん@英語勉強中 :2022/08/08(月) 10:51:40.25 ID:A5qrZQU+00808.net
>>940-941
いや、>>930を読めば、containの目的語はpursuant partiesではなくthemでしょ
あとdue to なんてどこにも書いてないのを勝手に足したらダメでしょう
>>938の上は間違ってて下が合ってるんだけど、なぜか上に納得してるようなので一応…

947 :名無しさん@英語勉強中 :2022/08/08(月) 11:03:30.62 ID:1rwido1F00808.net
>>937
確かにページは合ってはいるけどゲームではない。
SCP財団という怪奇現象を確保して収容している研究所という奴があるんだけど、
それの派生したのがRPC機構で、財団同様同じことをやっていて、
いわゆる「みんなで世界観を作りながら執筆する」コミュニティなんだ。

今回のこの特殊機動部隊であるキーパーズは化け物を収容するために活動している部隊の一つ。

>>938
RPC機構の世界観では「オブジェクト」と呼ばれる何か妖怪とか不思議な能力を持つ収容を回収を目的にしているので、恐らく後者かなと。

>>941
ちょっと擁護すると、コミュニティは原則として批評家(校正者)からチェックを受ける必要がある
推敲した上で記事として投稿されているので、この著者は説明するのがちょっと諄いのが癖。

他にも色々とレスポンス頂いて助かります。
とりあえず2番の部分はフランス語説もしくは938説の部分を両方追って
直接著者に聞いてみようと思います。

948 :名無しさん@英語勉強中 :2022/08/08(月) 11:11:00.15 ID:7JU4jvqj00808.net
>>938
contain の目的語が pursuant parties なら、
意味としてはより自然になるけど、
文のロジックとしては、めちゃくちゃ悪文にならないかな
(少し前に私が質問した、2つの意味に取れてしまう悪文の、凶悪なバージョン)

>>936
で指摘されているように、これはnative speakerが書いた文章ではないのかもしれない

949 :名無しさん@英語勉強中 :2022/08/08(月) 11:22:25.22 ID:4m/2kPIn00808.net
>>946
あなたがそう思うのも無理はなく、俺も最初はそう思ったんだよね。
でもここでは、the powers というのが「邪悪な力」つまりたぶん「魔力」みたいな
もんだろうと俺は想像しているけど、それが them で置き換えられているんだよね。

そしてその魔力みたいなものを pursuant parties つまりいわば「敵」たちが狙って
るんだよね。その魔力 (them) を奪って、それを駆使して世界を牛耳ろうとしている。

The Keepers という正義の味方は、その魔力(them, the anomalies)を
奪って自由に利用しようとしている連中(pursuant parties)に対して、
"contain and hide them (= the anomalies, the powers) from" pursuant parties
しようとしているというわけ。

contain というのは、contain the powers と言ってしまうとその魔力を
封じ込めて、その魔力が働かなくさせてしまうという意味合いになってしまう。

しかしここではそうではなくて、その魔力 (them, the anomalies) を自由に駆使したい
と当局 (the Authority) も pursuant parties (敵たち)も思っている。敵たちが
その魔力を奪うことのないように、contain (pursuant parties)
and hide them (= the powers) from pursuant parties したいと思っているわけだ。
そういうふうに解釈することによってはじめて、
"contain and hide them (= the powers) from" pursuant parties
という string of words がしっかり理解できる。

もしも contain the powers と理解してしまうと、
contain them (= the powers" and hide them (= the powers) from pursuant parties
という意味で contain and hide them from pursuant parties を理解するのは少し苦しくなる。

950 :名無しさん@英語勉強中 :2022/08/08(月) 11:22:53.47 ID:4m/2kPIn00808.net
>>946
さらに、すでにチラッと述べたように、contain という動詞の使われ方を考えると
contain the powers と言ってしまうと、その powers を封じ込める、つまり
動けなくさせる、無力にさせる、というような意味になるだろう?
だからこそ、the Cold War の時代にはソ連を contain しようとアメリカは息巻いていた
わけだ。そういうときに containment policy とか何とかいう言葉が流行っていたはずだ。
つまりソ連を封じ込めて、無力にさせる、というか、ソ連以外の国々に
共産勢力が飛び火しないようにする、という意味になる。

そういうときの contain the Soviet Union という使い方と
contain the powers (魔力という意味での powers) とが対応しない。
しかし contain pursuant parties と言えば「そういう敵どもを封じ込めて
動けなくさせる、無力にさせる」という意味になって、contain という
言葉の使い方がここで生きてくる。

そういうことがすべて、例の回答者である >>938 は理解していて、しかも
このゲームの全体の雰囲気を最初から呑み込んでいるんだろうと俺は思った。
だから彼の説明に納得した。俺は、そこまで contain という言葉をしっかり理解して
いなかったし、"contain and hide them (= the powers) from" pursuant parties という
言葉の並びに気づかず、"contain them (= the powers) and hide them from pursuant parties"
というふうに苦しい解釈をしてしまっていて、自分でも気持ち悪いけど
しかたがないか、と思っていた。

それから、whether due to A or B の due to は、最初の原文を書いた人の
英文の書き方が下手なだけであって、おそらくは時間がなくてきちんと
推敲する時間がなかったんだろう。明らかにこれは due to がないと変だ
と俺も納得したわけだ。

951 :名無しさん@英語勉強中 :2022/08/08(月) 11:32:33.02 ID:4m/2kPIn00808.net
>>947
怪奇現象を研究したり収容したりする研究所の活動に参加しているわけか。
すごく本格的だな。ゲームだなんて言って、ふざけた想像をしてしまって、
申し訳ない。というのも、ゲーム関係の英文について質問する人が
物凄く多いので、ついついその延長線上にあると思ってしまっていた。
大変、失礼してしまった。

952 :名無しさん@英語勉強中 :2022/08/08(月) 11:57:31.30 ID:1rwido1F00808.net
>>951
まぁ、説明不足だったのは質問者である私の落ち度ですから大丈夫です。
とりあえず今回の文は具体的な情報を除外して頂きたい。

原文が3行なので謳い文句だと思ってください。

戦争云々も関係ないので、科学では説明できないもんを兵器運用するのはまずいから盗む。
そんだけなので・・・。それほど深い意味はないです。

953 :875 :2022/08/08(月) 12:07:58.32 ID:26Z+NE4200808.net
>>898
>895
>今回の英文のthatは接続詞だよ。

「接続詞」説はそれはそれで首肯できるけれども、だからといって、「関係代名詞」説
は成り立たないという証明にはならない。

ちなみに、>>888は「その that は関係代名詞じゃないって、いろんな人がすでにこの
スレで言いつくしてる」と言っているが、「関係代名詞じゃない」と主張する書き込み
など一つもないのである。そもそも「関係代名詞」という言葉さえ出ていない。だから
こそ、自分は>>875でその可能性に言及したのである。

文法的もしくは理論的、論理的にこの that が関係代名詞ではないとする根拠は何一つ
示されてはいない。ひたすら、「この that は接続詞だー」と主張されているにすぎない。
もちろん、自分も「この that が接続詞ではない」とする論拠を持ち合わせてはいない
けれども(笑)。

というわけで、行きがかり上、自分はなお「関係代名詞」説に固執させていただく。 (´·ω·`)

(続く)

954 :875 :2022/08/08(月) 12:09:32.12 ID:26Z+NE4200808.net
(続き)
ということで、>>877にさらに用例を追加しておきます。

今度はフレーズ検索で「what have I done that should」にして、より問題の文章に
近いものを探してみた。

「疑問詞の what + それを先行詞とする関係代名詞の that」のパターンが現代英語
でもそれなりに使われているのがうかがえる。

1. The Congressional Gold Medal for Singer Marian Anderson
1977/03/08 — I don't like to use the word protesting but my reaction was, what have I done that should bring this onto my heart?

2. Holy Ghost Letter - Google ブック検索結果
Idowu Fatorisa · 2011 · &#8206;Religion
What have I done that should warrant a sanction? What haveI done that constituted anactof betrayal? I leave youto judge andmake your inference.

3. The Works of Beaumont and Fletcher: The Text Formed from a ...
Francis Beaumont, &#8206;Alexander Dyce · 1854
A tough hen : sure , it was ne'er known at Babel ; What have I done that should deserve this trial ? For they sold no apples , and this was made for I never ...

4. What is the best way to show others you are not accepting ...https://www.quora.com › What-is-the-best-way-to-show-o...
Now that I'm in my 50's, I take stock of my life in several ways, one being asking myself, “What have I done that should have been done differently?

955 :875 :2022/08/08(月) 12:11:15.68 ID:26Z+NE4200808.net
念のため言っておくと、

もちろん自分は、今回の質問の元々の出所が、『英文解体新書』などの英語参考書で
有名な北村一真氏のツイッターであることを知っている。
そして、8月5日(金曜日)の夜9時の時点で、同氏がこの that は接続詞であると回答
していることも知っている。

8月5日@Kazuma_Kitamura

正解は「副詞節を作る接続詞」です😇 予想以上に低い正答率となりました。

(to me)
should have been decreed (V)
the love of two such women(S)

というthat節内の倒置構文を見落とし、thatがshould have beenの主語だと考えて
「関係代名詞」を選んだ人が多かった結果でしょうかね😇 https://twitter.com/kazuma_kitamura/status/1555200530565431304

posted at 21:08:09

2022年08月05日(金)14 tweetssource
8月5日@Kazuma_Kitamura

ちなみに、What have I done that…should…?というのは「…するとは、…するなんて」
を意味するthatの副詞節が用いられる典型パターンの一種で、「なんの因果で…するのか」
などとも訳されます😇

posted at 21:08:09
(deleted an unsolicited ad)

956 :名無しさん@英語勉強中 :2022/08/08(月) 13:55:47.72 ID:1hO15afl00808.net
勝手にそう思ってればいいじゃん

957 :名無しさん@英語勉強中 :2022/08/08(月) 14:43:45.28 ID:A5qrZQU+00808.net
>>949-950
「the keeperが力を無力化(contain)するのはおかしい」ということだと思うんだけど、>>930を見ると、その前文に
The Keepers contain the most darkest and corrupting anomalies within the Authority database.
と書いてありますよね
だから問題の文
The Keepers are the ones to contain〜
でcontainの目的語がthemになっても全く問題ないというかむしろthemでしょ?

そもそもここでのcontainに無力化、制圧という意味は特に無く(そういう香りはあるかもしれないが)、ほぼhide と同じ意味で使われてると思うんですよね。haveでもいいけど。
とにかく、直前でthe keeper contain powersと言ってるので、次の文でthe keeper contain …と来て「この目的語がpowersだとおかしい」というのはしっくり来ないと思うんですがどうですか?

958 :名無しさん@英語勉強中 :2022/08/08(月) 14:48:10.25 ID:7JU4jvqj00808.net
>>957
私は同意見です

959 :名無しさん@英語勉強中 :2022/08/08(月) 14:51:25.53 ID:4m/2kPIn00808.net
>>947
実は僕も、今さっきいつもの用事で遠方まで歩いて往復しながら考えてたんだけど、
あなたの言う通りかもしれないと思い始めてた。

>>The Keepers contain the most darkest and corrupting anomalies within the Authority database.

こういう文もちゃんとあったとは気づかなかった。もう少しきちんと調べてから
コメントすべきだったと思ってる。いずれにしても、あなたのおかげで僕も
勉強になった。contain という言葉の使い方や意味合いが、僕自身、ろくにわかっても
いなかったんだということに気づいた。僕ももっとたくさん英文を読み続けていかないと、
この程度の簡単なことについてさえ自信を持ったコメントが書けないことを悟った。

960 :名無しさん@英語勉強中 :2022/08/08(月) 14:55:55.98 ID:4m/2kPIn00808.net
>>959 は、>>957 宛て。

961 :名無しさん@英語勉強中 :2022/08/08(月) 18:16:21.60 ID:/yTXi4wP00808.net
この濃い青のマフラーはどうですか?
→ How about this dark blue scarf

duolingoで出てきた問題なんだけど
この英文動詞がないように見えるんだけど
省略しないで書くとどうなるんでしょうか

962 :名無しさん@英語勉強中 :2022/08/08(月) 18:26:42.81 ID:pXXrUYzPd0808.net
>>961
How (do you think) about

963 :名無しさん@英語勉強中 :2022/08/08(月) 18:46:01.87 ID:/yTXi4wP00808.net
>>962
ありがとうございます

964 :名無しさん@英語勉強中 :2022/08/08(月) 18:53:36.65 ID:WBB3yaWl00808.net
How aboutはそのまま覚えておけばいいと思う

965 :名無しさん@英語勉強中 :2022/08/08(月) 18:54:21.77 ID:WBB3yaWl00808.net
How about (if) we go out for lunch?
というのもあってifが抜けてると難しいよね

966 :名無しさん@英語勉強中 :2022/08/08(月) 19:07:22.00 ID:4m/2kPIn00808.net
「どう思う?」を「How do you think?」というのは間違い

上記のタイトルでネット上に記事がある。

967 :名無しさん@英語勉強中 :2022/08/08(月) 19:12:40.55 ID:4m/2kPIn00808.net
ネット上の wordreference.com という英語質問サイト上の回答より

"How do you think?" - With my brain, like everyone else.
"What do you think?" - I think that "what" is correct in this context.

上のコメントでもやはり、「どう思うか?」という意味で How do you think?
と言うのは間違いだということだ。ただし、次の場合には how が正しい。

Questions like "How do you think your mother will feel if you don't call her" are different to me.
I understand the "do you think" as an insertion. The core sentence is "How will your mother feel if you don't call her"

上記のように英語ネイティブたちが言っている。

968 :名無しさん@英語勉強中 :2022/08/08(月) 19:16:31.65 ID:tMB0buQ300808.net
こういうやり取りで思うのがどの程度の間違いなのかが分からないんだよな。全く理解できなくなるの?

969 :名無しさん@英語勉強中 :2022/08/08(月) 19:19:44.23 ID:4m/2kPIn00808.net
結論として

Tom についてどう思うか?
と言いたいときは
(1) What do you think about Tom?
であって How を使ったら間違い。もし How do you think? と言ってしまうと、
「どうやって考えてるの?」という意味になってしまって、答えは「頭で考えてるのさ」
と言いたくなるそうだ。でも
(2) How do you feel about Tom?
ならば正しい。
というふうにいろんな英語ネイティブがネット上で言っている。

リンクを貼ると最近の5ちゃんねるはすぐにアクセス禁止をかけてくるので、
最近はリンクを貼るのをやめている。悪しからず。リンクは自分で探してほしい。

970 :名無しさん@英語勉強中 :2022/08/08(月) 19:22:14.67 ID:4m/2kPIn00808.net
>>968
そんなの、間違いは間違いなんだよ。しかし間違いでも意味はわかってくれる。
たとえば
「私は昨日、自転車で学校に行きました」
これは正しい。しかし
「私の昨日、自転車は学校が行きますた」
これは間違い。でも意味はわかってくれる。

どんな無茶苦茶な言葉でも、わかってはくれる。

971 :名無しさん@英語勉強中 :2022/08/08(月) 19:29:07.10 ID:tMB0buQ300808.net
この程度の間違いを英語ネイティブはどう感じるの?
日本語で上げてくれた例を、日本人なら「ました」の滑舌がよくなくて「ますた」になっちゃったね。以上で特に何も感じずに終わるけど

972 :名無しさん@英語勉強中 :2022/08/08(月) 19:42:57.66 ID:7f+6U52600808.net
どうも感じないよ よくあること以上の感覚はない
均一的な日本人の日本語と違ってルーツも歴もバラバラである以上英語は1人1人違うものって認識があるし個性にもなってる
文法は特にそう アクセントとスピードがそれっぽければ通じるし大体変に思われないよ もちろん論文だの公の場だのは別だけど

973 :名無しさん@英語勉強中 :2022/08/08(月) 20:57:19.43 ID:xNDqBivh00808.net
なにも感じない
なにも思わない
どうも感じない
どうも思わない

974 :名無しさん@英語勉強中 :2022/08/09(火) 06:34:33.33 ID:WwgUqbpGa.net
>>971
日本人がI thinkを連発して、考えたこととは思えない、感じただけのことを言うのを聞いて、「日本人はなんと哲学的な人たちなんだろう」と勘違いされる程度には違和感があるそうだ

975 :名無しさん@英語勉強中 :2022/08/09(火) 07:04:55.99 ID:wW3ngtCl0.net
確かに "I think" なんて、英語ネイティブはそんなに頻繁には使わないな。でもそれは単に、
英語ネイティブならそれに似た言い回しを何十通りも使い分けていて単調さを
避けているだけなんだろうな。でも日本人は、そんなにたくさんの表現を使い分けるのが
大変だから、すべて I think で済ませてしまうんだろうな。

It seems (that) S V.
It seems as if S V.
Seems like S V.
It seems like S V.
Looks like S V.
It looks like S V.
Looks as if S V.
I would think S V.
I would say S V.
Seems to me S V.
It seems to me S V.
You would think S V.
Probably S V.
Possibly S V.
Maybe S V.
Could be (that) S V.
Perhaps S V.
I wouldn't be surprised if S V.

その他、たぶん百通りくらい使い分けてるんだろうな。

976 :名無しさん@英語勉強中 :2022/08/09(火) 07:08:36.92 ID:wW3ngtCl0.net
"I think" に似た言い回しを検索してみたら、ちゃんと紹介してくれてるサイトがあるな。

I THINK - synonyms and related phrases
Some (people) say (that) ... I'd say that .../ I would say that ... Well, I must say
If I may say so
I suppose / suspect / feel that...
Personally, I strongly* believe (that) ... /deeply / firmly / truly/ completely/ fully/ honestly / genuinely / sincerely
It's my belief that ...
(その他いろいろ紹介してくれている。)

977 :名無しさん@英語勉強中 :2022/08/09(火) 07:19:44.25 ID:wW3ngtCl0.net
I think に似た言い回し

In my opinion
As far as I’m concerned – This phrase is often used in a more authoritative sense.
I believe that…
I am of the opinion that…
It is my belief…
It seems to me/It appears to me
To my way of thinking/In my way of thinking
I honestly think that/ I honestly believe that…
I am feeling that/I feel that – This is a more personal phrase which moves from thoughts and takes them to a more emotional level. This is a good phrase to use when trying to convince someone of something.
I assume that – In this example, the speaker would use the phrase when they are not 100% certain of something, to assume means to carry the belief that something is correct.
From my point of view
If you ask me…
My thoughts on the matter are…
I consider…
As far as I can see/ as far as I can tell
To my mind/ In my mind
It is my view/It is my opinion
The way that I see it is…

978 :名無しさん@英語勉強中 :2022/08/09(火) 07:44:50.21 ID:wW3ngtCl0.net
YouTube 上にも、この種の情報を提供している人がいるな。
DO NOT SAY 'I think...' - say THIS instead - 21 more advanced alternative phrases
例えばこの上のタイトルの付いたビデオ。それ以外にもたくさんあるようだ。

至れり尽くせりだな。人間はもう、自分で考えたり調べたり工夫したり
しなくても生きていけるようになったみたいだ、と人はよく言うけど、
本当にそうなったのかもしれない。その結果として、何をしても仕方ない
と思って何もしなくなっている人が、ひょっとしたら増えているのかもしれない。

ネット上での英語質問サイトでの質問も、それに応じてどんどん低下している
ような気がしてしまう。偏見かな?でも、どう考えても質問者は
自分では何も考えず、何も調べず、辞書も引かず、本屋へ行って
本を自分で探すこともせず、1時間ほどかけて自分で何か探すなんてことをしないで、
ネット上で質問して1分以内で回答がほしいと思う人が増えたと思ってしまうのは、
俺だけか?

979 :名無しさん@英語勉強中 :2022/08/09(火) 13:16:27.81 ID:MEMAaM+o0.net
テニス選手のプレカンとか見てるとI'd say 使ってる人多いイメージ

980 :名無しさん@英語勉強中 :2022/08/09(火) 17:24:39.98 ID:wW3ngtCl0.net
Facebook が盛んに話題になっていて、何冊もの本が書かれたが、1年前に英語で刊行された
"An Ugly Truth: Inside Facebook's Battle for Domination" (by Sheerra Frenkel and Cecilia Kang)
の邦訳がつい最近になって日本で販売されており、その結果、日本でも話題になっている
ようだ。それを読んでいて、面白い(というか口語らしく崩れている)表現が
見つかった。

“I don't know about you,” he (= Mark Zuckerberg) said, not bothering to hide his impatience,
“but ★I'm used to when people legally certify they're going to do something,
that they'll do it★, and that was a mistake," he said in the ...
(その本の p. 160 より)

この上に挙げた一節は、ネット上でも見つかる。

981 :名無しさん@英語勉強中 :2022/08/10(水) 06:16:37.74 ID:zfWAQlkna.net
トランプ大統領がなんて云ってるのか誰か通訳してください(´・ω・`)
ttps://www.youtube.com/watch?v=ouG9cVhjPds

982 :名無しさん@英語勉強中 :2022/08/10(水) 06:57:43.13 ID:Q5RMADMm0.net
>>981
通訳は嫌だけど、聴き取りだけはしておく。細かいところが聴き取れない。
あちこち間違ってるだろうとも思う。

DELIVERY BOY: And this 555 deal's really popular. Mr. Trump! Here you go of
three mini pizzas five dollars each.

TRUMP: Tell you what. I'll counter that offer with an even better one.
Here's the deal: You give me those three pizzas.
Only I'll give you just five dollars apiece.

DELIVERY BOY: Sure. Okay.

TRUMP: Still got it, Donald. Still got it.

ANNOUNCEMENT: Call now and get three more mini ??? pizzas for just five bucks each.
Get the door. It's the Domino's 555.

983 :名無しさん@英語勉強中 :2022/08/10(水) 07:02:15.17 ID:zfWAQlkna.net
トランプ:何を教えてください。 私はその申し出にもっと良いもので対抗します。

これが取り引きです:あなたは私にそれらの3つのピザをくれます.
私だけがあなたに 5 ドルだけあげます。

全く意味が判らない(´・ω・`)

984 :名無しさん@英語勉強中 :2022/08/10(水) 07:05:45.00 ID:zfWAQlkna.net
DELIVERY BOY: そして、この555の取引は本当に人気があるんだ。トランプさん!さあどうぞ
3枚のミニピザを 5ドルずつ

トランプ:こうしよう。そのオファーに対抗して、もっといいものを提供しよう。
取引はこうだ:その3枚のピザを私にくれ。
ただ、1枚5ドルでいいよ。

配達員: もちろんです。

まだあるぜ、ドナルド。まだある

アナウンス: 今すぐお電話を!ミニピザをあと3枚、1枚5ドルでご提供します。
ドアを開けろ ドミノ555だ


DeepL翻訳でもやっぱりよくわからない(´・ω・`)

985 :名無しさん@英語勉強中 :2022/08/10(水) 07:10:05.12 ID:zfWAQlkna.net
ピザ3枚セットなら1枚5ドル
5ドルのピザ1枚を3枚買う

この違い?アメリカンジョーク?面白くないわFuck!(´・ω・`)

986 :名無しさん@英語勉強中 :2022/08/10(水) 07:12:35.90 ID:zfWAQlkna.net
アメ公はきっとこのCM観てドカンドカン大爆笑してるんだろ?
アメ公ってイギリス人よりジョークのセンス無くない?もう一回アメ公と戦争した方がええわ(´・ω・`)

987 :名無しさん@英語勉強中 :2022/08/10(水) 08:30:21.86 ID:v3sh3ovm0.net
これは同じ取引きを口八丁で混ぜっ返して
結果金を払わないで済ましたことを
本人がカメラに向かって自慢するっていう感じなのか

988 :名無しさん@英語勉強中 (アウアウウーT Sa55-292U):[ここ壊れてます] .net
そういや金払ってないな?真似して金払わない奴が出て問題にならんのか?
つーか、トランプ大統領はピザハットのCMにも出てるけど節操ないな?
つーか5ドルって安いな?日本だとピザMサイズ1枚1000円ぐらいとられるぞ?(´・ω・`)

989 :名無しさん@英語勉強中 (ワッチョイW 019d-Owjd):[ここ壊れてます] .net
職権乱用だ許さないぞ!って英語でオナシャス

990 :名無しさん@英語勉強中 (ワッチョイ eb89-qy/x):[ここ壊れてます] .net
>>989
You're abusing your authority. I won't forgive you.

参考:
(1) “★You're abusing your authority★ as a sworn officer of the law,” I informed him.
“I wasn't even barely speeding.” “Yeah, yeah. Listen, where were you the ...
(ネット上で読める小説の一節)

(2) "I Won't Forgive You" というタイトルの歌がある。

991 :名無しさん@英語勉強中 (ワッチョイW 019d-Owjd):[ここ壊れてます] .net
>>990
thx
くだらない質問すまんかったw

992 :名無しさん@英語勉強中 :2022/08/10(水) 10:52:48.56 ID:Q5RMADMm0.net
>>981
和訳は嫌いだが、一応はやっておくか。

DELIVERY BOY: And this 555 deal's really popular. Mr. Trump! Here you go of
three mini pizzas five dollars each.
配達夫: それでこのスペシャル555セットは、ほんとに売れてるんだ。
(あっ)トランプさん。1枚5ドルのミニピザの配達にあがりました。

TRUMP: Tell you what. I'll counter that offer with an even better one.
Here's the deal: You give me those three pizzas.
Only I'll give you just five dollars apiece.
トランプ: あのね、そのスペシャルセットよりもさらにいいものを
提案してやるよ。こんなのはどうだ?そのピザ3枚くれたら、
1枚当たり5ドル払うよ。

DELIVERY BOY: Sure. Okay.
配達夫: ああ、いいですけど。

TRUMP: Still got it, Donald. Still got it.
まだ俺はいけてるな。まだやれるぞ。(歳は取ったけど、取引の才能が今でもあるぞ。)

ANNOUNCEMENT: Call now and get three more mini ??? pizzas for just five bucks each.
Get the door. It's the Domino's 555.
コマーシャル: 今すぐ電話して、ミニピザをあと3枚、注文してみてください。
1枚あたりたったの5ドル。
ドアを開けてくれますか?ドミノピザからのスペシャル555セットをお届けに参りました。

993 :名無しさん@英語勉強中 :2022/08/10(水) 11:00:10.10 ID:Q5RMADMm0.net
ミニピザ3枚を一度に買うと1枚当たり500円で買えるのだが、それよりももっと
いいセールスの仕方があるぞ、と自慢げにトランプが言うが、それはすでに
ドミノピザが提供しているものとまったく同じ。

それに気づきもせず、トランプは「歳は取ったけど、今でもまだ若い時の経営手腕は
衰えてないぞ、と自己満足しているというオトボケということだと思うんだが、
単純すぎて面白くも何ともない。

でも、トランプが大統領になったあとにあちこちでオトボケぶりを披露し続けた
みたいなので、どうやら彼はずっと前から頭が単純でトボケていることをむしろ
自虐ネタにして、労働者たちの間で逆にウケ続けていたのかもしれない、という気が
俺にはしている。

994 :名無しさん@英語勉強中 :2022/08/10(水) 11:05:22.82 ID:Q5RMADMm0.net
まあ、アメリカというところは腕力や胆力だけで荒野が延々と広がる西部を
開拓していったという自負を持つ人が多く、知識や教養なんてどうでもよく、
むしろ馬鹿なくらいの方がいいんだ、と自慢さえしている人が多いらしい。

言葉に関しても、正しい英語なんてしゃべらなくてもよく、むしろわざとデタラメの英語を
しゃべった方がカッコいい、と思うアメリカ人もけっこういるらしい。

(このことは、40代くらいのアメリカ在住の英語ネイティブの女性が
俺に対してしみじみとそう言っていた。)

だからこういう単純極まりない馬鹿丸出しと言いたくなるくらいのコマーシャルとか
キャラクターが受けるのかもしれないな。

995 :名無しさん@英語勉強中 :2022/08/10(水) 11:45:28.15 ID:c8P4gRvH0.net
トランプってそういう自虐ネタやるんだなw

996 :名無しさん@英語勉強中 :2022/08/10(水) 11:55:35.38 ID:+afgmdF70.net
And this 555 deal's really popular. Mr. Trump, here you go, uh three medium* pizzas five dollars each. (*edited)

Tell you what. I'll counter that offer with an even better one. Here's the deal. You give me those pizzas, only I'll give you just five dollars a piece.

Sure. Okay.

Still got it, Donald. Still got it.

Call now and get three or more medium one-topping pizzas for just five bucks each. Get the door. It's the Domino's 555.

ビジネスマンで海千山千のトランプはハッタリ(実際は同じことしか言ってない)で配達員を混乱させて気圧された配達員は(何故か)代金をもらわず行ってしまった。「俺の交渉の手腕はまだ錆びてない」

997 :名無しさん@英語勉強中 :2022/08/10(水) 12:15:18.17 ID:Q5RMADMm0.net
>>996
(1) medium
(2) one-topping
上の二つは、なるほどと思った。ありがとう。

(3) 代金を払っていないように見えるけど、これは本当に払っていないのではなく、
クレジットカードですでに電話か何かで払っていたか、あるいは1か月ごとにまとめて
払っているとかしているのだろうと俺は思っていた。

(4) a piece と apiece について
a piece はここでは間違いで、apiece が正しいらしい。それについては、
次のようなネット上の記事がある。

apiece / a piece
When you mean “each” the expression is “apiece”: these pizzas are really cheap
-- only ten dollars apiece.” But when “piece” actually refers to a piece of something,
the required two-word expression is “a piece ”: “This pizza is really expensiv
-- they sell it by the slice for ten dollars a piece.”

998 :名無しさん@英語勉強中 :2022/08/10(水) 12:18:57.63 ID:Q5RMADMm0.net
apiece と書くべきところを a piece と書いてしまうことについては、
たぶん教養ある英語ネイティブでも間違えるんだろうなと思う。

999 :名無しさん@英語勉強中 :2022/08/10(水) 12:24:46.21 ID:+afgmdF70.net
なるほどね、それが言葉で混乱させるトリックなんだろうな

ピザ一枚につき apiece
ビザ一切れにつき a piece

これで値段が大幅に変わってくるし音だけじゃ判別は難しいね

1000 :名無しさん@英語勉強中 :2022/08/10(水) 12:42:14.84 ID:xZWlCcd0d.net
apronって元々はnapronだったけど、a napronがan apronに間違われてそうなったらしいな

1001 :2ch.net投稿限界:Over 1000 Thread
2ch.netからのレス数が1000に到達しました。

総レス数 1001
424 KB
掲示板に戻る 全部 前100 次100 最新50
read.cgi ver.24052200